Income Tax 207677-237743170

You might also like

Download as pdf or txt
Download as pdf or txt
You are on page 1of 133

Income Tax Note 2076/77

Table of Contents
Basis of Tax.................................................................................................................................... 1
1) Definitions ........................................................................................................................ 1
2) Person chargeable to Tax ................................................................................................. 1
3) Calculation of Tax and Tax rate ....................................................................................... 2
2.1 Presumptive taxation .................................................................................................... 2
2.2 Tax on Turnover ........................................................................................................... 3
2.3 Detailed Tax Calculation .............................................................................................. 4
4) Income from Business ...................................................................................................... 6
5) Income from Employment ............................................................................................... 6
6) Income from Investment .................................................................................................. 7
Exempt Amount and Other Concessions .................................................................................... 9
1) Definition ......................................................................................................................... 9
2) Exempt Amount ............................................................................................................... 9
3) Donation, Gift to exempt Organizations ........................................................................ 10
Deductions ................................................................................................................................... 11
1) Definition ....................................................................................................................... 11
2) General Deductions ........................................................................................................ 15
3) Interest ............................................................................................................................ 15
4) Cost of Trading Stock .................................................................................................... 17
5) Repair and Improvement Expenses ................................................................................ 18
6) Pollution control Expenditure ........................................................................................ 18
7) Research and Development Expenditure ....................................................................... 19
8) Depreciation Allowance ................................................................................................. 19
9) Losses from Business or Investment .............................................................................. 25
10) Deduction Not Allowed ................................................................................................. 27
Tax Accounting and Timing ...................................................................................................... 29
1) Method of Tax Accounting ............................................................................................ 29
2) Cash Basis Accounting................................................................................................... 31
3) Accrual basis of Accounting .......................................................................................... 31
Income Tax Note 2076/77

4) Reversal of amount including bad debts ........................................................................ 32


5) Quantification of Amounts ............................................................................................. 32
6) Quantification in Rupees ................................................................................................ 35
7) Indirect payments ........................................................................................................... 35
8) Jointly Owned Investment .............................................................................................. 35
9) Characterization of Compensation Payment .................................................................. 37
10) Characterization of Payments under Annuities, Installment Sales and Finance Leases 39
11) Transfer pricing and other Arrangement between Associates ....................................... 40
12) Income splitting .............................................................................................................. 41
13) General Anti- Avoidance Rule (GAAR) ........................................................................ 42
Long Term Contract ................................................................................................................... 43
1) Definition ....................................................................................................................... 43
2) Recognition of Revenue & Expenses under Long-Term Contract ................................ 43
3) Treatment of loss on completion of Contract ................................................................. 44
Calculation of Net Gain from Assets and Liabilities ............................................................... 49
1) Definition ....................................................................................................................... 49
2) Net Gain from Asset and Liabilities ............................................................................... 50
3) Gain and Loss from Assets and Liabilities .................................................................... 51
4) Outgoing and Net Outgoings for Assets and Liabilities ................................................ 51
5) Incomings and Net Incomings for Assets and Liabilities .............................................. 52
6) Disposal of an Asset or Liability .................................................................................... 52
7) Disposal with Retention of Asset or Liability ................................................................ 56
8) Disposal by way of Installment Sale or Finance Lease .................................................. 57
9) Transfer of Asset to Spouse or Former Spouse .............................................................. 57
10) Transfer of Asset on Death............................................................................................. 59
11) Involuntary disposal of Asset or liability with Replacement ......................................... 59
Special Provision for Natural Person, Banking and Insurance Business .............................. 65
1) Couple ............................................................................................................................ 65
2) Medical Tax Credit......................................................................................................... 65
3) Banking Business ........................................................................................................... 66
4) General Insurance Business ........................................................................................... 68
Income Tax Note 2076/77

5) Investment Insurance Business ...................................................................................... 69


Entity and Distribution............................................................................................................... 72
1) Definition ....................................................................................................................... 72
2) Taxation Principles in respect of Entities....................................................................... 72
3) Distribution by Entities .................................................................................................. 73
4) Taxation of Dividend ..................................................................................................... 74
5) Liquidation of an Entity ................................................................................................. 77
6) Dealing between an Entity and a Beneficiary ................................................................ 78
7) Change in control ........................................................................................................... 79
8) Dividend Stripping ......................................................................................................... 81
Retirement Fund ......................................................................................................................... 83
1) Definition ....................................................................................................................... 83
2) Approval for Retirement Fund ....................................................................................... 83
3) Retirement Payment ....................................................................................................... 84
4) Taxation of Retirement Fund ......................................................................................... 86
International Taxation................................................................................................................ 88
1) Definition ....................................................................................................................... 88
2) Source of Income, Loss, Gain and Payment .................................................................. 89
3) Foreign Permanent establishment .................................................................................. 91
4) Controlled Foreign Entities ............................................................................................ 92
5) Taxation of Non-residents Providing Shipping, Air Transport or Telecommunications
Services in Nepal ...................................................................................................................... 93
6) Foreign Tax Credit ......................................................................................................... 97
Tax Administration and Official Documentation .................................................................. 102
1) International Agreement ............................................................................................... 102
2) Taxpayer’s Right .......................................................................................................... 102
3) Public Circular.............................................................................................................. 103
4) Personal Ruling ............................................................................................................ 103
5) Form of Documentation ............................................................................................... 104
6) Permanent Account Number ........................................................................................ 104
7) Service of Documents .................................................................................................. 105
Income Tax Note 2076/77

8) Defective Documents ................................................................................................... 105


Withholding ............................................................................................................................... 107
1) Withholding by Employer ............................................................................................ 107
2) Withholding on Investment Return and Service Fee ................................................... 107
3) Tax Withholding on Windfall Gain ............................................................................. 108
4) Withholding from Contract Payments.......................................................................... 109
5) Withholding Return and WTH Payment ...................................................................... 110
6) Withholding Certificate ................................................................................................ 110
7) Final Withholding Payment ......................................................................................... 111
8) Inclusions and Credit for Non-Final Withholding Tax ................................................ 111
Installment and Advance Tax .................................................................................................. 113
1) Payment of Income Tax by Installment ....................................................................... 113
2) Return of Estimated Tax Payable ................................................................................. 114
3) Advance tax collection ................................................................................................. 115
Return of Income and Assessments ......................................................................................... 117
1) Return of Income .......................................................................................................... 117
2) Return Filing Not Required .......................................................................................... 117
3) Extension of Time to File Return ................................................................................. 118
4) Tax Assessment ............................................................................................................ 118
5) Jeopardy Tax Assessment ............................................................................................ 118
6) Amended Tax Assessment ........................................................................................... 119
7) Notice of Tax Assessment ............................................................................................ 120
Review and Appeal ................................................................................................................... 121
1) Reviewable Decisions and Procedures:........................................................................ 121
2) Apply for Administrative Review ................................................................................ 121
3) Appeal to Revenue Tribunal ........................................................................................ 122
Charges and Interest................................................................................................................. 123
1) Charges for Failure to Maintain Documents or File Returns ....................................... 123
2) Interest for understating Estimated Tax Payable by Installment.................................. 123
3) Interest for Failure to Pay Tax...................................................................................... 124
4) Charge to be levied ....................................................................................................... 126
Income Tax Note 2076/77

5) Penalty for Making False or Misleading Statements ................................................... 126


6) Charges for Aiding and Abetting ................................................................................. 126
Offences and Penalties .............................................................................................................. 127
1) Offence of Failure to Pay Tax ...................................................................................... 127
2) Offence of Making False or Misleading Statements .................................................... 127
3) Offence of Impeding and Coercing Tax Administration ............................................. 127
4) Offences by Authorized and Unauthorized Persons .................................................... 127
5) Offences of Aiding or Abetting .................................................................................... 128
6) Offence of Failure to Comply with Act ....................................................................... 128
Income Tax Note 2076/77

Basis of Tax

1) Definitions
2(Ka(nga)) Resident Person: with respect to an income year means:

 In the case of an individual, an individual-


(A) whose normal place of abode is in Nepal;
(B) who is present in Nepal for 183 days or more in the 365 consecutive days, or
(C) Posted by GON d abroad at any time during the income year;
 Any partnership;
 In the case of a trust, a trust that-
(A) is established in Nepal;
(B) trustee of trust is a resident person for the income-year; or
(C) is controlled directly or through one or more interposed entities by a person or
persons one of whom is a resident person for the income-year;
 In the case of a company, a company that-
(A) is incorporated or formed under the laws of Nepal; or
(B) has its effective management in Nepal during the income year;
 Government of Nepal, State Government
 A Rural Municipality, Municipality or district development committee;
 In case of a foreign government or a political subdivision of the foreign government, such
an entity-
(A) if it is established under the laws of Nepal; or
(B) has its effective management in Nepal during the income year; and
 Any institution or entity established under treaty; and
 A foreign permanent establishment of a non-resident person situated in Nepal.

2) Person chargeable to Tax


As per section 3, following person are responsible to pay tax:
 Person having taxable income in an income year;
 Foreign permanent establishment repatriating income from Nepal; and
 Person receiving final withholding incomes in Nepal.

Naresh Singh Bhandari


bnsingh5102@gmail.com Page1
Income Tax Note 2076/77

3) Calculation of Tax and Tax rate


There are three types;

2.1 Presumptive taxation


Income tax authority presumes that some persons with the prescribed income and
turnover will have fixed amount of tax liability. These persons are not required to
compute tax in detail and are relieved from tax liability by depositing a fixed amount in
the revenue authority. The following persons pay presumptive tax as per income tax act.

a) Small business person


As per section 4(4), the income tax payable by a resident natural person in any
income year who meets the following conditions, is equal to the amount provided in
section 1(7) of schedule 1.
 A resident natural person’s income for an income year consists exclusively of
income from a business having source in Nepal;
 Has not claimed medical tax credit (Sec 51) and advance tax arising out of
withholding taxes (Sec 93).
 Income from business shall not be more than Rs. 2 lakhs and transaction shall not
exceed Rs. 20 lakhs during an income year.
 The person has opted this provision for that fiscal year.
 The person is not registered for value added tax purpose.

As per section 1(7) of schedule 1, the amount of tax to be paid by such person is as
follows:
Location of Business Amount of Tax(Rs.)
Metropolitan City and Sub Metropolitan City(per annum) 7,500
Municipality (per annum) 4,000
Area other than above (per annum) 2,500

b) Person owning vehicle used for public transportation


If the person owing a vehicle used for public transportation and paying the tax as
follows is a natural person (not necessarily resident person), the tax paid by such
person is final. Such person is also not required to submit the tax return unless
otherwise required by IRD in writing or through public notification. These taxpayers
are also treated as presumptive taxation. As per section 1(13) of schedule 1, the tax to
be paid by such persons for a fiscal year for a vehcile is as follows:

Type of Vehicle Amount


(NPR)
1) Car, Jeep, Van. Microbus

Naresh Singh Bhandari


bnsingh5102@gmail.com Page2
Income Tax Note 2076/77

a) Up to 1300 cc 4,000
b) 1301 to 2000 cc 4,500
c) 2001 cc to 2900 cc 5,000
d) 2901 cc to 4000 cc 6,000
e) More than 4000 cc 7,000
2) Minibus, Minitruck, Water Tanker 6,000
3) Minitripper 7,000
4) Truck, Bus 8,000
5) Dozer, Excavator, Roller, Loader and machinery equipment’s like crane. 12,000
6) Oil Tanker, Gas Bullet, Tipper 12,000
7) Tractor 2000
8) Power Tiller 1,500
9) Three wheeler, Tempo, Auto Rickshaw 1,000

2.2 Tax on Turnover


As per section 4(4ka), Resident Natural Persons who derives income only from business
having source in Nepal with annual business turnover exceeding Rs. 20,00,000 and which
is below Rs. 50,00,000 and not registered for VAT purpose may choose to pay tax as
below:

Nature of Business Registered Area Tax Amount


Person dealing in goods, which are sold on Metro and Sub 7500 + 0.25% of
the basis of commission that does not Metro excess than 2 million
exceed 3% of purchase price or sold with Municipality 4000 + 0.25% of
the value addition up to 3% including excess than 2 million
cigarette, gas. Others 2500 + 0.25% of
excess than 2 million
Person dealing in goods other than specified Metro and Sub 7500 + 0.75% of
above Metro excess than 2 million
Municipality 4000 + 0.75% of
excess than 2 million
Others 2500 + 0.75% of
excess than 2 million
Person dealing in services* Metro and Sub 7500 + 2% of excess
Metro than 2 million
Municipality 4000 + 2% of excess
than 2 million
Others 2500 + 2% of excess
than 2 million
*the person who derives income from consultancy or expert services that are provided by
natural person including, doctor, engineer, auditor, lawyer, sportspersons, artist or
consultant cannot choose to pay turnover tax.

Naresh Singh Bhandari


bnsingh5102@gmail.com Page3
Income Tax Note 2076/77

2.3 Detailed Tax Calculation


a) In case of natural person:
Tax shall be calculated as follows:
Income from Business ***
Income from Employment ***
Income from Investment ***
Income from windfall Gain ***
Total assessable income ***
Less: Allowable deductions ***
Taxable income ***
Less: Allowable reductions ***
Balance taxable income ***
Calculation of tax liability:
In case of individual In case of couple Rate
First Rs. 400,000 First Rs. 450,000 1%*
Next Rs. 100,000 Next Rs. 100,000 10%
Next Rs. 200,000 Next Rs. 200,000 20%
Next Rs. 1,300,000 Next Rs. 1,250,000 30%
Balance Balance 36%
Tax liability ***
Less: Tax credits:
Female tax credit ***
Foreign tax credit ***
Medical tax credit ***
Advance tax paid ***
Tax liability payable ***
*1% rate shall not be applied to the proprietorship firm and natural person having
pension income and contributing to Contribution based Social Security Fund.
*Female tax credit shall not be available to person who elect as couple.

b) In Case of entity
The tax shall be calculated as follows:
Income from business ***
Income from Investment ***
Income from windfall Gain ***
Total assessable income ***
Less: Allowable deductions ***
Taxable income (a) ***
Tax rate (b) ***
Tax Liability (a*b) ***
Naresh Singh Bhandari
bnsingh5102@gmail.com Page4
Income Tax Note 2076/77

In case of natural resident person fulfilling all following conditions, income tax payable
shall be equal to the sum of the amount withheld by employer (under section 87) from the
payment made to resident natural person during year;
 Income shall be from employment having a source in Nepal,
 All the employers are resident and has only one employer at a time during the year
and
 Has claimed medical tax credit for the medical expenses and deduction form
retirement contribution both paid only by the employer and has not claimed reduction
for donation.

Reductions to Natural person


Reductions available for the natural person while calculating the balance taxable income
are as follows:

a) Life Insurance Premium


Life Insurance premium paid by a resident natural person is deductible up to the limit
of Rs 25,000. If both husband and wife have opted as couple, the insurance paid can
be clubbed to claim for deduction, subject to maximum of Rs. 25,000.

b) Employee working in diplomatic areas


75% of foreign allowance is deducted from taxable income of an employee employed
at diplomatic agencies of Nepal situated at foreign countries.

c) Incapacitated natural persons


The minimum exemption limit is increased by additional 50%.

d) Remote Area Benefit


If a resident natural person is working in remote areas as prescribed, then minimum
exemption limit shall be increased as follows depending on remote area category as
follows:
Category A: Rs. 50,000
Category B: Rs. 40,000
Category C: Rs. 30,000
Category D: Rs. 20,000
Category E: Rs. 10,000
The facility is proportionately calculated for period of stay.

e) Additional limit for pension income


If income of a resident natural person includes pension income, the taxable income is
calculated by reducing additional 25% of basic exemption limit or pension amount
included in income whichever is lower from the assessable income.

f) Health Insurance Premium

Naresh Singh Bhandari


bnsingh5102@gmail.com Page5
Income Tax Note 2076/77

If a resident natural person pays health insurance premium to a resident insurance


company, then lower of Rs 20,000 or actual health insurance premium paid by the
person is deductible for tax purpose.

4) Income from Business


As per section 7, the provisions related to income from business are as follows:

 Following amount shall be included in calculating income from business for an income
year:
 Service fee
 Amount derived from disposal of trading stock
 Net gain from disposal of business assets and liabilities
 Net gain from disposal of depreciable assets (in case of pool disposal only)
 Gifts received by the person in respect of the business
 Amount derived for accepting restriction on tcapacity of conduction business
 Amount derived that are effectively connected with the business and that would
otherwise be included in calculating the person’s income from an investment
 Other amounts derived from
 Change in the basis of accounting(Sec.22(6))
 Gain from exchange fluctuation(Sec.24(4))
 Recovery of bad debt (25(1))
 Income from long term contract
 Interest rate lower than market rate
 Indirect advantage from associated person (Sec 29)
 Compensation received (Sec 31)
 Distribution of income other than profit
 Change in ownership (Sec 51)
 Income of general insurance
 Following amount shall not be included in calculating income from business;
 Amount exempted under section 10.
 Final withholding payment
 Amount distributed by resident entity except resident company and partnership and
redistribution of distributed profit by any resident entity.
 Amount exempted under section 69 (controlled foreign entities) for inclusion.

5) Income from Employment


As per section 8, provisions related to income from employment are as follows:

Naresh Singh Bhandari


bnsingh5102@gmail.com Page6
Income Tax Note 2076/77

 Income from employment = Natural person’s remuneration from employment.


 It includes following payment made by the employer to the natural person during the
year:
 Wages, salary, leave pay, overtime pay, fees, commission, gift, bonus, prize and other
facilities.
 Payment of personal allowances including any cost of living, subsistence, rent,
entertainment and transportation allowance.
 Payment providing any discharge or reimbursement of costs incurred by the natural
person or an associate of natural person.
 Payment for natural person’s agreement to any condition of employment.
 Payment for redundancy or loss or termination of the employment.
 Retirement contributions, including those paid by the employer to a retirement fund
in respect of employee, and retirement payment.
 Other payment made in respect of employment and
 Other amount derived from:
 Change in the basis of accounting(Sec.22(6))
 Gain from exchange fluctuation(Sec.24(4))
 Recovery of bad debt (25(1))
 Income from long term contract
 Interest rate lower than market rate
 Indirect advantage from associated person (Sec 29)
 Compensation received (Sec 31)
 The following are excluded in calculating a natural person’s income from employment:
 Amount exempt under section 10 and final withholding payment.
 Meals and refreshment provided by the employer to the employee at the work place
that is available to all the employees on equal terms.
 Any discharge or reimbursement of costs incurred by the natural person:
 That serve the proper business proposes of the employer,
 That are or would be otherwise be deductible in calculating the natural person’s
income from business or investment; and
 Payment of small prescribed amount i.e. up to Rs. 500 (Rule 6) which is so small
and thus unreasonable or administratively impracticable to make accounting for
them.

6) Income from Investment


As per section 9, the provisions related to income from investment are as follows:

 Income from investment = Profit and gains from investment during the year.

Naresh Singh Bhandari


bnsingh5102@gmail.com Page7
Income Tax Note 2076/77

 Amount derived from following shall be included while calculating profit and gains from
the investment during the year:
 Any dividend, interest, natural resource payment, rent, royalty, gain from investment
insurance, gain from an unapproved retirement fund, interest or retirement payment
made by an approved retirement fund in respect of investment.
 Net gain from the disposal of the person’s NBCA.
 Net gain from disposal of depreciable assets (in case of pool disposal only).
 Gift received by the person in respect of investment.
 Retirement contributions, including those paid to a retirement fund in respect of the
person, and retirement payment in respect of investment.
 Amount derived as consideration for accepting a restriction on the capacity to conduct
the investment.
 Other amount derived from:
 Change in the basis of accounting(Sec.22(6))
 Gain from exchange fluctuation(Sec.24(4))
 Recovery of bad debt (25(1))
 Interest rate lower than market rate
 Indirect advantage from associated person (Sec 29)
 Compensation received (Sec 31)
 The following amount shall not be included in calculating income from investment:
 Amount received which are exempted from tax
 Dividend distributed from the entity other than company for tax purpose is not
includible in inclusions under section 54.
 Dividend received from controlled foreign entity under sec 69, if taxed under the
head of income from investments.
 Final withholding Payment

Naresh Singh Bhandari


bnsingh5102@gmail.com Page8
Income Tax Note 2076/77

Exempt Amount and Other Concessions

1) Definition
2(Dha) Exempt Organization: means the following entity;
 Following entity registered with the Department as an exempt organization;
 A social, religious, educational, or charitable organization of public character
established without profit motive.
 An amateur sporting organization formed for the purpose of promoting social or
sporting facilities without taking any benefit by the organization or its members.
 A political party registered with Election Commission.

Provided that the organization is will not exempted for tax, in case a person is benefited
by the assets of the organization or from any amount received by the entity except in
pursuit of the entity’s function as per its objectives or as payment for assets or services
rendered to the entity by the person

2) Exempt Amount
As per section 10, the following amounts are exempt from tax:

 Amount derived by a person entitled to privileges under a bilateral or a multilateral treaty


or agreement concluded between GON and a foreign country or an international
organization.
 Amount derived by a natural person from employment in the public service of the
government of a foreign country provided that;
 The natural person is resident person solely by reason of performing the employment
or is non-resident person, and
 The amounts are payable from the public funds of the country.
 Amount derived from the public fund of the foreign country by a natural person who is
not a citizen of Nepal as above or by a member of the immediate family member of the
natural person.
 Amount received by the natural person who is not a citizen of Nepal from employment
by GON on terms of a tax exemption.
 All the allowances paid by GON, State Government or Local levels in the form of
security allowance.
 Amount derived by way of gift, bequest, inheritance or scholarship except as required to
be included in calculating income from business, employment and investment.
 Amount derived by an exempt organization by way of;
Naresh Singh Bhandari
bnsingh5102@gmail.com Page9
Income Tax Note 2076/77

 Donation, gift;
 Other contributions that directly relate to the organization’s function, whether or not
the contribution is made in return for consideration provided by the organizations.
 Pension received by a Nepalese citizen retired from army or police service of a foreign
country provided that the amounts are payable from the public fund of the country.
 Any type of income of GON, State Government or Local levels.
 Income earned by NRB under its objective.

3) Donation, Gift to exempt Organizations


As per section 12, provisions related to donation, gift to exempt organizations are as follows;

 A person may reduce the lower of the following amount donated to the exempt
organization while calculating the taxable income for the year;
 Actual amount donated to exempt organizations during the year.
 Rs. 100,000, or
 5 % of person adjusted taxable income for the year.
 GON may prescribe, by notification in the Nepal Gazette, as to allow full or partial
deduction at the time of assessing a person’s income of the expenses incurred for special
purpose, or the expenses of gift by the person.
 As per section 12(ka), a company incurring expenditure with prior approval from
Department for conservation and promotions of ancient, religious and cultural heritage
situated within Nepal and for construction of public physical infrastructure of sports may
claim the lower of the following amount for reduction in an income year in calculating
taxable income;
 Actual expenditure incurred,
 Rs. 1000,000, or
 10 % of person’s assessable income for the year.

As per section 12(Kha), contribution made by the person during an income year in Prime
Minister Relief Fund and National Reconstruction fund established by GON can be deducted
while calculating taxable income for the income year of the person.

Naresh Singh Bhandari


bnsingh5102@gmail.com Page10
Income Tax Note 2076/77

Deductions

1) Definition
2(Tta) Debt Obligation: means an obligation corresponding to a debt claim.
2(Thha) Debt Claim: means right of the one person to receive a payment from another
person and includes a right to get repayment of an amount paid by one person to another
person as well as the deposits in the BFIs, accounts receivable, notes, bill of exchange, bonds
and right under annuities, finance lease and installment sales.
2(KaJha) Interest: means the following payment or gain:
 A payment under a debt obligation except the principal;
 Any gain realized by way of discount, premium, swap payment or similar payment under
a debt obligation; and
 The portion which is treated as interest in the payment made to a person under an annuity
or by a person acquiring an asset under an installment sale or the use of an asset under a
finance lease.
2(KaYna)Trading Stock: means the assets owned by the person that are for sales in the
ordinary course of business conducted by the person, assets under work in progress, and
inventories of materials to be composed into such assets.
Provided that, this term does not include foreign currency assets.

2(KaNa1) Adjusted Taxable Income: means an amount of taxable income of a person for
an income year as calculated by ignoring reductions referred to in section 12 and deductions
referred in section 14(2), 17 and 18.

Elaboration: As mentioned in Page no. 40 of Income tax manual 2066(updated 2068)


Adjustable Taxable Income (A)

Particulars Amount (Rs.)


Income from employment *****
Income from Business except deduction of 14(2), 17 and 18 *****
Income from Investment except deducting of 14(2) *****
Assessable Income *****
Reduction Except donation *****
Adjusted Taxable Income *****

ATI for Sec. 14(2)


ATI as calculated in A *****
+ Interest expenses deducted *****
-Interest Income included *****
Naresh Singh Bhandari
bnsingh5102@gmail.com Page11
Income Tax Note 2076/77

-Actual Research and Development Expenses *****


-Actual Pollution control cost *****
ATI for Section 14(2) *****

Interest allowable under sec. 14(2)


Interest Income *****
50 % of ATI *****
Max. Allowable *****
OR, Actual interest expenses whichever is lower.

ATI for Sec.17


ATI as calculated in A *****
- Interest expenses under sec. 14(2) *****
-Actual Research and Development Expenses *****
ATI for Section 17 *****

 Pollution control cost allowable for deduction is actual expenses or 50 % of ATI


whichever is lower.

ATI for Sec.18


ATI as calculated in A *****
-Interest expenses under sec. 14(2) *****
-Actual Pollution control cost *****
ATI for Section 18 *****

 Research and development cost allowable for deduction is actual expenses or 50 % of


ATI whichever is lower.

ATI for Sec. 12


ATI as calculated in A *****
-Actual Interest u/s 14(2) *****
-Actual Research and Development Expenses *****
-Actual Pollution control cost *****
ATI for Section 12 *****
Allowable Donation: lower of
 Rs. 100,000
 5% of ATI or
 Actual Donation

Naresh Singh Bhandari


bnsingh5102@gmail.com Page12
Income Tax Note 2076/77

Example (1):

M/S Bhandari Pvt. Ltd. furnished the following particulars pertaining to this income year.
Expenses Rs. Income Rs.
Cost of Trading Stock 4,200,000.00 Sales 7,500,000.00
Administration expenses 600,000.00 Interest income 50,000.00
Donation 180,000.00
Pollution Control Cost 850,000.00
Research and Development Cost 1,200,000.00
Interest Expenses 450,000.00
Net profit 70,000.00
Total 7,550,000.00 Total 7,550,000.00
Additional Information:
30 % shares are held by the non-resident person to whom the company has paid Rs. 440,000 as
interest during the year.
Required: Calculate allowable PCC, R & D cost and Donation

Solution

1) Calculation of ATI
Sales 7,500,000.00
+Interest income 50,000.00
Total Income 7,550,000.00
-Cost of Trading Stock 4,200,000.00
-Administration expenses 600,000.00
-Interest 10,000.00
ATI 2,740,000.00

2) Calculation of ATI for Sec 14(2)


ATI as calculated in (1) 2,740,000.00
-Interest Income (50,000.00)
+Interest Expenses 10,000.00
-Pollution Control Cost (850,000.00)
-Research and Development Cost (1,200,000.00)
ATI for Interest 650,000.00

Allowable Interest
a) Normal Interest 10,000.00
b) Paid to Exempted Controlled entity
Interest Income 50,000.00

Naresh Singh Bhandari


bnsingh5102@gmail.com Page13
Income Tax Note 2076/77

50 % of ATI 325,000.00
Maximum Allowable 375,000.00
Actual 440,000.00
Allowable Interest 375,000.00
Total Interest Allowable(a+b) 385,000.00

3) Calculation of ATI for PCC


ATI as calculated in (1) 2,740,000.00
-Actual R & D Cost 1,200,000.00
-Actual Interest under 14(2) 440,000.00
ATI for PCC 1,100,000.00

Allowable PCC
50 % of ATI 550,000.00
Actual 850,000.00 W/h is lower.

4) Calculation of ATI for R & D Cost


ATI as calculated in (1) 2,740,000.00
-Actual PCC Cost 850,000.00
- Actual Interest under 14(2) 440,000.00
ATI for R & D Cost 1,450,000.00

Allowable R & D Cost


50% of ATI 725,000.00
Actual 1,200,000.00 W/h is lower.

5) Calculation of ATI for donation


ATI as calculated in (1) 2,740,000.00
-Actual Interest u/s 14(2) 440,000.00
-Actual PCC 850,000.00
-Actual R & D Cost 12,00,000.00
ATI for Donation 250,000.00

Allowable Donation Expenses


Eligible Donation as reduction is lower of
a) Rs. 100,000
b) 5% of ATI Rs.12,500.00
c) Actual Rs.180,000.00

Naresh Singh Bhandari


bnsingh5102@gmail.com Page14
Income Tax Note 2076/77

Summary
 Interest allowed u/s 14(2) is Rs. 375,000. Remaining balance i.e 440,000-375,000=Rs.
65,000 deferred for next year.
 PCC and R & D Cost allowed for deduction is Rs 550,000 and 725,000 respectively.
Remaining Balance of PCC and R & D cost Rs. 300,000 and Rs. 475,000 goes to pool
D of depreciation pool.
 Donation allowable for reduction is Rs. 12,500.

2) General Deductions
As per section 13, expenses fulfilling all of the following conditions shall be allowable for
the deduction at the time of calculating a person’s income for an income year from any
business or investment:
 Incurred during the year,
 Incurred by the person and
 Incurred in earning the income from business or investment.

3) Interest
As per section 14, provisions related to deduction of interest are as follows:
 Interest incurred by the person during the year under a debt obligation arisen for earning
income from business or investment shall be allowable for deduction while calculating
income of the person from business or investment if;
 The debt obligation was incurred in borrowing the money, the money used during the
year or was used to purchase an asset that is used during the year; or
 In any other case, the debt obligation was incurred.
 In case of interest paid by an exempt controlled resident entity to a person or associate of
persons having control over the entity, the deductible interest amount for an income year
shall not exceed the sum of the following;
 Interest income derived during the year +
 50 % of ATI calculated excluding interest income derived and interest expenses
incurred during the year.
 Interest expenses disallowed due to limit mentioned above shall be carried forward and
treated as incurred during the nest income year.
 An entity is an exempt controlled resident entity for any income year if it is a resident
entity for the year and at any time during the income year 25 % percent or more of the
underlying ownership or control of the entity is held by;
 Exempt organization or associates of exempt organization.
 Persons or associates of persons deriving during the year income for which a
concession is available under section 11.
 Non-resident or associates of non-resident persons; or
Naresh Singh Bhandari
bnsingh5102@gmail.com Page15
Income Tax Note 2076/77

 Any combination of person mentioned above.

Example (2):
Fewa Pvt. Ltd. has been established by Nepalese promoters under foreign direct investment.
The eighty percent of the total capital has been hold by non-resident persons. In addition, The
Pvt. Ltd. has borrowed the amount from the foreign investors. For the year ended Ashadh
end, 2070, the profit and loss account of the company has the following transactions.

Particulars Amount in Rs.


Sales 5,000,000
Interest income 75,000
Total income 5,075,000
Expenses
Cost of sales 2,000,000
Administrative expenses 1,100,000
Interest expenses 1,500,000
Pollution control expense 300,000
Total Expenses 4,900,000
Net profit 175,000
i) Interest expense Rs. 1,400,000 out of total has been charged on the borrowed amount from
foreign investors. Answer whether all the above expenses can be claimed under Income Tax
Act, 2058?
ii) If these expenses cannot be claimed during the year, what would be the implication?
CAP II June 2014 (5 marks)
Solution:
i) Calculation of allowable Interest Expenses

Particulars Amount
Sales 5,000,000
Expenses:
Cost of Sales 2,000,000
Administrative Expenses 1,100,000
Pollution control Expenses 300,000
Adjustable Income (ATI) 1,600,000
50 % of ATI 800,000
Interest Income 75,000
Maximum Allowable u/s 14(2) 875,000
Actual Paid to exempted controlled Entity 1,400,000
Allowable u/s 14(2) 875,000
Allowable u/s 14(1) 100,000
Total Allowable 975,000

Naresh Singh Bhandari


bnsingh5102@gmail.com Page16
Income Tax Note 2076/77

ii) As per section 14 (3) of the act any interest not allowed to be deducted or not deducted
pursuant to Sub-section (2) may be carried forward or credited in the forthcoming income
year. So, the balance interest of Rs. 525,000 (Rs. 1,400,000 - Rs. 875,000) will be carried
forward in the coming year and computed interest expenses as per above.
Note: For calculation of allowable pollution control cost, see page no. 18 under the
Pollution control cost heading.

4) Cost of Trading Stock


As per section 14, provision related to cost of trading stock are as follows:
 Cost of sale of trading stock sold is allowable for deduction while calculating income
from business of the person during the year.
 Cost of sale of the trading stock sold shall be calculated as follows:
Cost of opening stock xxx
Add: produced during the year xxx
Less: Cost of closing stock xxx
Cost of trading stock consumed or sold xxx
 Opening value of trading stock for an income year = closing value of trading stock of the
previous income year.
 The closing value of the trading stock shall be lower of the cost of the trading stock or
market value of trading stock.
 The person keeping account on cash basis may use prime cost or absorption cost method
and the person keeping account on accrual basis shall use absorption cost method for
determining the cost of trading stock.
 If the cost of trading stock is not readily identifiable, the person may choose that the cost
of trading stock be determined according to the FIFO method or weighted average
method.
 The prime cost is derived as follows:
Cost of raw materials consumed xxx
Add: Direct labor xxx
Add: Factory variable overhead (Exc. Repair) xxx
Prime Cost xxx
 The factory cost is derived as follows:
Cost of raw materials consumed xxx
Direct labor xxx
Factory overhead (Exc. Repair) xxx
Factory Cost xxx
 Factory overhead cost (fixed and variable both) does not include any amount of repair
and improvement costs and depreciation allowances.
Example (3): Describe the provision in the Income Tax Laws for valuation of finished goods
of a consumer food manufacturing industry? Which method of inventory valuation is
appropriate for an edible oil processing industry? CAP III June 2012 (4+3) marks
Answer:

Naresh Singh Bhandari


bnsingh5102@gmail.com Page17
Income Tax Note 2076/77

1st Part: Write the provision as mentioned above.


2nd Part: The product of the edible oil industry is Food category on which the batch-wise and
date-wise stocking is required.
To maintain the food fresh, first in first out method must be maintained so that the latest
production will be stocked through the adopted system of inventory management.
Therefore, the appropriate method of valuation will be FIFO for such edible oil processing
industry.

5) Repair and Improvement Expenses


As per section 16, provision related to repair and improvement expenses are as follows:

 Repair and improvement expenses incurred in respect of depreciable assets owned and
used by the person during the year in the production of person’s income from business or
investment are allowable for deduction while calculating income from business or
investment.
 The deduction allowable of the depreciable asset in a particular block of the depreciable
asset shall not exceed 7 % of the depreciation base of the block at the end of the year.
However, such limit of 7% shall not be applicable to repair and maintenance expenditure
incurred by a person providing air transport service in overall checking of the aircraft as
per standard determined by Civil Aviation Authority of Nepal.
 Repair and maintenance expenses disallowed due to the limit mentioned above shall be
added to the depreciation base of the relevant block prevailing in the beginning of the
subsequent year.

6) Pollution control Expenditure


Pollution control expenditure means costs incurred by a person with respect to a process or
an asset that seeks to control pollution or otherwise protect or sustain the environment.
As per section 17, provisions related to pollution control cost are as follows:

 Pollution control cost incurred by the person shall be allowable for the deduction for the
purpose of calculating income from business during the year.
 Pollution control cost allowable for deduction shall not exceed 50 % of ATI from all the
business conducted by the person.
 Pollution control cost disallowed due to the limit mentioned above shall be added to the
depreciation base of the relevant block prevailing in the beginning of the subsequent year.
Example (4): In example no. 2, allowable Pollution control cost is as follows:
Calculation of Allowable PCC:
Net profit 175,000
Add: Pollution Control Cost 300,000
Adjusted Taxable Income (ATI) 475,000

Naresh Singh Bhandari


bnsingh5102@gmail.com Page18
Income Tax Note 2076/77

50 % of ATI 237,500 OR,


Actual 300,000 W/h is lower.
Therefore, allowable pollution control cost is Rs. 237,500 and balance Rs. 62,500 (Rs.
300,000 – Rs. 237,500) shall be added to the depreciation base of the relevant block
prevailing in the beginning of the subsequent year.

7) Research and Development Expenditure


Research and development Expenditure means cost incurred by the person for the purpose
of developing the person’s business and improving business product or process.
Provided that, the term does not include any cost incurred that is an outgoing for any asset
including cost incurred in respect of natural resource prospecting, exploration and
development by a person in the production of the person’s income from a business. Such cost
shall be treated as if they were incurred in securing the acquisition of an asset that is used by
the person in that production.

As per section 18, provisions related to research and development expenditure are as follows:

 Research and development expenditure incurred by the person shall be allowable for the
deduction for the purpose of calculating income from business during the year.
 Research and development expenditure allowable for deduction shall not exceed 50 % of
ATI from all the business conducted by the person.
 Research and development expenditure disallowed due to the limit mentioned above shall
be added to the depreciation base of the relevant block prevailing in the beginning of the
subsequent year.

8) Depreciation Allowance
As per section 19, the provisions related to depreciation allowance are as follows:

 Depreciation of the depreciable asset used in an income year under own ownership for
generating the income from business or investment shall be allowable for the deduction at
the time of calculating income from business or investment during the year.
 The following provision shall be applied in respect of the depreciation of the machines,
equipment and other machinery installed in the electricity projects that are involving in
building power station, generating and transmitting electricity and in the projects so as to
build public infrastructure, own, operate and transfer to the GON;
 In case of old machines, equipment and other machinery that are already installed
require replacement in any income year as they are out of order due to being too old,
the balancing value of the old machine remained after subtracting the depreciation up
to the year from their cost shall be allowed as expenses for the year.

Naresh Singh Bhandari


bnsingh5102@gmail.com Page19
Income Tax Note 2076/77

 At the time of transfer of other assets to the GON except of the old asset replaced as
above, the balancing value, if remained, after subtracting the depreciation up to
previous year of the transfer from their cost shall be allowed as expenses.

As per schedule 2 provision related to calculation of depreciation are as follows:

i) Classification and Pooling of depreciable Assets {Sch. 2(1)}


 Depreciable assets are classified are as follows:
Pool Assets Included
A Buildings, Structures, and similar works of a permanent nature
B Computers, data handling equipment, fixtures, office furniture and office equipments.
C Vehicles ( Automobiles, Buses and minibuses)
D Construction and earth-moving equipment any other depreciable asset not included in
another class including carried forward Research & Development expenditure and
pollution control cost and specific assets of the business.
E Intangible assets other than depreciable assets included in above.

 At the time of the asset of class A, B, C and D first owned or so used, the asset shall be
added to the respective pool. In case of class E, separate pool of the asset made even if
the asset is of same class.
 The cost incurred in respect of natural resource prospecting, exploration, and
development in the production of person’s income from business shall be treated as
acquisition of assets that is used by the person in that production.

ii) Depreciation Allowance {Sch. 2(2)}


 Depreciation expenses allowable for deduction during the year = depreciation base of the
pool at the end of the income year * Depreciation Rate
 Depreciation Base for the pool A, B, C, D for the year shall be calculated as follows:

Opening Depreciation base/WDV ×××


Add: Addition during the year
Up to Poush end (Full cost) ×××
Up to Chaitra End (2/3rd of total cost) ×××
rd
Up to Ashad End (1/3 of total cost) ×××
Less: Disposal proceeds of Asset during the year ×××
Depreciation base for the year ×××
 Depreciation base for the pool E for the year shall be calculated as follows:
Opening Depreciation base/WDV ×××
Add: Addition during the year
Up to Poush end (Full cost) ×××
rd
Up to Chaitra End (2/3 of total cost) ×××
Up to Ashad End (1/3rd of total cost) ×××
Depreciation base for the year ×××

Naresh Singh Bhandari


bnsingh5102@gmail.com Page20
Income Tax Note 2076/77

 If at the end of the year, WDV of the assets of pool A, B, C and D after depreciation for
the income year becomes less than Rs. 2,000, then such amount shall be added to the
depreciation of that year.

iii) Rate of Depreciation {Sch. 2(3)}


 Depreciation rate applicable to the different pool are as follows:
Pool Rate
“A” 5%
“B” 25%
“C” 20%
“D” 15%
“E” (1/n) *100, Where n = useful life of assets (nearest half year).

 Additional 1/3rd to the rate mentioned above shall be applied to the following:
 Entity wholly engaged in operating a special industry as referred in section11.
 Entity engaged in construction and operation of road, bridge, tunnel, rope-way, or
flying bridge.
 Entity engaged in operation of trolley bus, or tram.
 Co-operatives registered and carrying out transaction under Co-operative Act, 2048
except those carrying out transaction with tax exemption.
 Entity engaged in the projects so as to build public infrastructure, own, operate and
transfer it to the GON and in building power station, generating and transmitting
electricity.
 Depreciation can be claimed for the deduction at 50% of the capital expenditure of the
capitalized assets for the production of the energy necessary for the industry by a
manufacturing company.
 Depreciation can be claimed for the deduction for the lump sum amount of the
expenditure incurred for fiscal printer and cash machine if a person issues invoices
through fiscal printer and cash machine.

iv) Disposal of depreciable Assets {Sch. 2(4)}


 If in case of pool A, B, C and D, person’s income from disposal of the depreciable assets
of the pool exceeds depreciation basis of the pool at the end of the year, then such amount
shall be included in the income of the business or investment.
 If before the end of an income year a person disposes of all the assets in a pool of
depreciable assets of the person, the pool shall be treated as dissolved and at the time of
disposal if;
 Income from disposal > WDV + outgoing during the year, excess amount shall be
included in income.
 WDV + outgoing during the year > Income from disposal, excess amount shall be
allowed for deduction at the time of calculation of income.

Example (5): XYZ & Co. has provided the following details of its assets during the
Income Year 2071-72:

Naresh Singh Bhandari


bnsingh5102@gmail.com Page21
Income Tax Note 2076/77

Block Opening Depreciation Base Addition During year (Rs.) Purchase Date Disposal
A 3,000,000 - - -
B 500,000 300,000 Bhadra 01,2071 50,000
C 150,000 500,000 Chaitra 30, 2071 175,000
D 700,000 - - -

Additional Information:
Before Magh 01, 2071, the company incurred Rs. 300,000 to acquire the patent right for
the period of 5 years and 7 months.
Required:
i) Calculate the allowable depreciation allowances for the Income Year 2071.72 in
respect of all the block of assets.
ii) What will be the implication on depreciation allowance if XYZ & Co. is a special
industry as defined under the Income Tax Act, 2058? CAP II, July 2015 (8+2 Marks)

Solution:

i) Calculation of allowable Depreciation

Pool A B C D
Opening WDV 3,000,000 500,000 175,000 700,000
Additions:
Up to Poush (Full) - 300,000 - -
Up to Chaitra (2/3 of Cost) - - 333,333 -
Sold - (50,000) (175,000) -
Depreciation Base 3,000,000 750,000 333,333 700,000
Rate 5% 25% 20% 15%
Depreciation 150,000 187,500 66,667 105,000

Calculation of allowable amortization on Patent right:


Cost of Patent: Rs. 300,000
No. of year = 5+7/12 = 5.5
Rate of Depreciation = (1/5.5) *100 = 18.18 %
Absorbed addition: 2/3*300,000 = 200,000 (since purchased in Magh)
Depreciation for the year: 200,000*18.18% = Rs. 36,363.64
Note:
 As per section 3 of Schedule 2 of Income Tax Act, 2058, the useful life of an
intangible asset is rounded down to the nearest half year.
 Problem is solved as per the method prescribed by Tax Directives.

ii) Entities referred to in section 19 (2) and paragraph 2(3) and (4) of schedule 1 shall be
entitled to an additional depreciation rate added by 1/3 in the depreciation rates

Naresh Singh Bhandari


bnsingh5102@gmail.com Page22
Income Tax Note 2076/77

referred to in subparagraph (1) applicable to pools of depreciable assets in Class A, B,


C, and D.
Computation of Allowable Depreciation if XYZ & Co is a special Industry
For Block A: 6.67% of Rs. 3,000,000 = Rs. 200,000
For Block B: 33.33% of Rs. 750,000 = Rs. 250,000
For Block C: 26.67% of Rs. 308,333 Rs. 82,232.42
For Block D: 20% of Rs. 700,000 = Rs. 140,000
For Block E: Rs. 36,363.64
Additional 1/3 depreciation allowance is not applicable for the pool of asset under
Block E.

Example (6)
Following are the excerpts of the transaction of fixed assets in the books of Times
Commercial Bank Limited. Calculate the depreciation and repairs and maintenance as per
tax and the closing value of the pool of assets. CAP III, Dec 2012 (5 Marks)
Opening pool Value Rs.
Pool A 120,000,000
Pool B 5,550,000
Pool C 12,000,000
Pool D 13,500,000
Pool E 1,236,000
Addition of assets: First half (Rs.) Quarter 3 (Rs.) Quarter 4 (Rs.)
Furniture 500,000 1,200,000 2,000,000
ATM Machine 1,500,000 1,400,000 1,400,000
Generator Set 500,000 600,000 1,200,000
Leasehold improvement - 9,600,000 -
Sale of Assets: Value (Rs.)
Furniture 200,000
Building 3,500,000
Repairs and Maintenance
Repairs of Furniture 100,000
Repairs of Vehicles 550,000
Repairs of Generators 200,000
Repairs of Computers 400,000
Repairs of ATM machine 400,000
Repairs of leased buildings 500,000
Additional information:
i). Opening WDV of leasehold assets are to be amortized for last installment this year.
ii).Rs. 3,600,000 of leasehold addition has life of 60 months and addition of Rs.
6,000,000 has life of 120 months, these were put to use from 1st Magh of the year.

Solution:
Calculation of Allowable Repairs, Depreciation and WDV
Pool A B C D E Total

Opening WDV 120,000,000 5,550,000 12,000,000 13,500,000 1,236,000 152,286,000

Naresh Singh Bhandari


bnsingh5102@gmail.com Page23
Income Tax Note 2076/77

Additions:

Up to Poush (Full) - 500,000 - 2,000,000 - 2,500,000

Up to Chaitra (2/3 of Cost) - 800,000 - 1,333,333 9,600,000 11,733,333

Up to Ashad (1/3 of cost) - 666,667 - 866,667 - 1,533,333

Sold (3,500,000) (200,000) - - - (3,700,000)

Depreciation Base 116,500,000 7,316,667 12,000,000 17,700,000 10,836,000 164,352,667

Rate 5% 25% 20% 15% Portrate -

Depreciation 5,825,000 1,829,167 2,400,000 2,655,000 1,896,000 14,605,167

Repairs - 500,000 550,000 600,000 - 1,650,000

Limit ( 7% of Base) 8,155,000 512,167 840,000 1,239,000 - 10,746,167

Allowed Repairs - 500,000 550,000 600,000 - 1,650,000

WDV after Depreciation 110,675,000 5,487,500 9,600,000 15,045,000 8,940,000 149,747,500

Unabsorbed Addition - 1,733,333 - 2,400,000 4,133,333

Closing WDV 110,675,000 7,220,833 9,600,000 17,445,000 8,940,000 153,880,833

Repairs allowed on depreciable assets 1,650,000


Repairs allowed on leasehold u/s 500,000
Total Repairs allowed 2,150,000
Allowed Depreciation 14,605,167

Note: The allowable depreciation of pool E is calculated as per the method prescribed by
suggested answer published by ICAN which is different from the method prescribed by
tax directives. The calculation of allowable depreciation of pool E as per the method
prescribed by tax directives is as follows:
Particulars Opening 60 months 120 month
Opening 1,236,000 - -
Purchased During the year - 3,600,000 6,000,000
Purchase Date - 1st Magh 1st Magh
Absorbed Addition - 2,400,000 4,000,000
Useful Life (in years) - 5 12
Depreciation rate (%) - 20.00 10
Depreciation 1,236,000 480,000 400,000
WDV after depreciation - 1,920,000.00 3,600,000
Unabsorbed Addition - 1,200,000 2,000,000
Closing WDV - 3,120,000 5,600,000

i.e. allowable depreciation of pool E is Rs. 2,116,000.

Naresh Singh Bhandari


bnsingh5102@gmail.com Page24
Income Tax Note 2076/77

9) Losses from Business or Investment


As per section 20, provisions related to losses from a business or investments are as follows:
 Business losses can be set off with business gain and investment gain and investment
gain can be set off set off with investment gain only.
 Any unrelieved loss of the previous 7 income years incurred by the person from business
can be set off with income from other business and investment.
 Any unrelieved loss of the previous 7 income years incurred by the person from
investment can be set off with income from investment.
 In the following case, unrelieved loss of previous 12 years can be set off with the income
of the current year:
 Project to be transferred to the GON after construction and operation of Public
infrastructure.
 Project for constructing power station, generating and transmission, and
 Entity dealing in petroleum under Nepal Petroleum Act, 2040.
 A person may deduct an unrelieved loss from foreign source only in calculating a
person’s foreign source income and an unrelieved loss incurred in deriving non-taxable
income only in calculating the person’s non-taxable income.
 Losses incurred by the person in a long term contract won through international
competitive bidding can be carried backward for set off with the income under the same
contract in previous income years. (See chapter long term contract for more details of
losses from long term contract.)
 Losses from business/investment = Deductions – Inclusions
 Loss incurred by the person in the year in which the person is exempted from tax shall
not be allowed to carry forward to the next year for the deduction.

Example (7):Singha Road Ltd. located at Hetauda is a public infrastructure project. The project
will be completed on Chaitra end 2075 as per estimation, and then it will be handover to
Government of Nepal. The Ltd. has incurred loss continuously from income year 2061/62 to
2068/69 as follows:

Income Year Loss amount Rs.


2061/62 1,000,000
2062/63 500,000
2063/64 600,000
2064/65 200,000
2065/66 150,000
2066/67 100,000
2067/68 75,000
2068/69 10,000
During income year 2069/70, it has incurred the profit of Rs. 1,500,000 Compute the taxable
income for the income year 2069/70 and carry forward losses for the income year 2070/71 for set
off with references to the provisions of Income Tax Act, 2058 relevant to this project.
CAP II, June 2014 (5 marks)
Answer:

Naresh Singh Bhandari


bnsingh5102@gmail.com Page25
Income Tax Note 2076/77

Section 20 (1) of income tax act, 2058 has the following provisions for carry forward the
losses:
For purposes of computing the income earned by any person from any business or investment
in any income year, such person may deduct the loss as mentioned below: -
(a) Loss suffered by that person from any other business and not deducted in that year, and
(b) Loss suffered by that person from any business which was not deducted in the last seven
income years.
Provided that, in the case of the projects which involve building and operation of public
infrastructures to be transferred to the Government of Nepal, power houses construction,
generation and transmission of electricity and petroleum works pursuant to the Nepal
Petroleum Act, 2040 any loss not deducted in the last twelve income years.
As per the provisions, the project has not fulfilled the conditions of BOT. It will be handover
after the completion. So, it can carry forward only 7 years’ losses.

Calculation of taxable income for F/Y 2069/70.


Net income during the year Rs. 1,500,000
Less: carry forward losses Rs. 1,635,000
2062/63 Rs. 500,000
2063/64 Rs. 600,000
2064/65 Rs. 200,000
2065/66 Rs. 150,000
2066/67 Rs. 100,000
2067/68 Rs. 75,000
2068/69 Rs. 10,000 Rs. 1,635,000
Taxable loss for the year 2069/70 Rs. 135,000
Carry forward losses for the year 2070/71:
The taxable loss for the income year 2069/70 is Rs. 135,000. The loss of 2063/64 to 2065/66
is 1,450,000 which is fully setoff in FY 2069/70. The loss of Rs. 50,000 of 2066/67 and the
loss of 2067/68 and 2068/69 is carried forward to 2070/71, total of Rs. 135,000.

Example (8): ABC Ltd. has taken a long term contract for the construction of a building for
Nepal Government through an international bidding for Rs. 50 million to be constructed
within three years commencing from the year 2067-68. The company has recognized the
profit of Rs. 500,000 from the contract during the year 2067-68 and Rs. 200,000 during
2068-69 on the basis of estimations at that time but during the final year the accounts show a
net loss of Rs. 800,000. The company seeks permission from IRD to carry back loss and to
recognize the net loss. By stating the relevant provisions mentioned under section 20(4) of
the act, suggest whether the loss can be carried back? Will your answer differ if ABC Ltd.
has taken the contract not through international bidding? CAP III, June 2014 (5 Marks)
Answer:
As per section 20(4) of the act, if a person has acquired a long term contract through
international bidding and incurred a loss in a year of completion or the year of disposal, it has
to obtain permission from IRD for its better treatment. IRD may give permission imposing
the following terms;

Naresh Singh Bhandari


bnsingh5102@gmail.com Page26
Income Tax Note 2076/77

a) The loss could be carried back up to the period of first year from the start of the work of
the long term contract and
b) The net loss from the contract may be calculated by considering the cumulative contract
cost and cumulative contract revenue and the loss may be treated as a loss to be allowed
to set off or carried forward.
In the given case, since the contract is taken through international bidding, if IRD gives
permission, then loss could be carried back.
Final figure of net loss Rs. 800,000
Less; Profit recognized in 2067-68 Rs. 500,000
Less; profit recognized in 2068-69 Rs. 200,000
Net loss to be set off or carried forward Rs. 100,000
A credit of tax is allowed to the company for Rs. 500,000 and Rs. 200,000 at the average rate
of the tax paid by the company during the year of profit recognition.

But if ABC Ltd. has taken the contract not through international bidding, IRD shall not allow
the company to enjoy the facility of loss carry back. The profit recognized in earlier income
years shall not be amended and the loss of Rs. 800,000 could set off from any other income
from business or investment of the company or could be carried forward for 7 years for set
off.

10) Deduction Not Allowed


As per section 21, provisions related to deduction not allowed are as follows:
 Deduction for the following is not allowed while calculating income from business,
employment and investment:
 Personal or domestic nature expenses.
 Tax payable under this act and fines and penalties paid to the government for breach
of any law or regulation or bylaws framed there under. However, tax paid to the local
level and Province Government is deductible expenses
 Expenses incurred in deriving exempt income (Sec 10) and final withholding
payment.
 Expenses of more than Rs. 50,000 paid in cash by a person whose annual turnover
exceeds Rs. 2 million other than in the following condition
 Payment made to GON, a constitutional body, a corporation owned by GON or a
bank and financial institution.
 Payment made to the farmer or producer producing primary agricultural produces
even in the case the produce is primarily processed by the farmer.
 Retirement contribution or retirement payment.
 Payment is made in the area where banking service is not available.
 Payment is and must necessarily be made in cash or on a day when banking
service are closed; or
Naresh Singh Bhandari
bnsingh5102@gmail.com Page27
Income Tax Note 2076/77

 Payment is made into a bank account of the payee.


 Salary and wages paid to the labor and employee who does not have Permanent
Account Number (PAN).
 Expenses incurred in excess of NPR 1000 without PAN number in the bill.
 Distribution of the profit by an entity.
 Expenses of capital nature and foreign income tax except as mentioned in section 14, 15,
16, 17, 18, 19, 20 and 71 are no allowed for deduction.
 Personal or domestic nature expenses means:
 Interest incurred on an amount borrowed to the extent to which it is used for personal
purpose.
 Expenses of personal nature such as residence, meals, refreshment, entertainment and
similar nature expenses.
 Conveyance and travel expenses for to and from residence except business trip during
workings.
 Clothing expenses except solely suitable to wear at work.
 Expenses incurred in education and training except those directly relevant to the
business and investment and which does not lead to the degree and diploma.
 An area not having banking services available means the area where there is no banking
facilities available within the surrounding of 10 kilometers.
 Capital expenditure means following expenditure;
 Expenditure of prospecting, exploration and development in case of natural resources.
 Acquisition of assets having a useful life for more than twelve months.
 Expenditure on Disposal of Liability.

Naresh Singh Bhandari


bnsingh5102@gmail.com Page28
Income Tax Note 2076/77

Tax Accounting and Timing

1) Method of Tax Accounting


As per section 22, the provisions related to the method of tax accounting are as follows:

 The time for determination of income derived and expenses incurred shall be made
according to generally accepted accounting principles.
 Income from employment or investment of a natural person shall be accounted on cash
basis for tax purpose.
 Books of account of the company shall be kept on accrual basis for tax purposes.
 A person may account for tax purposes on a cash or accrual basis unless the department
prescribes otherwise by notice in writing.
 A person who wants to change the basis of accounting may make an application for the
change and department may, by notice in writing and if satisfied that the change is
necessary to clearly reflect the person’s income, approve the application.
 If a person’s method of accounting for tax purpose is changed, adjustment shall be made
in the income year of the change and that following the change so that no amount
included, deducted, or to be included or deducted in calculating the person’s income is
omitted or repeated.

Example (1): Munal Trade Link is a various goods supplier. The trade link is using accrual
basis of accounting for the income generated from the business. He gave an application to the
department to change his basis of accounting from accrual to cash in Income Year 2070/071.
In accordance with the generally accepted accounting principles, Income Tax Department has
given the approval to account for his income on cash basis in Income Year 2070/71. Some
accounting information of the trade link at the end of 2069/070 before changing the basis of
accounting was as below;
i) Goods worth of Rs. 75,000 has already been supplied to a retailer in Income Year
2069/070 but amount has not been received. Advance Rs. 45,000 has been received from
another retailer.
ii) Goods worth of Rs. 60,000 has been purchased from a vendor in Income Year 2069/070
but amount has not been paid. Advance Rs. 35,000 has been paid to another vendor.
iii) Communications and electricity expenses of Rs. 3,500 were paid on Bhadra, 2071. These
expenses were incurred in Jestha and Ashadh, 2070.
State how you make adjustments of income and expense in FY 2070/071 in above
conditions? CAP II, Dec 2014 (5 Marks)
Answer:
As mentioned in section 22 of Income Tax Act, 2058, if any person takes the approval from
Inland Revenue Department for the change in basis of accounting for the purpose of taxation,
or his basis of accounting is changed because of various provisions mentioned in the same
Naresh Singh Bhandari
bnsingh5102@gmail.com Page29
Income Tax Note 2076/77

section, inclusion of his income, claim of expenditure shall be computed in such a manner
that there is no repetition or short of income or claim of expenditure in that Fiscal Year.
Accordingly, the following adjustments shall be made in Income Year 2070/71;
i) When goods are supplied, the income shall be recognized in accrual basis of accounting.
So, no need to adjustment for the goods of Rs. 75,000.00 that was included in Income Year
2069/70. Advance amounting Rs. 45,000.00 was not assessable income for the Income year
2069/70 as per accrual basis of accounting, so, it shall be included in Income Year 2070/71
on cash basis.
ii) When goods are purchased, the expenses shall be recognized in accrual basis of
accounting. So, no need to adjustment for the goods of Rs. 60,000.00 that was deducted in
Income Year 2069/70. Advance payment Rs. 35,000.00 was not deductible expenses for the
Income year 2069/70 as per accrual basis of accounting, so, it shall be deducted in Income
Year 2070/71 on cash basis.
iii) Communications and electricity expenses of Rs. 3,500.00 were also eligible expenses for
the Income Year 2069/70 on accrual basis of accounting, so no need to adjustment in Income
Year 2070/71. After making adjustments of inclusion of income and claim of expenditure in
Income Year 2070/71, the income or expenditure shall be accounted on cash basis for
subsequent years.

Example (2): Mr. Devan Mahara is lawer. He is using cash basis of accounting for the
income generated from his profession. He gave an application to the department to change
his basis of accounting from cash to accrual in Fiscal Year (FY) 2067/68. In accordance with
the generally accepted accounting principles, Income Tax Department has given the approval
to account for his income on accrual basis in Fiscal Year 2067/68. The accounts of Mr.
Mahara immediately before changing the basis of accounting, was as below;
i) Service has already been rendered in FY 2066/67 but amount has not been received and
hence no income is included in that FY NPR 80,000
ii) In FY 2066/67 advance has been taken (service has been rendered on FY 2067/68) but
amount is included in that FY on cash basis though service is still to be rendered NPR 30,000
iii) In FY 2066/67 House Rent from Magh, 2066 to Poush 2067 has been paid and expense
has been claimed in FY 2066/67 on cash basis of accounting NPR 48,000
State how you make adjustments of income and expense in FY 2067/6 in above conditions?
CAP II, Dec 2011 (5marks)
Answer:
As mentioned in Chapter 6 of Income Tax Act, 2058, if any person takes the approval from
Inland Revenue Department for the change in basis of accounting for the purpose of taxation,
or his basis of accounting is changed because of various provisions mentioned in the same
Chapter, inclusion of his income, claim of expenditure shall be computed in such a manner
that there is no repetition or short of income or claim of expenditure in that Fiscal Year.

Accordingly, the following adjustments shall be made in Fiscal Year 2067/68;


i) For the service which has already been rendered in FY 2066/67 but no amount is
received, the amount of NPR 80,000 shall be included in income of FY 2067/68 on
accrual basis.

Naresh Singh Bhandari


bnsingh5102@gmail.com Page30
Income Tax Note 2076/77

ii) The amount of NPR 30,000 taken as advance in FY 2066/67 and included in the income
of same FY is not required to be included in FY 2067/68 though service has been
rendered in this FY. The amount of NPR 30,000 already included in income of FY
2066/67.
iii) House Rent of Six months amounting to NPR 24,000 though pertains to FY 2067/68 has
already been claimed in FY 2066/67 on cash basis of accounting, so no further claim is
allowed in FY 2067/68.
After making adjustments of inclusion of income and claim of expenditure in FY 2067/68,
the income or expenditure shall be accounted on accrual basis for subsequent years.

2) Cash Basis Accounting


As per section23, provisions related to cash basis of accounting are as follows:
A person keeping account on cash basis for tax purposes in calculating income from an
employment, business or investment shall;
 Treat an amount as an income when amount is received or made available to the person;
and
 Treat an amount as an expense when the payment is made.

3) Accrual basis of Accounting


As per section 24, provisions related to accrual basis of accounting are follows:
 A person keeping account on accrual basis for tax purposes in calculating income from a
business or investment shall;
 Treat an amount as an income when right to receive is established and
 Treat the following amount as an expenses in the following condition;
o In case the payment constituting the expenses is to be made in return for a
payment or payments received from another person, the expenses shall be treated
as incurred in the following condition;
 The person has obligation to make payment,
 The value of obligation can be determined accurately and
 The other payment has been received.
o In any other case, the expenses shall be treated as incurred when the payment is
made.
 Bank and financial institution shall keep the books of account as per the basis prescribed
by the Nepal Rastra Bank. Co-operative limited may account interest income on cash
basis.
 Foreign exchange difference due to fluctuation in foreign exchange rate shall be adjusted
at the time of settlement of the transaction.

Naresh Singh Bhandari


bnsingh5102@gmail.com Page31
Income Tax Note 2076/77

4) Reversal of amount including bad debts


As per section 25, provisions related to reverse of amount including bad debts are as follows:
 Where a person has accounted for an amount derived or expenses incurred in calculating
the person’s income from an employment, business, or investment the person shall make
an appropriate adjustment in the following case, at the time of refund, recovery,
disclaimer, write-off, or forgiveness occurs;
 The person later refunds the amount or recovers the expenses, as the case requires; or
 In case of the income recorded on accrual basis, the person later disclaims an
entitlement to receive the amount or in case of debt claims, the person write off the
debt as bad; or
 In case of the expenses recorded on accrual basis, the person later disclaims an
obligation to incur expenses or, in case of debt obligation, the person to whom the
debt is owed forgives the debt.
 A person shall disclaim the entitlement to an amount or write off as bad debt claim of the
person in the following case only;
 In case of the debt claim of BFIs, after the debt claim has become bad debt as
determined in accordance with the prescribed standards; and
 In any other case, after the person has taken all reasonable steps in pursuing payment
and the person reasonably believes that the entitlement or debt claim will not be
satisfied.

5) Quantification of Amounts
As per section 27, provisions related to quantification of amount are as follows:
Particulars Section Basis of Valuation
Transfer of assets by one person to another person 27(1ka) Market Value
Amount for Vehicle facility 27(1kha) 0.5% of Salary
Amount for Accommodation facility 27(1kha) 2% of Salary
Amount for: 27(1ga) Actual amount paid
 Facility by chauffeur, kitchen man, guard, gardener or less contribution
other domestic helpers.
 Any meal, refreshment or entertainment; or
 Drinking water, electricity, telephone and like utilities in
respect of accommodation.
Interest Saving on privileged loan from the employer. 27(1gha) Amount saving then
market rate.
Payment other than above 27(1nga) Market value
In case of the transfer of assets (1ka) and other payment(1nga) except above, the time of
payment derived, incurred, made, received or otherwise taken into account for tax purposes
shall be treated as the time of quantification of the amount.
Naresh Singh Bhandari
bnsingh5102@gmail.com Page32
Income Tax Note 2076/77

Note:
 Salary means basic pay plus grade, if any vide circular dated 2059.04.28
 Accommodation associated with security persons like army or police in barrack or
guard or similar is not accounted as accommodation benefits, so no quantification
requires.
 As per rule 13(kha), amount to be included for providing vehicle facility except to the
employee shall be 1% p.a. of market value of vehicle provided.

Example (3): Standard Chartered Bank has appointed AX Consultancy Pvt. Ltd. as Tax
Consultant on Magh 20, 2070 and the effective date of contract is Shrawan 1, 2071. Under
the contract, the bank is liable to pay an annual fee of Rs. 500,000. As per the contract, an
advance amount of Rs. 100,000 is to be paid on appointment date (i. e. on Magh 20, 2070).
AX Consultancy Pvt. Ltd. is in confusion whether the amount of Rs. 100,000 to be included
inits income of Income Year 2070/71 as the appointment letter is already received during the
Income Year. Advise. CAP II, Dec 2014 (5 Marks)
Answer:
As per section 22(3) of Income Tax Act, 2058 a company has to account for its income on
accrual basis for tax purpose. Further, as per section 27(2), the time the payment is derived,
incurred, made, received or otherwise taken into account for tax purpose shall be treated as
the time of quantification of amount. In the given case, AX Consultancy Pvt. Ltd is a
company and has to account for its income on accrual basis for tax purpose. Performance of
service is the time to recognize the income. The time the payment is derived shall be treated
as the time of quantification of the amount. Hence, advance payment is irrelevant to
recognize the income and shall be simply recorded as advance payment received in Income
Year 2070.71. Since the effective date of contract is Shrawan 01, 2071 the service will accrue
only on Income Year 2071.72 and shall be recognized the income accordingly.

Example (4): Mr. A has been working in the capacity of senior manager for XYZ Co. Ltd.
The company provided him a Volkswagon car for official as well as personal use. The
current market price of the car is Rs. 57,00,000. Details of Mr. A ‘s remuneration during the
Income year 2066/67 are as follows:
Salary Rs. 150,000 per month
Grade Rs. 75,000 per annum
Allowance Rs. 7,000 per month
Other Allowances Rs. 9,000 per month
Quantify the amount for the vehicle facility provided for computing Mr. A‘s taxable income
from remuneration for the F/Y 2066/67.
Will the answer be different in case XYZ Co. Ltd. hired Mr. A for four months in the
capacity of consultant for a fee of Rs. 700,000 in total? CAP II, Dec 2010 (5 Marks)
Answer:
As per the provisions of Income Tax Act 2058 and rules made there under, cost free vehicle
facility provided to be used for official and personal use by employer to employee, or worker
or other persons getting remuneration on monthly basis, has to be quantified in terms of value
to be included in his total remuneration for tax purpose. In such cases, value of such free of
Naresh Singh Bhandari
bnsingh5102@gmail.com Page33
Income Tax Note 2076/77

cost vehicle facility has to be quantified at the rate of 0.5% of total salary and grade.
Therefore, amount to be included in his taxable income has been quantified as:

Salary (Rs. 150,000 *12) Rs. 1,800,000


Grade Rs. 75,000
Total salary with grade Rs. 1,875,000

Amount to be quantified = Rs. 1,875,000 * 0.5/100 = Rs. 9,375


Therefore, Rs. 9,375 has to be included in his remuneration on account of cost free vehicle
provided to be used for official as well as personal uses.

In case Mr. A is employed in the capacity of consultant then values for such cost free vehicle
is to be quantified for the purpose of including in his taxable business income. Such value to
be quantified is 1% of the current market price of such vehicle or car.

Car will be quantified as:


1% of Rs. 5,700,000 = Rs. 57,000/3 = Rs. 19,000

Total taxable business income of Mr. A is


Consulting fees Rs. 700,000
Cost free vehicle/car Rs. 19,000
Total Rs. 719,000
Example (5): Mr. Anup pays an advance amounting to Rs. 10 lakhs in cash to Mr. Binod,
who required it urgently to clear his goods from customs. The same was returned by Mr.
Binod in cash after six months. Discuss the tax consequence of the above transaction quoting
the relevant provisions of Income Tax Act, 2058. CAP II, Dec 2009 (5 Marks)
Answer:
Cash payment of Rs. 10 lakhs as advance does not attract the provisions of Section 21, as it is
not a payment of expenses. But Mr. Binod has utilized the amount free of cost for six
months. Though Mr. Binod has paid no amount as interest there on, but utilized the fund for
his business and so according to section 27 of Income Tax Act, 2058, a difference between
the actual rate of interest and market rate of interest should be charged as deemed income
from business of Mr. Binod. In this case, the actual interest is zero and so the amount
calculated for six months at the prevailing market rate shall be included in taxable income of
Mr. Binod.

Example (6): MNO Insurance Company Ltd. has taken a flat on rent for which it pays Rs.
20,000 p.m. and the flat is given to Mr. Puspendra Rana Manager, of Claims Department for
his residence. Mr. Raju Shrestha, Manager Marketing Department resides in another flat of
same area and Mr. Raju gets reimbursement from the company of Rs. 20,000 p.m. only
whereas he pays rent Rs. 30,000 p.m. Basic Salary of both staff is Rs. 50,000 p.m.
Withholding Tax is deducted by Insurance Company & Mr. Raju while making payment to
related house owner. What amount is to be included in taxable salary of both regarding the
above transaction? CAP III, June 2011 (3 Marks)
Answer:
Naresh Singh Bhandari
bnsingh5102@gmail.com Page34
Income Tax Note 2076/77

i) Section 27 (v) (2), of Income Tax Act, 2058 and Rule 13 (2), of Income Tax Rule, 2059
In case of Mr. Puspendra Rana, 2% of Salary income shall be include in taxable income in
case the employer provides residence to the employee. Thus, amount of perquisite to be
included in taxable salary income = Rs. 50000*2%= Rs. 1000 per month.
ii) In case of Mr. Raju, Section 8(2) (v) reimbursement of rent for personal house shall be
included in employment income.
Thus, amount of perquisite to be included in taxable salary income= Rs. 20,000 per month
and he will not get any deduction for rent paid.

6) Quantification in Rupees
As per section 28, provisions related to quantification of Rupees are as follows:

 Income and expenses to be included in calculating income of the person shall be


quantified in Nepali rupees if they are quantified in currency other than rupees.
 The amount quantified in other currency shall be converted in Nepali rupees at the
standard rate applying between the currency and the rupees at the time the amount is
derived, incurred, made, received or otherwise taken into account for tax purposes.
 For the purpose of conversion as above, if department permits by notice in writing, a
person may use the average exchange rate applying during the income year as determined
by Department.

7) Indirect payments
When a person takes indirect benefits from the payers or an associate of the payer or directs
the payee of a payment, the Department may, by notice in writing, treat the person as the
payee of the payment.

8) Jointly Owned Investment


As per section 30, for the purposes of calculating a person’s income from investment that is
jointly owned with another person, amount to be included and deducted in that calculation
shall be apportioned among the joint owners in proportion to their respective interests in the
investments.

Example (7): Ram and Krishna had contributed Rs. 20 lakhs and Rs. 30 lakhs respectively
and deposited the amount into common bank account to be used to purchase a plot of land to
resale the same later on. After 6 months of depositing the money into the bank account, they
purchase a plot of land for Rs. 45 lakhs and got the land registered in their joint name (i.e. in
the name of both Ram and Krishna jointly). Expenses on registration and commission at the

Naresh Singh Bhandari


bnsingh5102@gmail.com Page35
Income Tax Note 2076/77

time of purchase of land amounted to Rs. 500,000. They later on land that plot of land at Rs.
86 lakhs and they have paid Rs. 6 lakhs as sales commission and other incidental expenses
related to sales. The bank has given interest @ 18 % p.a. on such bank deposit. Calculate the
amount to be incorporated as “income” in the hand of Ram and Krishna respectively from
abovementioned transaction. How much would be the interest income and how it is treated in
the hand of Ram & Krishna?
Answer:
As per section 30, joint income and associated expenses is taxed on individual hand. So
computation is to be done at 2:3 ratios as below:
Amount
Particulars Total Ram Krishna Remarks
Expenses
Cost of Plot of Land 4,500,000
Other Expenses 500,000
Total cost of Land 5,000,000 2,000,000 3,000,000 Apportioned on the basis of 2:3
Sales Income
Sales Price 8,600,000
Less: Sales Commission (600,000)
Net Sales Income 8,000,000 3,200,000 4,800,000 Apportioned on the basis of 2:3
Net income 1,200,000 1,800,000

Apart from above mentioned income Ram and Krishna have received Rs. 450,000 from the
bank as interest for 6 months. This interest income shall be shared between Ram and Krishna
in the ratio of 2:3 as follows;
Ram: Rs. 180,000
Krishna: Rs. 270,000
However, the bank will deduct TDS from such interest payment and such withholding shall
be final withholding. Hence, Ram and Krishna need not incorporate the interest income in
their annual income.

Example (8): Mr. Ram & Mr. Shyam jointly owned a house property costing Rs 50,000,000.
At the time of acquisition of the said house property, Mr. Ram had invested Rs. 1 crore and
rest of the amount invested by Mr. Shyam. This house property has been sold out for Rs.
64,000,000, and incurred Rs. 700,000 on account of brokerage commission plus other
incidental charges.
You are required to appropriate between Mr. Ram & Mr. Shyam- as per sec 30 of Income
Tax Act 2058- for the net income that originated from the jointly owned investment.
Answer:
For the purpose of calculating a person ‘s income from an investment which is jointly owned
with another person, amounts to be included and deducted in that calculations should be

Naresh Singh Bhandari


bnsingh5102@gmail.com Page36
Income Tax Note 2076/77

apportioned among the joint in proportion to their respective interests in the investment. [sec
30]

Mr. Ram Mr. Shyam


Investment Rs. 1 crore Rs. 4 crore
Investment ratio is 1:4

Computation of net income:


Sales (Incoming) Rs. 6.40 crore
Less: brokerage commission Rs. 0.07 crore
Balance Rs. 6.33 crore
Total initial investment (Outgoing) Rs. 5.00 crore
Net Income Rs. 1.33 crore

Net income of Rs. 1.33 crore to be appropriated based on contributory ratio of 1:4 between
Mr. Ram & Mr. Shyam are as follows:
Mr. Ram Mr. Shyam Total
Net Income Rs. 2,660,000 Rs. 10,640,000 Rs. 13,300,000

9) Characterization of Compensation Payment


As per section 31, provisions related to characterization of compensation payment are as
follows:
If a person or associate of person derives a compensation amount, including a payment under
insurance, which compensates for the following things, then, at the time of compensation
amount is derived; it shall be included in calculating the income from the employment,
business or investment, as the case requires:
 Compensation for income from or an amount to be included in calculating the person’s
income from business, employment or investment, which the person expects or expected
to derive or
 Compensation for loss or an amount to be deducted in calculating the person’s income
from business, employment or investment, which the persons expects or expected to
incur.
Provided that,
 The compensation payment amount against the compensation of physical loss of a
natural person from a personal accident shall not be included in income and treatment
expenses incurred for the loss from the accident cannot be claimed for tax credit u/s
51.
 Compensation received against death of the natural person shall not be included in
calculating income of the natural person.

Naresh Singh Bhandari


bnsingh5102@gmail.com Page37
Income Tax Note 2076/77

Example (9): There was a fire in Suvas Trading Company Pvt. Ltd. due to which its entire
furniture and office equipments were lost. The company lodged a claim with the insurance
company on 2067.03.25. The company had Rs. 500,000 of tax base for this particular pool of
assets at the end of the year and it claimed Rs. 600,000 from the insurance company as that
was the amount insured. On 2067.08.20 it received Rs. 500,000 as compensation from the
insurance company. You are required to derive the amount that is to be included in the
income and the deduction that can be allowed as per Act. CAP III, Dec 2010 (5 Marks)

Answer:
Above case is a situation where the entire pool of a particular asset is destroyed by fire and
the company has made a claim with the insurance company for the loss of the assets. Section
31 of the Act has prescribed that in case of insurance compensation the compensation should
be included in the income as and when such amount is received. Similarly, when the pool is
dissolved with no assets, then depreciation can be claimed for the entire written down value
as per Section 19.
For the financial year 2066-67
Book value of the pool 500,000
Depreciation expenses 500,000 as no assets left in the pool
Depreciation base for the next year nil
For the financial year 2067-68
Opening balance of the pool nil
Compensation received from insurer 500,000
The amount of compensation therefore is to be included in the income.

Alternatively, the company need not include the full compensation in to income if the
condition mentioned in Section 46 of the Act is fulfilled. In such case the excess of
compensation received than that of the value of retired assets is only considered as income.
There will be timing difference between the loss (involuntary retirement of assets),
compensation receipt and the new assets purchase, therefore after the loss, depreciation will
continuously be charged for such assets. The conditions for this purpose are:
- Company purchases the similar assets within a year,
- Company applies for such facility with Inland Revenue Department,
Example (10): Staff of MNO (P) Ltd., a dealer in liquors, was injured on 01/02/2067 during
the course of stocking of cartons of liquors. He became disabled & the company is going to
compensate him as per the company’s rule. The amount of compensation is Rs. 500,000.
Whether the compensation payment attracts withholding tax and shall it be included in
taxable salary of the staff. CAP III, June 2010 (5 Marks)
Answer:
As per the provisions of section 31 of Income Tax Act 2058, if a person receives the
following amount as compensation apart from the insurance claim, it shall be included in the
employment, business or investment income of the person as the case may be.

Naresh Singh Bhandari


bnsingh5102@gmail.com Page38
Income Tax Note 2076/77

a) Any compensation against income received or probable income of any business,


employment or investment.
b) Any amount of compensation received against any losses or probable losses of a person
from business or investment.
But any compensation received due to personal accident of any individual is not includible in
income and any expenses incurred for treatment of payment is not taxable.
Therefore, it does not attract TDS u/s 88, and shall not be included in taxable income.

10) Characterization of Payments under Annuities, Installment Sales and


Finance Leases

As per section 32, provisions related to characterization of payment under annuities,


installment sales and finance leases are as follows:
 Payment made to the person as following shall be treated as interest and repayment of
capital under a debt claim;
 Payment made under an annuity or
 Payment made by the person acquiring an asset under an installment sale or
 Payment made for the use of the asset under a finance lease.
 Total payment made shall be divided into;
 Capital portion = Market value of assets at time it is sold or leased.
 Interest portion = Total payment – Capital portion
 When determining installment, the portion of interest and capital shall be segregated at
the time of settlement and provide with a list of total payments. Those who cannot
provide with the list, shall be required to treat the interest and capital portion of annuity,
installment sale, or finance lease as a blended loan with the interest compounded six
monthly and divided them according to payments as above.
 A borrower under a blended loan shall be required to make a part in part a payment of
interest and in part a repayment of a capital where the interest part is calculated on capital
outstanding at the time of each payment so as to be uniform rate of interest over the term
of the loan.
 The following condition shall be required to be satisfied while conducting finance lease;
 Transfer of ownership or lessee has an option to purchase the asset at the end of lease
term for the fixed price,
 Lease term exceed 75 % of the useful life of asset,
 Market value of asset after the expiry of lease term is less than 20 % of its market
value at the commencement of lease,
 In case of the lease that commences before the last 25 % of the useful life of the asset,
the present value of minimum lease payment equals or exceeds 90 % of the market
value of the asset at the commencement of the lease term; or
Naresh Singh Bhandari
bnsingh5102@gmail.com Page39
Income Tax Note 2076/77

 The asset is the custom made for the lease and after expiry of the lease term the asset
will not be practical use to anyone other than the lessee.
 The lessee shall be treated as the owner of property leased to the lessee under the finance
lease and the lesser shall be treated as the holder of a debt claim against the lessee.
 Present value of lease is calculated using discount rate equal to standard interest rate.

Example (11): M/s ABC Construction Company Pvt. Ltd., a construction company, has
purchased a heavy duty earth moving machine from M/s XYZ Company Pvt. Ltd. with the
amount payable in five installments which are payable at the beginning of the year. The
amount of each installment is Rs. 10 lakhs. Calculate the amount of interest to be claimed
under Section 14 of the act by showing the year wise interest and repayment of debt claim.
CAP III, Dec 2013 (5 Marks)
Answer:
In the case given,
- Per Installment amount Rs. 10 Lacs payable at the beginning of the year.
- Standard Interest Rate for discounting as per Section 32(8): 15 percent per annum.
Year Installment amount (Rs.) Discount Factor Discounted Value (Rs.)
1 1,000,000 1 1,000,000
2 1,000,000 0.8695 869,500
3 1,000,000 0.7561 756,100
4 1,000,000 0.6575 657,500
5 1,000,000 0.5717 571,700
Present value of payments Rs. 3,854,800
Total payments Rs. 5,000,000
Interest Rs. 1,145,200

Year wise Break Up (Rs.)


Year Opening Balance Installment Interest Portion Capital Portion
1 3,854,800 1,000,000 - 1,000,000
2 2,854,800 1,000,000 428,220 571,780
3 2,283,020 1,000,000 342,453 657,547
4 1,625,473 1,000,000 243,821 756,179
5 869,294 1,000,000 130,706 869,294
Total 5,000,000 1,145,200 3,854,800
The amount of total interest to be claimed as interest under section 14 is Rs. 1,145,200 on
year wise basis as shown above.

11) Transfer pricing and other Arrangement between Associates


As per section 33, provisions related to transfer pricing are as follows;

Naresh Singh Bhandari


bnsingh5102@gmail.com Page40
Income Tax Note 2076/77

 In any arrangement between persons who are associates, the Department may, by notice
in writing, distribute, apportion, or allocate amounts to be included or deducted in
calculating income between the persons as is necessary to reflect the taxable income or
tax payable that would have arisen for them if the arrangement had been conducted at
arm’s length.
 In making any adjustment as above, the Department may;
 Re-characterize the source and type of any income, loss, amount, or payment; or
 Allocate costs, including head office expenses, incurred by one person in conducting
a business that benefit an associate or associates in conducting a business to the
associates based on the comparative turnovers of the businesses.
Example of transfer pricing between holding and subsidiary companies;
Within the Group of business may be between Holding and Subsidiary companies, the profit
making unit may sell the product at the price on gross loss to another unit (associate or
subsidiary or related party organization) which is loss making so as to manipulate tax
liability. Such transaction is especially taken care of while assessing the income of selling
company as well as buying company to manage tax.

12) Income splitting


As per section 34, provisions related to income splitting are as follows:
 Where a person attempts to split income with another person that is likely to cause a
reduction in tax, the Department may, by notice in writing, adjust amounts to be included
or deducted in calculating the income of each person to prevent any reduction in tax
payable as a result of the splitting of income.
 A person having attempted to split income includes, but is not limited to, a reference to a
transfer of the following amounts so as to lower the total tax payable by the person or an
associate, either directly or indirectly through one or more interposed entities, between
the person and the associate:
 Amounts to be derived or costs to be incurred; or
 An amount received or enjoyed by the transferee of an asset that is derived from the
asset; or an amount paid or expenses incurred in owning the asset.
 In determining whether a person is seeking to split income, the Department shall consider
the market value of any payment made for the transfer.

Example (12): ABC Ltd. is holding company with 100% investment in XYZ Ltd. XYZ Ltd.
has accumulated loss and ABC Ltd. is in profit in tax account. Sales and costs of ABC Ltd.
as below:
Sales Rs. 22,450 thousand
Direct cost Rs. 24,450 thousand

Naresh Singh Bhandari


bnsingh5102@gmail.com Page41
Income Tax Note 2076/77

Adm. cost Rs. 200 thousand


General markup is 10% on the direct cost.
How Tax Department treats this transaction of ABC Ltd.’s sale to XYZ Ltd. on tax loop.
CAP III, Dec 2012 (6 Marks)
Answer:
When ABC sells to its related company at loss means there is profit transfer. Tax Department
may split the income as below shown as tax loop and assess the tax liability.
Income Splitting is possible under current tax practice. Within the market price environment,
XYZ can sell at high profit and adjust the accumulated loss as set off with the profit of the
year.
So profit portion of the ABC now is set off with accumulated loss of XYZ thereby nullifying
the tax liability.
In normal situation, there is profit of Rs. 2,245 but shown as loss of 2,200 by ABC.

Particulars Business Profit/(loss) Tax Loop


Sales 22,450 26,895
Direct Cost 24,450 24,450
Administrative cost 200 200
Profit/(loss) (2,200) 2,245

13) General Anti- Avoidance Rule (GAAR)


As per section 35, provisions related to anti avoidance rule are as follows:
 For the purposes of determining liability to tax under this Act, the Department may;
 Re-characterize an arrangement or part of an arrangement that is entered into or
carried out as part of a tax avoidance scheme;
 Disregard an arrangement or part of an arrangement that does not have substantial
economic effect; or
 Re-characterize an arrangement or part of an arrangement the form of which does not
reflect its substance.
 Tax avoidance scheme means any arrangement, one of the main purpose of which is the
avoidance or reduction of tax liability.

Naresh Singh Bhandari


bnsingh5102@gmail.com Page42
Income Tax Note 2076/77

Long Term Contract

1) Definition
Long term Contract:

As per section 2(fa), long term contract is a contract specified in section 26 having term of
more than 12 months.
As per the clarification given in section 26, long term contract of a person means a contract
of the following condition;
 The term of contract exceeds 12 months and
 The contract is either a contract for manufacture, installation, or construction, or, in
relation to each, the performance of related services; or a contract with a deferred return
that is not an excluded contract.

Contract with deferred return:


As per rule 10, any contract of a person shall be a contract with a deferred return if the person
does not show the details as prescribed by the department regarding estimated gain and
estimated loss during every six-month period from the commencement of the contract.

Excluded Contract:
As per rule 11, the following contracts shall be termed as excluded contract:
 A contract created by a reason of an interest in an entity or by obtaining a membership in
a retirement fund; or
 Any contract of investment insurance.

2) Recognition of Revenue & Expenses under Long-Term Contract


As per section 26(1), while calculating a person’s income for an income year from an
employment, business, or investment, estimated cumulative inclusions and deductions under
a long-term contact shall be calculated on the percentage completion method.
As per Rule 12, other provision regarding long-term contact is as follows:
 Cumulative inclusion under a contract of a person at a particular time means amount
under the contract required to be included at the time or a previous time in calculating the
person’s income from an employment, business, or investment, disregarding the
calculations as for a normal income year.
Further, as per NAS 11- Contract, contract revenue shall include following:
 The initial amount of revenue agreed in the contract; and
 Variations in contract work, claims and incentive payment.
Naresh Singh Bhandari
bnsingh5102@gmail.com Page43
Income Tax Note 2076/77

 Cumulative deductions under a contract of a person at a particular time means amounts


under the contract that may be deducted at the time or a previous time in calculating the
person’s income from business or investment, disregarding the calculations as for a
normal income year.
 Percentage of completion shall be calculated as follows:
a) Calculate the actual cost incurred till date.
b) Calculate the estimated contract cost at that date. At each reporting date, the
estimation may vary so it should be calculated separately for each reporting date.
c) % of Completion = (a/b) *100
 Income from the long term contract for each year shall be calculated as follows:
Total Contract Revenue A
Total estimated cost calculated at the date B
Total cumulative income till date C=A-B
Percentage of completion at the date D
Cumulative income till date E=C*D
Cumulative Income till previous year F
Income of the current year E-F

Note: While calculating the contract revenue, price adjustment and actual payment shall
not be considered.

3) Treatment of loss on completion of Contract


As per section 20(4), if a person incurs a loss or has an unrelieved loss available for carry
forward during an income year in which a long term contract of the person’s business is
completed or otherwise disposed off by the person, that is attributable to the long term
contract won by the person through international competitive bidding, the department may by
notice in writing, allow the loss to be:
 Carried back to a preceding income years or years; and
 Treated as unrelieved loss of that year or years in an amount not exceeding the amount by
which inclusions in calculating the income from business to which the long term contract
relates for that year or years exceeds deduction relating to contract.

Example (1): M/s Hazma construction company Limited entered into an agreement with
Rudra Trade Concern to construct a building for the administrative block of Rudra Trade
Concern. The key points of the agreement are as follows:
The contract price is fixed at Rs. 15 Lakhs. The expected period of completion of the contract
is three years. As per the provision for escalation, if the cost of materials and wages go high
at the end of each year by up to 10% of the level at the commencement of the contract, the
remaining contract value will remain unchanged. But if the price goes higher than 10%, the

Naresh Singh Bhandari


bnsingh5102@gmail.com Page44
Income Tax Note 2076/77

remaining contract price will increase by 5%. The contractor at the outset had estimated the
total contract cost at Rs. 12 Lakhs.
The construction work commenced on Shrawan 01, 2066. The portion of the work completed
at the end of each financial period was: 2066-67 = 30%, 2067-68 = 65% and 2068-69 =
100%. The market price of material and wages increased by 7% of the original at the end of
the first year and by 14% at the end of the second year. The actual contract cost during each
financial year was as follows:
2066-67: Rs. 3.78 Lakhs.
2067-68: Rs. 8.48 Lakhs.
2068-69: Rs. 13.31 Lakhs.
You are required to calculate the gain from the long term contract for each year.
CAP III, June 2013 (9 Marks)
Solution:

Particulars 2066/67(Rs.) 2067/68(Rs.) 2068/69(Rs.)


Total Contract Revenue (A) 1,500,000 1,500,000 1,526,000
Total estimated cost at the date (B) 1,277,000 1,327,000 1,331,000
Total cumulative income till date(C=A-B) 223,000 173,000 195,000
Percentage of completion (D) 30% 65% 100%
Cumulative income till date(E=C*D) 66,900 112,450 195,000
Cumulative Income till previous year(F) 0 66,900 112,450
Income of the current year(E-F) 66,900 45,550 82,550

Working Note:
For 2066/67:
The contract Revenue was Rs. 15 lacs because the price escalation was not effective.
The contract cost was estimated as follows:
Original contract cost Rs.12 Lacs
Additional Expenses incurred during the year
(Rs. 3.78 Lacs – Rs. 3.60, 30% of Rs.12 Lacs) Rs. 0.18 Lacs
Expected increase in the rest of expenses Rs. (12-3.6) *7% Rs. 0.59 Lacs
The estimated contract cost was Rs. 12.77 Lacs
For 2067/68:
The contract revenue was the same because the price rise was only 7% and thus the revenue
was Rs. 15 Lacs. The contract cost was estimated as follows:
Original contract cost Rs.12 Lacs
Additional Expenses incurred during the year
(Rs. 8.48 Lacs – Rs. 7.80, 65% of Rs.12 Lacs) Rs. 0.68 Lacs
Expected increase in the rest of expenses Rs. (12-7.8) *14% Rs. 0.59 Lacs
The estimated contract cost was Rs. 13.27 Lacs
For 2068/69:
The contract revenue increased by 5% due to the price escalation clause. Thus the contract
revenue was Rs. (Rs. 15 Lacs + Rs. 15 * 35%*5%) = Rs. 15.26 Lacs.
The final actual contract cost was Rs. 13.31 lacs.
Naresh Singh Bhandari
bnsingh5102@gmail.com Page45
Income Tax Note 2076/77

Example (2): Torrent Co. ltd. is a large public listed company. The main activity of which
involves construction contract. It won a global contract to build a new football stadium
owned by Star Youth Club at a contracted price of Rs. 4,000,000. This construction contract
was to be completed over a period of three years starting from F/Y 20X1/X2. Initially the
total cost of completion of contract was estimated as Rs. 3,600,000.
Star Youth Club has placed additional variation order in 2nd and 3rd F/Y with the details as
given below.
20X2/X3 20X3/X4
Variation Order Rs. 70,000 Rs. 80,000
As a consequence of variation order as aforesaid, it has to incur additional cost which had
been estimated as follows:
20X2/X3 20X3/X4
Additional estimated cost Rs. 50,000 Rs. 420,000
Details of actual accumulated cost over a period of three years to build this football stadium
were as follows:
20X1/X2 20X2/X3 20X3/X4
Actual Accumulated cost to date Rs. 1,800,000 Rs. 2,628,000 Rs. 3,986,000
Rectification work - - Rs. 34,000
The rectification costs are the costs incurred in widening access to the stadium. This was the
result of an error by Torrent’s architect when he made his initial drawing.

Compute the extracts of taxable income over those three years ending 20X3/X4 and the
amount of losses carried back. Dec 2011, CAP-II (15 Marks)

Solution:

Particulars 20X1/X2 20X2/X3 20X3/X4


Inclusions;
Original Contract Price 4,000,000.00 4,000,000.00 4,000,000.00
Variation order - 70,000.00 150,000.00
Total Revenue (A) 4,000,000.00 4,070,000.00 4,150,000.00
Deductions;
Original estimated cost 3,600,000.00 3,600,000.00 3,600,000.00
Additional cost - 50,000.00 420,000.00
Total estimated cost(B) 3,600,000.00 3,650,000.00 4,020,000.00
Estimated profit(C=A-B) 400,000.00 420,000.00 130,000.00
Actual Cost(D) 1,800,000.00 2,628,000.00 4,020,000.00
% of completion(E=D/B*100) 50 72 100
Profit to date(F=C*E) 200,000.00 302,400.00 130,000.00

Naresh Singh Bhandari


bnsingh5102@gmail.com Page46
Income Tax Note 2076/77

Profit till previous year(G) - 200,000.00 302,400.00


Profit for the year(G=F-G) 200,000.00 102,400.00 (172,400.00)
Losses to be carry backward - - (172,400.00)
Adjusted loss 172,400.00 - -

Example (3): The civil works for Lower Tamakoshi Hydropower project is awarded to
Laligurans/Eastwater JV through a global contract process. The contract value is Rs. 2,450
million. The contract shall have to be completed over the period of 5 years. After the works
were awarded, there has been change in the design, to cover for the change variation order
was awarded to JV for Rs. 130 million inthe second year of the contract. The cumulative cost
for respective year has been Rs. 400 million, 800 million, 1,290 million, 1,600 million and
2,000 million respectively. It was estimated that the cost of the construction is 90% of the
value of the contract.

Calculate the taxable income on this long term contract of Laligurans/Eastwater JV for each
of the five years.
Solution:
Particulars 20X1/X2 20X2/X3 20X3/X4 20X4/X5 20X5/X6
Inclusions;

Original Contract Price 2,450.00 2,450.00 2,450.00 2,450.00 2,450.00

Variation order - 130.00 130.00 130.00 130.00


Total Revenue (A) 2,450.00 2,580.00 2,580.00 2,580.00 2,580.00
Deductions;

Original estimated cost 2,205.00 2,205.00 2,205.00 2205 2205

Additional cost - 117.00 117.00 117 117


Total estimated cost(B) 2,205.00 2,322.00 2,322.00 2,322.00 2,322.00

Expenses incurred this year 400.00 400.00 490.00 310.00 400.00

Cumulative Expenses till date 400.00 800.00 1,290.00 1,600.00 2,000.00


Additional cost to complete
Contract 1,805 1,522 1,032 722

% of Completion 18.14 34.45 55.56 68.91 100.00

Income included for tax 444.44 888.89 1,433.33 1,777.78 2,580.00


Income considered till last - 444.44 888.89 1,433.33 1,777.78

Naresh Singh Bhandari


bnsingh5102@gmail.com Page47
Income Tax Note 2076/77

year

Income for current year 444.44 444.44 544.44 344.44 802.22


Allowed expenses for the year 400 400 490 310 400

Taxable Income 44.44 44.44 54.44 34.44 402.22

Alternatively, the answer can be calculated as follows;


Particulars 20X1/X2 20X2/X3 20X3/X4 20X4/X5 20X5/X6
Inclusions;
Original Contract Price 2,450.00 2,450.00 2,450.00 2,450.00 2,450.00
Variation order - 130.00 130.00 130.00 130.00
Total Revenue (A) 2,450.00 2,580.00 2,580.00 2,580.00 2,580.00
Deductions;
Original estimated cost 2,205.00 2,205.00 2,205.00 2205 2205
Additional cost - 117.00 117.00 117 117
Total estimated cost(B) 2,205.00 2,322.00 2,322.00 2,322.00 2,322.00
Estimated profit(C=A-B) 245.00 258.00 258.00 258.00 258.00
Actual Cost(D) 400.00 800.00 1,290.00 1600 2000
% of completion(E=D/B*100) 18.14 34.45 55.56 68.91 100
Profit to date(F=C*E) 44.44 88.89 143.33 177.78 580.00
Profit till previous year(G) - 44.44 88.89 143.33 177.78
Profit for the year(G=F-G) 44.44 44.44 54.44 34.44 402.22

Naresh Singh Bhandari


bnsingh5102@gmail.com Page48
Income Tax Note 2076/77

Calculation of Net Gain from Assets and Liabilities

1) Definition
2(Da): Non-Business Chargeable Asset: means land, building and any interest or securities
in an entity excluding the following assets:
 Business assets, depreciable assets or trading stock.
 A private building of a natural person that has been-
 Owned continuously for ten years or more; and
 Stayed in by a natural person continuously or intermittently for a total of ten years or
more.
 An interest of a beneficiary in a retirement fund.
 Land and a private residence of a natural person that is disposed off for less than Rs. 10
lakhs
 Assets of the natural person that is disposed off by transfer in any other way than
purchase and sale within three generations.

2(Ya): Disposal: means disposal of an asset or liability including sale or transfer as


mentioned in section 40.

2(Shha): Market Value: of an asset and service denotes the normal transaction price for the
asset or service in the ordinary course of a business amongst unrelated persons.

2(KaTta): Business Asset: means an asset to the extent to which it is used in a business.
Provided that, the term does not include trading stock or a depreciable asset of business.

2(KaDha): Asset: means a tangible or an intangible asset and includes currency, goodwill,
know-how, property, an owner’s interest in a foreign branch, a right to income or future
income and a part of an asset.

2(KaRa): Depreciable Asset: means an asset to the extent to which it is used in the
production of income from business or investment and that is likely to lose value because of
wear and tear, obsolescence, or passing of time.
Provided that, the term does not include trading stock.

Example (1) Mr. Rich has purchased a private building amounting to Rs. 2 crores on Kartik
23, 2060 at Kathmandu. He sold such building on Rs. 3 crores on Poush 26, 2070. During
such period, Mr. Rich was gone abroad, intermittently for a period of 120 days. Will such
building be considered as "Non-Business Chargeable Assets"? What will be your answer if
such building was sold after two months (i.e. on Falgun 26, 2070)?
CAP II, Dec 2014 (5 Marks)

Naresh Singh Bhandari


bnsingh5102@gmail.com Page49
Income Tax Note 2076/77

Answer:
Section 2 (da) (2) of Income Tax Act, 2058 has excluded the following assets from the
definition of "Non-Business Chargeable Assets" in case of an individual;
A private building of an individual that has been;
 Owned continuously for ten years or more; and
 Lived in continuously or intermittently for a total period of ten years or more.
In the given case, Mr. Rich has owned his private building for a period of ten years in which
first condition is satisfied. But he lived in for a period of less than ten years in which second
condition is not satisfied.
Hence, such building is to be considered as "Non-Business Chargeable Asset" as per Income
Tax Act, 2058.
But, in case where Mr. Rich sold such building after two months (on Falgun, 26, 2070)
second condition of 'lived in continuously or intermittently for a total period of ten years or
more' will also be fulfilled and hence such building shall not be considered as "Non-Business
Chargeable Asset".

2) Net Gain from Asset and Liabilities


As per section 36, provisions related to Net gain from Asset and Liabilities are as follows:
 Net gain from disposal of business asset and liabilities of a business of a person for an
income year is calculated as follows:
Gain from disposal of business asset or liabilities of business ×××
Less: Losses from disposal of business asset and liabilities during the year ×××
Less: Unrelieved net loss out of other business losses for the year ×××
Less: Unrelieved net loss for previous year (including other business) ×××
Net Gain from disposal of business asset or liabilities of a business ×××
 Net gain from disposal of Non-business chargeable asset and liabilities of an investment of
a person for an income year is calculated as follows:
Gain from disposal of NBCA of an Investment ×××
Less: Losses from disposal of NBCA of investment during the year ×××
Less: Unrelieved net loss out of business or investment for the year ×××
Less: Unrelieved net loss of previous year (including other business and Investment) ×××
Net Gain from disposal NBCA of an investment of the person for an income year ×××
 Loss suffered from disposal of an asset or liability with a foreign source shall be allowed
for deduction from the gain from disposal of an asset or liability with a foreign source
only.
 Net Loss of business for an income year = loss from disposal of business or liabilities of
business during the year – Gains from disposal of business or liabilities of business during
the year.

Naresh Singh Bhandari


bnsingh5102@gmail.com Page50
Income Tax Note 2076/77

 Net Loss of an investment for an income year = Losses from disposal of NBCA of
investment during the year – Gains from disposal of NBCA of the investment during the
year.
Example (2): Mr. Shyam has an income of Rs. 120,000 from the disposal of business assets.
He has to adjust the loss from the disposal of non-business chargeable assets amounting to
Rs. 50,000 and he has a loss of Rs. 24,000 from the asset of a foreign source. State your view
regarding the computation of taxable capital gain.
CAP III, June 2014 (2 Marks)
Answer:
Mr. Shyam has to include Rs. 120,000 as part of his taxable business income. He is not
entitled to deduct Rs. 50,000 as set off from the income because as per section 36 a los from
the disposal of non-business chargeable asset can be set off from income from another non
business chargeable asset only. Further, the loss from assets situated in a foreign country
cannot be set off from the income from asset or income from a business in Nepal. So the loss
of Rs. 24,000 is also not allowed for deduction.

3) Gain and Loss from Assets and Liabilities


As per section 37, provisions related to gain and loss from assets and liabilities are as follows:
 Gain of a person from disposal of assets or liability;
Incomings for an asset or liability ×××
Less: Outgoings for the asset or liability at the time of disposal ×××
Gain from Disposal of asset or liability ×××
 Loss of a person from disposal of assets or liability;
Outgoing for the assets or liability ×××
Less: Incoming for the asset or liability at the time of disposal ×××
Loss from disposal of asset or liability ×××

4) Outgoing and Net Outgoings for Assets and Liabilities


As per section 38, provisions related to outgoings and net outgoings are as follows:
 The following expenses shall be included in calculating the outgoing for an asset or
liability of a person:
 In case of asset;
 Expenses incurred in acquiring the asset
 Cost of construction or production of asset, if relevant,
 Amount required to be included in calculating the person’s income by reason of
the acquisition of the asset.

Naresh Singh Bhandari


bnsingh5102@gmail.com Page51
Income Tax Note 2076/77

 Expenses incurred by the person in owing the asset or owing the liability including
expenses of;
 Altering,
 Improving,
 Maintaining the asset or liability and
 In case of asset, repairing of asset.
 Expenses incurred by the person for the disposal of asset or liability; and
 Incidental expenses incurred by the person in the acquisition of asset or incurring of
liability and in disposing of asset or liability.
 Following expenses shall not be required to include in calculation of outgoings;
 Expenses not allowed u/s 21 and
 Expenses to the extent to which they may be deducted in calculating the person’s
income.
 Net outgoing for an asset or liability at a particular time = cumulative outgoings –
cumulative incoming at that time.

5) Incomings and Net Incomings for Assets and Liabilities


As per section 39, provisions related to incomings and net incomings are as follows:
 The following amount shall be included in calculating incomings for asset or liability;
 Amount derived by the person in respect of incurring liability;
 Amount derived by the person in respect of;
 Owing the asset or owing the liability,
 Altering or decreasing the value of asset or
 Increasing the liability
 Amount derived by the person in respect of disposal of asset or liability.
 Net incoming for an asset or liability to a particular time = Cumulative incomings –
cumulative outgoings at that time.

6) Disposal of an Asset or Liability


As per section 40, provisions related to disposal of an asset or liabilities are as follows;
 A person disposes an asset when;
 A person parts with ownership of asset,
 The asset is distributed by the owner of asset,
 Asset is merged with other asset or liability,
 Asset is leased to another person under finance lease,
 Asset is cancelled, destroyed, lost, expired or surrendered.

Naresh Singh Bhandari


bnsingh5102@gmail.com Page52
Income Tax Note 2076/77

 A person disposes liability when;


 A person parts with the obligations constituting the liability,
 Liability is satisfied, cancelled, released, expired; or
 Merged with another liability or an asset.
 In the following case, an asset or liability shall be deemed as disposed by the person;
 In case of natural person, immediately before the death of natural person;
 In case of asset, when,
Incoming for asset > Outgoings for assets
 In case of asset that is debt claim, when;
 In case of debt claim of BFIs, the debt claim becomes bad debt as determined in
accordance with the prescribed standard.
 In any other case, the person reasonably believes that the debt claim will not be
satisfied. Provided that, the person has to take all reasonable steps in pursuing th
debt claim.
 In case of asset that is a business asset, NBCA, depreciable asset, or trading stock,
immediately before the person begins to use the asset in such a way that it ceases to be
an asset of the type it was immediately prior to that use;
 In case of an entity, if the ownership changes by 50 % or more as compared to 3 years
previously (Section 57);
 Other than in case of land and building situated in Nepal, immediately before the
person becomes a non-resident person.
 In case of disposing an asset under finance lease, the person to whom the asset is leased
shall be as acquiring the ownership of the asset at the time of disposal.
 While calculating gain from disposal of asset or liability, the following provisions shall
apply;
 Net outgoings for asset owned by the person at the time of commencement of this act
= MV of asset at that time; and
 Net incomings for a liability owed by the person at the time of commencement of this
act = MV of liability at that time.

Example (3): Mr. Ram Bahadur Sinjali has migrated to UK on 10th Poush 2068. He has
equity shares in commercial banks purchases at Rs. 500,000 and a house which he recently
purchased for Rs. 10,000,000. These properties he could not sell and therefore decided to
leave in the country and went out of the country on permanent basis. He has the plans to sell
the shares and houses later when he will come to Nepal on visit. Explain the tax implication
on the assets while he is going out of the country and the time when he will come back and
sell. CAP III, June 2012 (5 Marks)
Answer:

Naresh Singh Bhandari


bnsingh5102@gmail.com Page53
Income Tax Note 2076/77

As per the normal provisions of the act, when the ownership of the asset is relinquished, it is
considered that the asset is sold, however there as special provisions in section 40(3) of the act
which has prescribed the conditions on which the asset are deemed to have been sold even if
the ownership is not transferred.
Provision clarifies that where if the owner of asset other than land or land & building,
becomes non-resident then by the operation of section 40(3), the asset is deemed to have been
sold at the prevailing market rate. The act clearly provides the exception for land and building.
It therefore is applicable for all other assets. The assets are deemed to have been sold the day
on which the person becomes non-resident.
In this case the shares in commercial bank is deemed to have been sold on the day the person
goes abroad, because from then on the person is going to be non-resident. The market price of
the share (stock exchange price) is considered the consideration for the shares sold and the
cost price will be the cost he incurred while purchasing those shares. The net will be
considered to have been realized by Mr. Sanjil and he will have to pay tax.
Subsequently, when he will come to the country to sell these asses, he will have to consider
the cost of the share to be the earlier notional income, while the income will be the market
price/transaction price of the shares sold on the date of the transaction.
There will be special treatment for the land and building sale as these are excluded from
presumed sale of assets mentioned in section 40(3), therefore goes as per the normal process
prevailing at the time the transaction is done.
Since the house was recently purchased and he is non-resident, he will not be eligible for any
concessional rate of tax on the sale of house and he would have to pay tax at 25 % on the
gains on sale of the house as well as shares, as per clause 1(8) of schedule 1 of income tax act
2058.

Example (4): JKL Limited purchased 100 shares of MNO Bank Limited for each Rs. 200
each including brokerage and other expenses, as on Chaitra 19, 2058, the company was
holding 250 shares of MNO including the shares purchased and bonus shares paid by MNO.
The quoted closing price on Chaitra 18th, 2058 on MNO shares was Rs. 800 each.
Calculate the gain from the disposal of the shares under the circumstances given below:
i) On Jestha 15, 2071 the company had disposed of the total shares at the rate of Rs. 1,000
each. The brokerage paid was 1.2% of the sales amount.
ii) On Jestha 15, 2071 the company had disposed the total shares at the rate of Rs. 750 each.
The brokerage paid was 1.2% of the sales amount.
iii) On Falgun 10, 2070 the company had received 125 bonus shares for the 250 shares held
by it. On Jestha 15, 2071 the company had disposed 250 shares at the rate of Rs. 750 each.
The brokerage paid was 1.2% of the sales amount. CAP II, Dec 2014 (10 Marks)
Answer:
i) Solution under the first circumstance:
Cost of shares (100 shares @ Rs. 200 each Rs. 20,000
Cost of bonus shares (150 Bonus Share) Nil
Total Cost of 250 shares Rs. 20,000

Naresh Singh Bhandari


bnsingh5102@gmail.com Page54
Income Tax Note 2076/77

But the index value of the shares as on Chaitra 18, 2058 was Rs. 800 per share and so the
deemed cost of the 250 shares shall be Rs. 800×250=Rs. 200,000 + Rs. 3,000 (1.2% of the
sale proceeds for brokerage on sales.)
The amount received from the disposal is 250×Rs. 1000 = Rs. 250,000.
Thus the gain is of Rs. 250,000 - Rs. 203,000 = Rs. 47,000.

ii) Solution under the second circumstance:


The index value of the shares as on Chaitra 18, 2058 was Rs. 800 per share and so the deemed
cost of the 250 shares shall be Rs. 800×250=Rs. 200,000 + Rs. 2,250 (1.2% of the sale
proceeds for brokerage on sales.)
The amount received from the disposal is 250×Rs. 750 = Rs. 187,500.
Thus the loss is of Rs. 187,500-Rs. 202,250 = Rs. 14,750.

iii) Solution under the third circumstance:


The index value of the shares as on Chaitra 18, 2058 was Rs. 800 per share and so the deemed
cost of the 250 shares shall be Rs. (800×250) = Rs. 200,000
Plus, the cost of 125 bonus shares received after Chaitra 18, 2058 = Rs. Nil

The total cost of 375 shares is Rs. 200,000 and so the cost of one share comes to Rs. = 533.33.
The cost of 250 shares sold comes to Rs. 133,332.50+2,250 (1.2% of the sale proceeds for
brokerage on sales).
The amount received from the disposal is 250×Rs. 750 = Rs. 187,500.
Thus the gain is of Rs. 187,500-Rs. 135,582.50 = Rs. 51,917.50.

Example (5): Mr. X is a super distributor of a reputed Indian Drug Manufacturer. Mr. X died
on 31 Chaitra 2066. Mr. X has two sons Mr. Y and Mr. Z. Both of his sons are engaged in
retail sales of medicines. Since the business of super distributorship was owned by Mr. X,
both of his sons have equal share in the business. Since they are engaged in separate
businesses, on death of his father they agreed to continue the business and hired Mr. A, their
brother in law, to look after their father's business. They could not change the legal status of
their father's business till the end of Ashadh 2067.
On 25 Ashwin 2067 Mr. A prepared statement of income for full year 2066/67 and signed the
documents on behalf of the firm. The IRO refused to accept the statement on the ground that
Mr. A is not authorized to submit the statement of Income.
Required:
i) Advise Mr. Y and Z regarding the course of action to be taken.
ii) Mr. Y received 15% discount on all purchases from his father ‘s business. The firm
provides discount of 10% to other customers. Mr. Y was provided discount of 10 after
death of his father. Advice regarding treatment of discount given to Mr. Y.
CAP III, June 2011 (10+5 Marks)
Answer:
i)
a) Section 96(2) kha provides that the statement of income should be certified by either the
person or the manager. Assuming that Mr. A has powers to make decision he will be

Naresh Singh Bhandari


bnsingh5102@gmail.com Page55
Income Tax Note 2076/77

considered as manager under section 2 (ka chha). Therefore, Mr. A is authorized to


submit the statement of Income.
b) Since Mr. X died in the middle of the year and the interests of the business has vested
among his sons, status of the firm has changed to partnership.
c) Since status of the firm has changed to partnership statement of income should be
submitted separately for proprietorship firm and partnership.
d) Therefore, the tax officer has rightly refused to accept the statement of income.
e) Since the deadline for submission of statement is on the same day, Mr. Y and Z should
apply for extension of time for submission of statement of Income.
f) Statement of income for proprietorship firm and partnership should be separately
prepared and submitted within the time.

ii)
a) Mr. Y is relative of Mr. X. Thus he is associated person within the meaning of section
2(ka na). Therefore, treatment of discount received by Mr. Y is correct under Sec. 33.
b) Since the discount given to Mr. Y is transaction between associated, it has to be judged as
per arm length transaction. Since, Mr. Y is given more discount than others for no
justified reason; it cannot be termed as arm length transaction.
c) In view of the tax officer shall be authorized to re quantify expense and income of Mr X
and Y respectively under sec.33(1).
d) Thus, while preparing statement of Income of Mr. X, excess discount given to Mr. Y
should be disallowed.
e) While preparing Statement of Income of Mr. Y, excess discount should not be considered
and should be added in the purchases. But it is not necessary for the tax officer to do so in
every case, depending on the quantum of amount involved.

7) Disposal with Retention of Asset or Liability


As per section 41, provisions related to disposal with retention of assets or liabilities are as
follows:
 In case of deemed disposal of asset and liability, following provisions shall be applied;
 In case of asset;
 Income from disposal = MV at the time of disposal, and
 For the purpose of subsequent disposal-
Net outgoings for the asset at the time of disposal = Amount derived from disposal.
 In case of liability;
 Expenses incurred for disposal = MV of liability at the time of disposal, and
 For the purpose of subsequent disposal-
Net incomings for the liability to the time = Amount of Outgoings.

Naresh Singh Bhandari


bnsingh5102@gmail.com Page56
Income Tax Note 2076/77

8) Disposal by way of Installment Sale or Finance Lease


As per section 42, provisions related to disposal of asset by way of installment sale or finance
are as follows;
When a person disposes an asset by way of installment sale of finance lease (under transfer
between associated persons and other non- market transfers);
 Income from disposal for the person disposing asset = MV at the time of disposal,
 Cost of asset for the person acquiring asset = Income from disposal for the person
disposing asset

9) Transfer of Asset to Spouse or Former Spouse


As per section 43, provisions related to transfer of asset to spouse or former spouse are as
follows;
If a natural person disposes an asset by way of transfer to spouse or former spouse as a part of
divorce settlement or bona fide separation agreement and an application is made to the
department for the application of this section, then;
 Income from disposal for the person disposing asset = Net outgoings for the asset for
immediately before disposal and
 Expenses incurred by spouse or former spouse in acquiring asset = Income from disposal
for the person disposing asset as above.

Example (6): Mr. Ram had purchased a property on 2068.07.12 for Rs. 1 crore. 2069.10.15,
Ram gets divorce from his wife and during the settlement he transfers the property to his wife
Ms. Sita without any financial consideration. Ms. Sita incurs Rs. 20,000 as transfer charges to
transfer the property in her name. Mr. Ram notifies the Inland Revenue Department about his
option of application of Section 43 of the Income Tax Act in writing. On 2069.12.15 Ms. Sita
sold the property for Rs. 1.5 crores. Derive the amount that is taxable as gain in the hands of
Mr. Ram and Ms. Sita. What would be the impact if Mr. Ram does not notify the department
about his option? CAP III, June 2013 (5 Marks)
Answer:
Section 43 of the Income Tax Act prescribes the basis of valuation of properties which are
transferred by the couple to other as per the process of their separation.
In the given case Mr. Ram transferred the property as part of settlement during the separation
therefore for his case the net gain will be as follows:

Incomings Rs. 10,000,000


Outgoings for the property Rs. 10,000,000
Net gain Nil

Income calculation in the hands of Ms. Sita;


Incomings Rs. 15,000,000

Naresh Singh Bhandari


bnsingh5102@gmail.com Page57
Income Tax Note 2076/77

Outgoings for the property


Value of the property transferred Rs. 10,000,000
Registration expenses incurred Rs. 20,000
Net gain Rs. 4,980,000

In above situation if Mr. Ram does not notify his selection of options u/s 43 then he will have
to pay tax in the income calculated u/s 45 of the Act which prescribes of procedure of
determination of income where the property is transferred to the related parties without
considerations.

Example (7): Mr. Ram had purchased a land on 2063.10.01 for Rs 1 crore. On 2066.05.06, he
divorced his wife Mrs. Seeta. As a part of divorce settlement, he transferred the land to his ex-
wife Seeta on 2066.05.10. At the time of transfer, the market value of the land was Rs. 1 crore
40 lacs. During the transfer, Mrs. Seeta incurred Rs. 10,000 toward registration and other legal
charges. Mr. Ram has informed the details of transfer of land to his ex-wife and elected the
application of section 43 in writing to Inland Revenue Office. Mrs. Seeta has sold the land on
2066.05.10 for Rs. 1 crore 50 lacs. By mentioning the relevant provisions of section 43,
calculate gain or loss on disposal of the land in the hands of both Mr. Ram and Mrs. Seeta. If
they do not elect the application of this section, how incomings and outgoings are calculated?
CAP II, Dec 2012 (7 Marks)
Answer:
As per section 43 of the act, in case a property is transferred due to a divorce settlement or to
one’s husband, wife, ex-husband or ex-wife and election has been done for this section, then;
- The net outgoings incurred by the transferor will be treated as if they have been received
from transfer and
- The same amount will be treated as net outgoings for the transferee.
Since, election has been done for the application of section 43, gain or loss is calculated as
follows;

In the hands of Mr. Ram;


Incomings 1 Crore
Less: Outgoings 1 Crore
Gain or loss Nil
The transferor has neither gains nor loss from the disposal of property.

In the hands of Mrs. Seeta;


Incomings 15,000,000
Less; Outgoings
Cost of transferred asset 10,000,000
Cost of registration and legal charges 10,000 10,010,000
Gain from disposal of land 4,990,000
If they do not elect the application of this section, then the market value of the property or the
actual outgoings, whichever is higher is treated as incomings for the transferor and outgoings
for the transferee.

Naresh Singh Bhandari


bnsingh5102@gmail.com Page58
Income Tax Note 2076/77

Thus, if they do not elect section 43 to be applicable in this case, the gain shall be calculated
as under;

In the hands of Mr. Ram;


Incomings 14,000,000
Less: Outgoings 10,000,000
Gain or loss 4,000,000
The transferor experiences Rs. 40 lacs as gain from disposal of the property.

In the hands of Mrs. Seeta;


Incomings 15,000,000
Less: Outgoings
Deemed cost of transferred asset 14,000,000
Registration and legal charges 10,000
Gain from disposal of land 990,000

10) Transfer of Asset on Death


As per section 44, provisions related to transfer of asset on death are as follows;
When a person disposes an asset on death by way of transfer of ownership of asset to another
person and an election for this section to apply is made by spouse of former spouse in writing,
then;
 Income from disposal of a natural person disposing asset = MV of asset at the time of
disposal and
 Acquisition cost incurred by the person acquiring asset = Income from disposal of a
natural person disposing asset as above.

11) Involuntary disposal of Asset or liability with Replacement


As per section 46, the provisions related to involuntary disposal of Assets or liabilities with
replacement are as follows;
 If a person involuntarily disposes an asset (as prescribed in section 40 (1)), and the person
acquires ownership of a replacement asset of the same type within 1 year of the disposal
and elects in writing for this subsection to apply, then;
 Amount derived by the person from disposal = Net Outgoing of the asset immediately
before disposal + (Sales proceeds from disposal - Expenses incurred in replacement)
and
 Expenses incurred by person in replacement of assets = Net Outgoings for the asset
immediately before disposal + (Cost incurred in acquiring replacement asset – sales
proceeds from disposal)

Naresh Singh Bhandari


bnsingh5102@gmail.com Page59
Income Tax Note 2076/77

 If a person involuntarily disposes liability (as prescribed in section 40(2)), and the person
incurs a replacement liability of the same type within 1 year of the disposal and elects this
section to apply in writing, then;
 Expenses incurred for disposal of liability = Net incomings for the liability
immediately before the disposal – (Cost incurred for the disposal – Amount derived in
respect of incurring replacement liability)
 Amount derived in respect of incurring replacement liability = Net incoming for the
liability immediately before the disposal + (Amount derived in respect of incurring
replacement liability – Cost incurred for the disposal)
 The circumstances shall be as prescribed in which the replacement of one security in an
entity with another security in an entity as a result of conversion of the security or
reconstruction of the entity constitutes an involuntary disposal.
As per rule 16, condition arising involuntary disposal with replacement are as follows;
 If a person replaces a security in an entity with another security in the entity or with a
security in another entity under a merger or reconstruction of the entity, then such
conditions shall be treated as an event of involuntary disposal with replacement.
 If the event of involuntary disposal takes place as above, the person or the entity shall file
an application with the department for the approval.
 The department may give approval to the application file as above.

Example (8): Mr. Ram has purchased a land situated at Bhaktapur on 2062.05.01 for Rs. 50
lacs. During the extension of Arkino Highway, government took possession of land with a
notice of compulsory acquisition on 2066.05.07. As compensation, Nepal government has
given a compensation of Rs. 80 lacs to Mr. Ram. Mr. Ram purchased another land
approximately 2 km away on 2067.04.05 for Rs. 60 lacs. Mr. Ram has requested in writing to
Inland Revenue office for the application of section 46 of the act. Compute the gain or loss in
the hands of Mr. Ram by mentioning the relevant provisions of the act. Will the amount of
gain or loss differ if he has not elected for the application of this section?
CAP II, Dec 2012 (7 Marks)
Answer:
Section 46 of Act envisages the circumstances when a person has to dispose of an asset or
liability due to orders issued by the government or due to changes in legal decisions or when a
person has dispose an asset or liability due to forceful conditions. The conditions availing this
section are as under;
- There must be the case of the involuntary disposal of asset or liability.
- The asset or liability is exchanged for another same kind of asset or liability within 1 year
of disposal.
- The person requests in writing for the application of this section for the disposal with
replacement.

Naresh Singh Bhandari


bnsingh5102@gmail.com Page60
Income Tax Note 2076/77

Section 46 further states that the total incomings for the transferred asset shall be the sum of
the following amounts;
- The net outgoings for the assets up to the disposal of the asset.
- In case, the amount received from involuntary acquisition exceeds the amount paid or
payable for the replaced asset, the excess amount.
In the Given case;
Compensation received from the Government Rs. 8,000,000
Cost of replaced Asset Rs. 6,000,000
Excess Rs. 2,000,000
Net outgoings for the asset before disposal Rs. 5,000,000
Total Incomings Rs. 7,000,000
Total Outgoings Rs. 5,000,000
Gain from disposal Rs. 2,000,000

If he has not selected for the application of this section, then the gain is calculated as under;
Incomings Rs. 8,000,000
Less; Outgoings Rs. 5,000,000
Gain from disposal Rs. 3,000,000

Example (9): Company A owns a clothing retail outlet at Koteswor, Kathmandu. The
government has decided to upgrade a highway between Koteshwor and Bhaktapur.
Unfortunately, Company A’s store is in the proposed path of the highway. The Government
serves Company A with a notice of compulsory acquisition. During year 1 the government
pays company A Rs. 5 million compensations for the acquisition of its building and Rs. 3
million for the acquisition of the land on which the building is situated. It also pays company
A Rs. 1 million for loss of profit due to disruption of business. At the time of acquisition,
Company A has net outgoings for the land of Rs. 2 million and the written down value of its
class A pool of depreciable assets (which included buildings only) is Rs. 6 million. Before the
end of year 1, Company A buys a new store in Kathmandu. It pays Rs. 2.5 million for the land
and Rs. 6 million for the buildings. Now you are requested to advice about procedural aspect
and tax implication in such involuntary disposal of the assets as well as the loss of profit.
Your answer must be based on the provisions of the Act.
Answer:
There are three primary issues arising from the compulsory acquisition;
i) Disposal of land
ii) Disposal of Building
iii) Compensation for losses of Profit

i) Disposal of Land; this is governed by section 46.


As per section 46, gain from disposal of the land is calculated as follows:
a) Amount derived from disposal of land;
Compensation received from government 3 million
Expenses incurred for replacement assets 2.5 million
Naresh Singh Bhandari
bnsingh5102@gmail.com Page61
Income Tax Note 2076/77

Excess of compensation over replacement cost 0.5 million


Net outgoings immediately before disposal 2 million
Amount derived from disposal 2.5 million
b) Expenses incurred in replacement of assets
Expenses incurred for replacement assets 2.5 million
Compensation received from government 3 million
Excess of replacement cost over compensation Nil
Net outgoings immediately before disposal 2 million
Expenses incurred for replacement of assets 2 million
Gain from disposal of Land 0.5 million
This gain shall be included in the taxable income of year 1.

ii) Disposal of Buildings


Building is a depreciable asset subjected to depreciation i.e. depreciable assets. So loss or
gain shall be calculated according to schedule 2(4).

Opening balance 6 million


Less; Compensation received 5 million
Remaining WDV 1 millions
New Building acquired 6 million
Depreciation base at the end of year 1 7 Million
On this value, depreciation can be claimed in year 1.
Assumption; Building is acquired before Poush end of the year.

iii) Compensation for losses of Profit


Compensation of Rs. 1 million for loss of profit for the temporary closure of business.
This situation is dealt with under section 31 of the act. If the amount compensates for an
amount that was expected to be included in calculating company A’s income from
business, then the compensation amount must be included. So that the company A must
include the compensation received amount of Rs. 1 million in his income from business
under section 7(2)(h).

Example (10): Smart Pvt. Ltd. purchased a piece of land on Ashwin 25, 2064 for Rs.
1,000,000 and incurred an expenditure of Rs. 150,000 on registration and brokerage. It
constructed a building on the land costing Rs. 2,500,000. The building was ready for use on
Shrawan 25, 2065. The depreciated value in block A was Rs. 1,000,000 at the end of year
2064/65 excluding the cost of building under construction. During the year 2065/66 the
company capitalized Rs. 60,000 as repair and improvement cost in block A.
On Shrawan 1, 2066, the company sold the land and newly constructed building for Rs.
4,000,000. The market value of land on that date was Rs. 1,500,000 and that of building was
Rs. 2,500,000. You are required to calculate taxable gain for the disposal of land and building
for the year 2066/67.
Answer:
Calculation of Gain or loss from disposal of Land
Selling Price of Land Rs. 1,500,000

Naresh Singh Bhandari


bnsingh5102@gmail.com Page62
Income Tax Note 2076/77

Less: Purchase Price Rs. 1,000,000


Less; Registration and Brokerage Charge Rs. 150,000
Gain on disposal of land Rs. 350,000

Calculation of Gain or Loss from disposal of Building


Depreciation basis at the end of year 2064/65 1,000,000
Add. Absorbed addition during the year 2,500,000
Depreciation base for 2066/67 3,500,000
Depreciation rate 5%
Depreciation for the year 175,000
WDV after depreciation 3,325,000
Add; Repairs Capitalized 60,000
3,385,000
Less: Sale of Building 2,500,000
Depreciable basis for 2066/67 885,000

Example (11): Mr. Komal purchased a piece of land at Rs. 30 lakhs. He sold the land at Rs.
45 lakhs. He paid registration expenses Rs. 2 lakhs for this land. In this case, what would be
the tax implications on the following situations?
i) The land was purchased on Chitra 2064 and sold it on Magh 2070.
ii) The land was purchased on Magh 2067 and sold it on Magh 2070.
iii) The land was purchased on Chitra 2059 and sold it on Baishak 2071.
iv) If selling and buying of the land were completed through a sole shareholder of a Pvt. Ltd.
The shareholder is Mr. Komal. CAP II, Dec 2014 (5 Marks)
Answer:
Land and building disposed for a value less than 30 lakhs by any natural person is not treated
non-business taxable assets as per Section 2(Da). The disposal value more than the limit shall
be taken into account in computing the gain and loss on income from investment. Tax liability
is calculated on the basis of the net gain from the disposal of those assets. Withholding tax is
managed for such transactions under section 95 Ka (3) of the Act for natural person. If the
land is sold after holding it for more than 5 years, the tax is 2.5 % and if it is sold after holding
it for less than 5 years the tax rate is 5%. The Land Revenue Office withholds the tax on such
net gain.

Calculation of net gain on this land:


Particular Amount Amount
Incoming Rs. 45 Lakhs
Outgoings Rs. 32 Lakhs
Purchase cost Rs. 30 Lakhs
Transfer expenses Rs. 2 Lakhs
Net gain Rs. 13 Lakhs
So, TDS shall be as follows in this case. Any exemption limit available reduces the tax
liability of the natural person. This depends on other taxable income of the tax payer and a tax
return should be submitted for this.

Naresh Singh Bhandari


bnsingh5102@gmail.com Page63
Income Tax Note 2076/77

i) The land was purchased on Chitra 2064 and sold it on Magh 2070. Mr. Komal sold the
land after holding 5 years. The TDS is Rs. 32,500.00 (2.5% of 13 Lakhs)
ii) The land was purchased on Magh 2067 and sold it on Magh 2070. He sold the land
within 5 years. The TDS is Rs. 65,000.00 (5% of 13 Lakhs
iii) The land was purchased on Chitra 2059 and sold it on Baishak 2071. Mr. Komal sold the
land after holding more than 10 years. However, the TDS is Rs. 32,500.00
iv) If selling and buying of the land were completed through a sole shareholder of a Pvt Ltd,
it will be regarded as disposal of business assets. Land Revenue Department (Malpot
Karyalaya) will deduct 10 % at the time of transfer which is not final withholding. The
TDS amount is Rs. 130,000. The income should be included in taxable income and TDS
should be claimed ad advance tax.
Example (12): Define “Non-business chargeable asset” under Income Tax Act, 2058 with
minimum 2 examples. Mention the procedures of tax imposition when a company operates
real estate business for buying and selling properties like building and land.
Answer:
Within the definition under section 2(da) of the Income Tax Act 2058, generally, this type of
assets includes properties and securities or an interest in an entity belong to an individual.
However, asset not used for generation of income is also included in this category.
Land, building, share, debenture etc. are classified in the category.
The following assets are excluded from the definition of non-business chargeable assets:
 A private residence of an individual that has been owned continuously for ten years or
more and used as residence continuously or intermittently for total of ten years or more.
 A private residence or a land of an individual that is disposed off for less than Rs. 30
Lakhs.
 Non business assets which are disposed-off by way of any type of transfer other than sale
and purchase made within three generations.
In additions to above, personnel belongings like gold and ornaments, vehicles, furniture etc.
are excluded from the definition, the income from disposal of these are also non-taxable
income.
Tax imposition on real estate business activities
Tax charging on real estate business is very highlighting issues. When an individual natural
person deals in land and building as trading, tax liability is calculated on the basis of the net
gain from the disposal of those assets. For example, Mr. Ram has purchased a piece of land at
Rs. 60 lakhs, incurred Rs. 4 lakhs as malpot while getting transfer of the land when he
purchased it. He sold the land at Rs. 80 lakhs. The net gain on the land is Rs. 16 lakhs.
Withholding tax is managed for such transactions under section 95 ka (3) of the Act for
natural person.
If the land is sold after holding it for more than 5 years, the tax is 2.5% and if it is sold after
holding it for less than 5 years the tax rate is 5%.
But when such transaction is carried out by a company, it will be taxed as normal business
income and no TDS application. However, the company will have to pay advance tax on the
income earned i.e. land revenue office shall deduct 1.5% at the time of registration.

Naresh Singh Bhandari


bnsingh5102@gmail.com Page64
Income Tax Note 2076/77

Special Provision for Natural Person, Banking and Insurance Business

1) Couple
As per section 50, provisions related to Couple are as follows;

 A resident natural person and a resident spouse of the person may, by notice in writing,
elect to be treated as a single natural person for a particular income year.
 Each spouse of a couple making an election as above with respect to an income year is
jointly and severally liable with other spouse for any tax payable by couple for the year.
 The resident widower or windows taking care of dependent are taken as couple.

2) Medical Tax Credit


As per section 51, provisions related to medical tax credit are as follows;

 A person may claim medical tax credit if all of the following conditions are satisfied;
 The person shall be a resident natural person
 The medical expenses shall be a approved medical expenditure and
 The medical expenditure shall be incurred for the treatment of the person him/herself
or through a third person.
 The limit of medical tax credit shall be lower of following;
 15% of approved medical expenditure incurred during the year + unabsorbed tax
credit carried from previous year.
 Rs. 750 or
 Actual tax liability after allowing all tax credit but before allowing medical tax credit.
 The excess amount which cannot be claimed during the year due to the above limit is
allowed to be carried forward to be claimed in subsequent income year(s) for an
unlimited period of time.
As per Rule 17, provisions related to approved medical expenditures and limits are as
follows;

 Following medical expenses are treated as Approved medical expenditure;


 Medical expenses incurred for the treatment of a natural person in an approved
hospital, nursing home, health center or by a medical practitioner on the basis of
actual invoice produced.
 Following expenditure are not qualified for approved medical expenditure;
 Expenses incurred on cosmetic surgery.
 To the extent that any amount of reimbursement received against health insurance of
person.

Naresh Singh Bhandari


bnsingh5102@gmail.com Page65
Income Tax Note 2076/77

3) Banking Business
Banking business means business carried by bank and financial institution after receiving
approval as per prevailing laws for the bank or financial institution.

Tax Rate @ 30 %

As per section 59, the provision related to the banking business is as follows:
 A person running banking business along with other business has to treat the business as
being operated by separate person and the person’s income or loss from the business for
any income year shall be calculated separately.
 A banking business is allowed a deduction of loan loss provision provided by it according
to the regulation of Nepal Rastra Bank up to maximum ceiling of 5% of the outstanding
loan and Non-banking assets on the last day of income year.
 Cooperative is allowed a deduction of loan loss provision up to maximum ceiling of 5%
of outstanding loan on the last day of income year.
 A banking business or cooperative claiming loan loss provision shall not be allowed bad
debt for the deduction for tax purpose.
 If any amount of the loan loss provision is capitalized or utilized for distribution or for
the payment of the dividend, then such amount shall be included in calculating income of
the year when it is so capitalized or distributed.
Format
Particulars Amount(Rs.)
A. Outstanding Loan before write off ***
B. 5% of A ***
C. LLP till date including written off ***
D. Claimable provision till this year(Min. of B & C) ***
E. Claim up to previous year ***
F. Eligible for this year (D-E) ***

Example (1): XYZ Bank Limited has supplied following information:

Particulars F/Y 2069/70 ‘Rs. 000’ F/Y 2070/71 ‘Rs. 000’ F/Y 2071/72 ‘Rs. 000)
Outstanding loan 12,000,000 15,000,000 14,000,000
Loan Loss Provision 500,000 800,000 850,000
Calculate the LLP allowable for deduction for each year.

Solution:

For F/Y 2069/70


Outstanding loan; 12,000,000
Loan loss provision; 500,000
Maximum Ceiling; 5 % of Rs. 12,000,000

Naresh Singh Bhandari


bnsingh5102@gmail.com Page66
Income Tax Note 2076/77

= Rs. 600,000
i.e. allowable loan loss provision this year for deduction is Rs. 500,000.

For F/Y 2070/71


Outstanding loan; 15,000,000
Loan loss provision; 800,000
Expenses charged to P/L this year; 800,000-500,000 = Rs. 300,000
Allowed expenses up to previous year; 500,000
Maximum ceiling up to this year; 5 % of 15,000,000
= Rs. 750,000
Maximum Expenses allowable for deduction this year; Max Ceiling of this year – Allowed
expense up to previous year
=750,000-500,000 = Rs. 250,000
Since maximum expenses allowable for the deduction for this year is Rs. 250,000 only even
expenses charged to P/L is Rs. 300,000 during the year.

For F/Y 2071/72


Outstanding loan; 14,000,000
Loan loss provision; 850,000
Expenses charged to P/L this year; 850,000-800,000 = Rs. 50,000
Allowed expenses up to previous year; 750,000
Maximum ceiling up to this year; 5 % of 14,000,000
= Rs. 700,000
Maximum Expenses allowable for deduction this year; Max Ceiling of this year – Allowed
expense up to previous year
=700,000-750,000 = Rs. -50,000
During the year the expenses charged to the P/L is Rs. 50,000. The maximum ceiling for the
deduction for the year is Rs. 700,000 but the bank has already deducted Rs. 750,000 up to
previous year so the excess amount Rs. 50,000 claimed shall be included in the income of the
current year.

Example (2): XYZ bank limited has furnished the following information:
Year Outstanding loan Loan loss Provision
2070/71 15,000,000 800,000
2071/72 14,000,000 850,000
Loan loss provision charged to P/L during the current year is Rs. 70,000.
Calculate the loan loss expenses allowable for deduction during F/Y 2071/72.

Solution:

Naresh Singh Bhandari


bnsingh5102@gmail.com Page67
Income Tax Note 2076/77

Here loan loss provision at the end of F/y 2070/71 is Rs. 800,000 and loan loss charged to
P/L during 2071/72 is Rs. 70,000. It means loan loss provision at the end of F/y 2071/72
should be Rs. 800,000+70,000= 870,000 but LLP given is Rs. 850,000 which indicates that
during the year loan of Rs. 870,000-850,000= Rs 20,000 is written off. Now we have to
calculate the outstanding loan before write off.

Loan outstanding before write off: 14,000,000+ 20,000 = Rs 14,020,000


5% of loan outstanding: 14,020,000*5% = Rs. 701,000
Claimed till previous year: 15,000,000*5%= 750,000
Allowable for this year: 701,000-750,000= -49,000
i.e. Rs 49,000 shall be included in the income of the current year.

4) General Insurance Business


As per section 60, provisions related to insurance business are as follows:

 A person running a general insurance business along with other business has to treat the
insurance business as carried by separate person. It means the accounts, activities,
transaction etc. related to the general insurance business should be kept separate from the
other business.
 The following income shall be included in calculating the income of the general
insurance business:
 Amount to be included under Section 7.
 Amount derived during the year as a premium or a reinsurance premium on the
risk covered by the business.
 Amount derived by it during the year under any contract of re insurance,
guarantee, security or indemnity in respect of payment to be made by it as an
insurer.
 Following expenses along with the other expenses allowed by this act are deductible
while calculating the income of general insurance business:
 Payment made during the year by the person as insurer in conducting the
business,
 Payment made as re insurance premium to the re-insurer, guarantor etc.
 Sum of the following amount mentioned in the risk bearing fund:
 50% of the net insurance premium credited to the income statement
during the year and
 115 % of the claim outstanding but not accepted by the insurer at the end
of year.
 The amount of the risk bearing fund allowed for the deduction in an income year should
be included while calculating the income from insurance business in the following year.
Naresh Singh Bhandari
bnsingh5102@gmail.com Page68
Income Tax Note 2076/77

Example (3): Nepal insurance company shows following transaction for the year 2070/71
and 2071/72.
Particulars Amount(Rs.)
Net insurance premium for 2071/72 400,000
Commission on insurance ceded for 2071/72 20,000
Unexpired risk reserve at the end of 2070/71 150,000
Claim lodged but not accepted at the end of 2070/71 23,000
Commission paid on re insurance accepted for 2071/72 10,000
Agent commission for 2071/72 15,000
Management Expenses for 2071/72 100,000
Compensation paid during 2071/72 100,000
Interest Income 50,000
Depreciation Expenses 60,000
Other income 25,000
Claim lodged but not accepted by insurer at the end of 2071/72 30,000
Calculate the taxable income.

Solution:

Particulars Amount(Rs.)
Inclusions:
Unexpired risk reserve at the end of 2070/71 150,000
Claim lodged but not accepted at the end of 2070/71 23,000
Net insurance premium for 2071/72 400,000
Commission on insurance ceded for 2071/72 20,000
Interest Income 50,000
Other income 25,000
Total Inclusions (A) 668,000
Deductions:
Compensation paid during 2071/72 100,000
Agent commission for 2071/72 15,000
Management Expenses for 2071/72 100,000
Commission paid on re insurance accepted for 2071/72 10,000
Depreciation Expenses 60,000
Claim lodged but not accepted by insurer at the end of 2071/72 34,500
(30000*115%)
Unexpired risk reserve at the end of 2071/72 (50% of 400,000) 200,000
Total deductions (A) 519,500
Income for the year (A-B) 148,500

5) Investment Insurance Business


(KaGa) Investment insurance: means insurance of any of the following classes:

Naresh Singh Bhandari


bnsingh5102@gmail.com Page69
Income Tax Note 2076/77

 Insurance where the event covered is the death of an individual who is the insured or
an associate of the insured;
 Insurance where the event covered is an individual who is the insured or an associate
of the insured sustaining personal injury or becoming incapacitated in a particular
manner;
 Insurance where the insurance agreement is expressed to be in effect for at least five
years or without limit of time and is not terminable by the insurer before the expiry of
five years except in special circumstances specified in the contract;
 Insurance under which an amount or series of amounts is to become payable to the
insured in the future; and
 Reinsurance of insurance referred to under subparagraphs (1), (2), or (3); and
 Reinsurance of reinsurance referred to under subparagraph (5)

Example (4): Excel Insurance Co. Ltd. issued a life insurance policy for Rs. 500,000 to Mr.
Murali on 1.4.2068 for 25 years at an annual premium of Rs. 1200 with a condition that if the
premium is not paid every year on due date, the policy will be cancelled.
Mr. Murali did not pay the premium for FY 2070/071 and the insurance company cancelled
the policy within 5 years of its issue. Is this investment insurance or not? CAP III, Dec 2013
Answer:
Here the policy is for more than 5 years. But it was cancelled as per the terms of the contract.
Even though it is cancelled for non-payment of premium within 5 years, it is an investment
Insurance.

As per section 61, provisions related to investment insurance are as follows:


 A person running a investment insurance business along with other business has to treat
the insurance business as carried by separate person. It means the accounts, activities,
transaction etc related to the investment insurance business should be kept separate from
the other business.
 In calculating the income from investment insurance, all amount which are required to be
included as per the other provision of the act should be included except the following;
 Amount derived in respect of insurance as premium and as a re insurance premium.
 Amount derived by it during the year under any contract of re-insurance, guarantee,
security or indemnity in respect of the payment to be made by it as an insurer.
 In calculating income from investment insurance, all amount which are required to be
deducted as per the other provision of the act should be deducted except the following;
 Payments made during the year by an insurer in the capacity of the business of
investment insurance.
 Payment made as re insurance premium to the re-insurer, guarantor etc.
Example (5):

Naresh Singh Bhandari


bnsingh5102@gmail.com Page70
Income Tax Note 2076/77

Safeguard life insurance company ltd. is an entity involved in Investment Insurance Business.
The company has following transaction for 2071/72.
Particulars Amount(Rs.)
Insurance Premium Receipts (Gross) 2,000,000
Sum Insured of Insurance policy 100,000,000
Commission paid against reinsurance taken 20,000
Commission received against re insurance done 30,000
Interest Income on Investments 3,000,000
Other miscellaneous income 60,000
Amount paid against the policy surrendered by a policy holder who has paid Rs. 5 400,000
lacs premium against it.
Amount paid to dependent of a policy holder on his death, against which premium 700,000
Rs. 70,000 was received.
Amount including bonus paid on maturity of a policy, against which a total 550,000
premium of Rs. 300,000 was received.
Commission expenses 200,000
Management Expenses 1,000,0000
Medical Fee 50,000
Allowable depreciation expenses 200,000

Solution:
Particulars Amount(Rs.)
Inclusions:
Interest Income on Investments 3,000,000
Commission received against re insurance done 30,000
Other miscellaneous income 60,000
Total Income(A) 3,090,000
Deductions:
Commission paid against reinsurance taken 20,000
Commission expenses 200,000
Management Expenses 1,000,0000
Medical Fee 50,000
Allowable depreciation expenses 200,000
Gross loss on disposal of the liability ( Working Note ) 780,000
Total Deductions(B) 2,250,000
Taxable Income (A-B) 840,000

Working Note: Calculation of Profit/loss on disposal of Liability:


Particulars Incomings Outgoings Gain/(loss)
Matured Policy 300,000 550,000 (250,000)
Death Claim Policy 70,000 700,000 (630,000)
Policy Surrendered 500,000 400,000 100,000
Net Gain/(loss) (780,000)

Naresh Singh Bhandari


bnsingh5102@gmail.com Page71
Income Tax Note 2076/77

Entity and Distribution

1) Definition
2(Bha) Entity: means the following institution or organization:
 A partnership, trust or company
 Rural Municipality, Municipality or District Coordination Committee
 Government, Concerned State Government or Local body
 A foreign government or political sub division of the foreign government, or a public
international organization established under a treaty: or
 A PE of an institution.

2(Ma) Interest in an Entity: means a right, including a contingent right to participate in the
income or capital of an entity.

2(Ra) Underlying ownership: means following ownership:


 in relation to an entity, an ownership created on basis of an interest held in the entity directly
or indirectly through one or more interposed entities by a natural person or by an entity in
which no individual has an interest; or
 in relation to an asset owned by an entity, an ownership of the asset that is determined on
basis of proportion to the ownership held by the persons having underlying ownership of the
entity.

2) Taxation Principles in respect of Entities


As per section 52, taxation principles in respect of entities are as follows:

 An entity shall be liable to tax separately from its beneficiaries.


 Distribution of the entity shall be as per section 53 and tax in respect of such distribution
shall be as per section 54.
 Amount derived and expenses incurred by an entity, whether or not derived or incurred
on behalf of another person shall be treated as derived or incurred by an entity.
 Assets owned and liabilities owed by an entity shall be treated as owned or owed by
entity and not by other person.
 Foreign income tax paid with respect to the income of an entity, whether paid by
manager, beneficiary, or the entity shall be treated as paid by the entity.
 Transaction between an entity and its managers and beneficiaries shall be recognized on
the basis of quantification of amount, re-characterization and classification of amount,

Naresh Singh Bhandari


bnsingh5102@gmail.com Page72
Income Tax Note 2076/77

transfer pricing and other arrangement between associates and transfer between
associated persons and other non-market transfer.

3) Distribution by Entities
As per section 53, provision related to distribution by entities is as follows:

 A distribution by the entity shall include the following:


 A payment made by the entity to any of its beneficiaries in any capacity, or
 Any capitalization of profits.
 Payment made by the entity to any of its beneficiaries in any capacity shall be treated as
distribution only to the following extent:
 In case the amount of payment exceeds the amount of any payment made by the
beneficiary to the entity in return for the entity’s payment.
 In case the payment does not include the following amount:
 The amount included in calculating the beneficiary’s income, or
 A final withholding payment, other than by reason of being a distribution.
 A distribution of an entity shall be treated as a distribution of profits or repayment of
capital only if it reduces the net value of the entity’s assets and liabilities.
 Distribution by the entity shall be treated as distribution of profit if it fulfills following
condition:
 If it reduces the net value of entity’s assets and liabilities.
 If at the time of distribution, the market value of the entity’s assets exceeds the
market value of the entity’s liabilities plus capital contribution including by way of
capitalization of profit.
i.e. MV of assets > MV of liabilities + Paid up Capital
 If it is a capitalization of profits.
 Repayment of capital = Distribution by entity – distribution of profit.
 Dividend = Distribution by the entity – repayment of Capital.

Example (1): Suppose A ltd. has net assets of Rs. 10 lacs and share capital is Rs. 950,000.
Hari has purchased the shares of Rs. 400,000. Hari is also the debtor of the company having
outstanding Rs. 2 lacs payable to the company. This amount has been included in calculating
the net assets of the company. The company has waived the payment of the Hari. Here the
net assets of the company have been reduced by Rs 2 lacs due to waiver so this amount is
distribution of the entity. Further, the market value of the net assets exceeds the capital by
only Rs. 50,000 so the distribution to the extent of Rs 50,000 is distribution of profit and
remaining amount Rs. 150,000 is repayment of the capital since it reduces the capital of the

Naresh Singh Bhandari


bnsingh5102@gmail.com Page73
Income Tax Note 2076/77

company. Such distribution of profits i.e. Rs. 50,000 is taxed @ 5% which is final
withholding.

Example (2): A company has share premium amount being carried forward from previous
year. The Board of Directors of the company decides to appropriate the amount of share
premium to issue right shares to existing shareholders in proportionate to their holdings
without charging any amount. The tax officer while making an amended assessment under
section 101 has assessed the appropriation as distribution of profit and thus charged tax and
penalty for not deducting TDS from the distribution of the profit. The management of the
company has a view that it is not a case of distribution of profit. Share premium is not
created out of profit but it is collected from shareholders as part of capital contribution.
As a tax professional state your arguments in favor or against the order of the tax officer.
Answer:
Section 53(3) of the act states that a distribution by an entity to its beneficiaries shall be
treated as a distribution of profit or a return of capital only if it reduces the value of the
entity’s assets and liabilities. In this case, as the shares are issued without charging any
amount, it is treated as bonus shares. But, as the shares are issued by conversion from share
premium to share capital, the value of any asset or liability of the company does not reduce.

Share premium is not created out of profit of the company but it is collected as part of
capital. When it is not created out of profit, it cannot be treated as distribution of profit. It is
not a case of capitalization of profit as the company is not providing bonus shares from the
retained earnings or general reserve of the company.
Hence the conversion from share premium to share capital is not treated as dividend and the
company is not required to deduct TDS from the amount of conversion.

(Note: It is assumed that the balance in share premium account represents the amount
paid by the shareholders who are entitled for these right shares.)

4) Taxation of Dividend
As per section 54, provisions related to taxation of dividend are as follows:

 In case of dividend distributed by resident entity:


 Distributed by a company to its shareholder and partnership firm shall be taxed in the
form of a final withholding tax and
 Distributed by other entities shall be exempt from tax.
 In case of dividend distributed by non-resident entity to a resident beneficiary: it shall be
included in calculating the income of the beneficiary and taxed accordingly.

Naresh Singh Bhandari


bnsingh5102@gmail.com Page74
Income Tax Note 2076/77

 Distribution of the dividend from the amount which is derived from the resident entity
after withholding tax shall be exempt from tax.

Example (3): Suppose Bhandari Pvt. Ltd. has received dividend Rs. 30 lacs net of tax from
his investment in A ltd. The distributable income of the Bhandari Pvt. Ltd. is Rs. 70 lacs
before inclusion of dividend received. After dividend received the distributable income of the
company is Rs 1 crore. The company has decided to distribute all the income to its
shareholder. Suppose Mahesh Bhandari is shareholder of the Bhandari Pvt. Ltd. having 30%
shareholding. Calculate the net amount to be received by the Mahesh after withholding tax.
Solution:
Total dividend distribution: Rs 10,000,000
Dividend distributed from dividend income: 3,000,000
Portion of dividend income in distribution: 30%
Gross dividend to be received by Mahesh: 10,000,000*30% = 3,000,000
Portion of Tax exempted dividend: 3,000,000*30% = 900,000
Portion of taxable dividend: 3,000,000-900,000 = 2,100,000
Tax on dividend @ 5 %: 105,000
Net payment to Mahesh: 2,895,000

Example (4): A Ltd. receives dividend of Rs. 400,000 and interest (net) of Rs. 200,000. A
Ltd. distributes Rs. 450,000 as dividend. How much tax A Ltd. should deduct from the
dividend distributed by it?
Answer:
As per Income Tax Act, if the dividend is declared from dividend income which is final tax
withholding income, there is no need to pay again dividend tax from such income.
Unless a Company specifies, from which source how much dividend is paid, it will be
assessed that dividend is paid proportionally that the total such available. The total such is
400,000+200,000=600,000 out of which Rs. 200,000 is 1/3 of the total remaining. So, the
dividend paid shall be deemed to have been paid Rs. 150,000 out of interest income and on
that 5% viz. Rs. 7,500 have to be deducted at source.
Note: There may be alternative solution. Since the question said Interest income is net of Tax
that is gross Interest Income is 235,294. Therefore, proportion is 400,000+235,294= 635,294
out of which Rs. 235,294 is 37.04% of total.
Therefore, dividend paid shall be deemed to have been paid Rs. 166,680 out of
Interest Income and 5% of such amount is subject to Tax i.e. 8,334.

Example (5): Mr. Ram, a resident person, received dividend of Rs. 50,000 from BalGopal
Limited. BalGopal Limited distributed dividend of Rs. 600,000 out of Rs. 950,000 received
as dividend from A One Ltd., who declared a dividend of Rs. 2,500,000 and distributed the
dividend after deducting the tax at source.
Calculate the amount of tax to be deducted by Bal Gopal Ltd. on the dividend amount to be
paid to Mr. Ram.

Naresh Singh Bhandari


bnsingh5102@gmail.com Page75
Income Tax Note 2076/77

Will your answer differ if Bal Gopal Ltd. is non- resident entity? What will be your answer if
Bal Gopal Ltd. is a cooperative, incorporated under Co-operative Society Act, 2048, industry
based on agro-products or forest products?
Provide your answer by mentioning the relevant provisions of the act.
CAP III July 2015 (5 Marks)
Answer:
As per section 54(3) of the act, a company that receives an amount of dividend after
deduction of tax, is not obliged to deduct tax on dividend paid by it to its shareholders out of
the amount of dividend received. In this case, Mr. Ram received the dividend from Bal Gopal
Ltd. and Bal Gopal Ltd. has distributed dividend out of the dividend received from AOne
Ltd. The dividend from AOne Ltd. is received after deduction of the tax. So Bal Gopal Ltd. is
not obliged to deduct tax on payment of dividend to Mr. Ram.

Yes, the answer will differ. As per section 54(2) of the act, if a non-resident entity pays
dividend to a resident person, the amount of the dividend is to be included in income from
investment of the payee.
As per section 11(2) of the act, if dividend is distributed by a cooperative registered under the
cooperative society act, 2048 and running industry based on agro- products or forest
products, then it is exempt from tax.
Example (6): The paid up share capital of ABC Ltd. consists of 150,000 shares of Rs. 100
each. As of Ashadh end 2068, the P&L Appropriation Accounts of the company is abstracted
as below:

P&L Appropriation Account:


Profit for the year Rs. 1,000,000
Accumulated profit b/d Rs. 1,500,000
Total distributable profit Rs. 2,500,000
The Board of Directors (BOD) of the company proposed to issue bonus share at 10%. Also, it
is proposed that dividend tax on the capitalized profit (sec 88(1)) is to be borne by company
by distributing cash dividend so that shareholders do not have to pay any tax at source as
required under sec. 88(1).
You are required to compute the amount of such cash dividend.
Answer:
Total number of bonus share to be issued @ 10% = 15,000 shares
Total amount of bonus shares = 15,000 share @ Rs. 100 each
= Rs. 1,500,000
Dividend Tax @ 5% = 5% of Rs. 1,500,000
= Rs. 75,000
In case withholding tax on such bonus shares is to be paid by ABC Ltd. itself by proposing to
declare cash dividend, such cash dividend (i.e. tax on bonus shares) is deemed as further
distribution of dividend subject to dividend tax @ 5%.
Therefore, total cash dividend to be proposed = Rs. 75,000 + (5/95 *75,000)
= Rs. 75,000 + Rs. 3,947.37
= Rs. 78,947
Note: Alternatively, it can be calculated as follows:
Naresh Singh Bhandari
bnsingh5102@gmail.com Page76
Income Tax Note 2076/77

Cash dividend = (1,500,000/95) *5 = Rs. 78,947

Example (7): X & Co. has received from Y & Co., a resident company, Rs. 10 lakhs as
dividend after deduction of taxes. The profit of X & Co. before inclusion of such dividend
income is Rs. 90 lakhs. X & Co. has decided to declare the dividend distribution to its
shareholders during current fiscal year amounting to Rs. 100 lakhs (including Rs. 10 lakhs of
dividend received from Y & Co.). Mr. Bibhu is one of the shareholders of X & Co. and
supposed to receive from Rs. 5 lakhs as dividend. Mention the relevant provision and
calculate the amount of Tax to be deducted while making payment to Mr. Bibhu.
CAP II,Dec 2013 (5 Marks)
Answer:
As per section 54(1) of Income Tax Act, 2058, dividends distributed by a resident company
to its shareholders shall be taxed in the form of final withholding tax.
Section 54(3) further envisages that notwithstanding anything contained in section 54(1),
distribution of dividend that is derived after withholding tax as per subsection (1) shall be
exempt from tax.

Computation of Tax to be deducted while making payment of dividend to Mr. Bibhu


Particulars Amount (Rs)
Total amount of dividend distribution 100 Lakhs
Dividend income after deduction of tax 10 Lakhs
Proportion of dividend income (Rs. 10 lakhs/Rs. 100 lakhs) 10 %
Payment to Mr. Bibhu 500,000
Amount not subject to tax (10%) 50,000
Amount subject to tax (90%) 450,000
Tax amount at the rate of 5% 22,500

5) Liquidation of an Entity
As per section 55, the provision regarding to liquidation of an entity are as follows:

 In Case where all the following conditions are satisfied, the distribution shall be treated
as partly a dividend and partly as a repayment of capital of the entity in the proportion
that the beneficiary would be entitled to share in the profits and contributed capital of the
entity if the interest were disposed of in the course of liquidating the entity at that time;
 If the distribution by an entity is in respect of the cancellation, redemption, or
surrender of an interest in the entity, or, where allowed by law, as a result of the entity
purchasing an interest in itself;
 The distribution is not and may not be reasonably calculated, in any respect, as
approximately referable to the rights of beneficiaries to share in the profit of the entity
other than on a complete liquidation and
 After the disposal the beneficiaries to whom the distribution is made is not an
associate of the entity.

Naresh Singh Bhandari


bnsingh5102@gmail.com Page77
Income Tax Note 2076/77

Example (8) (Directive): Suppose Lalitpur Company Ltd. has paid up capital Rs. 6 crores.
The company has been completely liquidated. At the time of liquidation, the company has
received Rs. 7 crores from diposal of assets. In such condition of liquidation, distribution of
capital portion and profit portion shall be done on the basis of profit first concept and of the
total distribution, first Rs. 1 crore shall be considered as profit distribution and Rs. 6 crores
shall be considered as repayment of capital.
But if only 3 crores is distributed than payment shall be treated as partly distribution of profit
and partly repayment of capital; on the basis of capital-first concept. In this case, if the value
of the assets before the distribution is Rs. 7 crores than distribution of Rs. 3 crores shall be
treated as follows:
Capital Refund: 600/700* 300 = Rs. 257.143 lacs
Profit distribution: 300-257.143 = Rs. 42.857 lacs
Tax shall be deducted @ 5% on 42.857 lacs at the time of payment.
In case if the MV of the assets is less than the paid up capital, then the total amount shall
be treated as repayment of the capital.

 Provision relating to distribution (Sec 53) by the entities and liquidation of an entity (55)
do not apply to a distribution made by an entity to one of its beneficiaries in the course of
purchasing an interest held by the beneficiary in the entity in the ordinary course of
business on a recognized stock exchange.

Example (9) (Directive): Suppose Danfe Company Ltd. has purchased its 1000 share of Rs.
100 each from one of its shareholders @ Rs. 500 each through Nepal Stock Exchange
Market. This is considered as Treasury Stock investment. In such case the amount Rs.
500,000 received by the shareholder from the company shall not be treated as distribution of
profit and repayment of capital. It shall be treated like investment in other company. Since
section 53 does not attract, capital gain tax shall be applied.

6) Dealing between an Entity and a Beneficiary


As per section 56, provision regarding dealing between an entity and a beneficiary are as
follows:

 Subject to section 45, where an asset is disposed of by way of transfer of ownership of


the asset by an entity to one of its beneficiaries, including by way of distribution, or vice
versa-
 the transferor shall be treated as deriving in respect of the disposal an amount equal to
the market value of the asset immediately before the disposal; and

Naresh Singh Bhandari


bnsingh5102@gmail.com Page78
Income Tax Note 2076/77

 the transferee shall be treated as incurring in the acquisition of the asset costs of an
amount equal to the market value of the asset immediately before the disposal.
 Subject to section 45, where a liability is disposed of by way of transfer of the liability
by an entity to one of its beneficiaries or vice versa-
 the transferor shall be treated as incurring costs for the disposal in an amount equal to
the market value of the liability immediately before the disposal; and
 the transferee is treated as deriving in respect of incurring the liability an amount
equal to the market value of the liability immediately before the disposal.
Example (10): Suppose a company dealing in car trading business has sold a car of value Rs.
45 lacs to one of its shareholder at Rs. 30 lacs. In such case Rs 45 lacs shall be considered as
income of the company and Rs. 15 lacs (45-30) shall be treated as distribution of profit.

 Where an entity distributes a dividend to a beneficiary that is not a distribution of profits,


the amount of the dividend shall be included in calculating the income of the entity.
Provided that, the provision made under this subsection may be made as not to apply in
any prescribed circumstances.

Example (11): Suppose the tax base shown by the balance sheet of the Bhandari Ltd. is as
follows:
Capital & Book Value Market Assets Book Market
liabilities (Rs.) Value (Rs.) Value(Rs.) Value (Rs.)
Capital 100,000 - Trading Stock 100,000 115,000
Profit 100,000 - Depreciable asset 100,000 95,000
Liabilities 100,000 100,000 Business Assets 100,000 110,000
Total 300,000 100,000 Total 300,000 320,000

In the above case, the maximum amount available for the distribution is as follows:

Maximum Distribution = MV of Assets- MV of liabilities- Paid up capital


= 320,000-100,000-100,000
= Rs. 120,000
Suppose the company has paid Rs. 115,000 to any of its beneficiary than in such case the
payment shall be considered as distribution of profit since it is within Rs. 120,000 but as per
tax base balance sheet the profit available for distribution is Rs. 100,000 only so the excess
distribution Rs. 15,000 shall be included in calculating income from business of the
company.

7) Change in control
As per section 57, provision regarding the change in control is as follows:

Naresh Singh Bhandari


bnsingh5102@gmail.com Page79
Income Tax Note 2076/77

 If the underlying ownership of an entity as compared with its ownership 3 years


previously changes by 50% or more, then the entity shall be treated as disposing of any
assets owned by it and any liabilities owed by it.
 For the purpose of calculating the change of 50% or more in the ownership of the entity,
the following ownership shall be considered:
 The ownership occupied by the shareholder holding 1% or more of the total
ownership; and
 The ownership occupied by any of the shareholders holding less than 1% of the total
ownership but associated with the shareholder holding 1% or more ownership.
 If there is change in control, after the change the entity is not permitted to;
 Deduct the interest carried forward (Sec 14(2)) that was incurred by an entity prior to
change,
 Deduct a loss from business or investment (Sec 20) that was incurred by the entity
prior to the change,
 Carry back a loss from long term contract (Sec 20(4)) that is incurred after the change
to an income year occurring before the change.
 Adjustment regarding foreign exchange gain/loss (Sec 24).
 Adjustment regarding reversal of the amount including bad debt (Sec 25).
 Reduce gain from disposal of assets or liabilities (Sec 36) after the change by the
losses incurred on the disposal of assets or liabilities before the change.
 In case where the entity accounted for an insurance premium in terms of section 60(2)
(kha)(1) prior to the change and the entity after that change returns the premium to
the insured, claim a deduction under that provision: or
 Carry forward foreign tax (Sec 71(3)) that is paid with respect to foreign income prior
to change.
 Where there is change in ownership by 50% or more during the income year of an entity,
the parts of the income year before and after the change in ownership are treated as
separate income years.

Example (12): More than fifty percent of the ownership of M/s Neptune International Pvt.
Ltd. is transferred in comparison to the ownership three years previously. The date of such
transfer of ownership was 2068.11.10. The company has not sought for extension of time
limit for filing the return of income. The company seeks your advice on the due date of filing
of tax returns in the referred case. From Income Tax Department’s point of view, what would
be the due date for Amended Assessment in such case?
CAP III, June 2013 (2.5 Marks)
Answer:
An entity is deemed to be disposed off when 50% or more of its ownership in comparison to
the ownership three years previously is transferred. In such a case as per section 57(3) of the
act, period before the date of such ownership and period after such ownership are treated as

Naresh Singh Bhandari


bnsingh5102@gmail.com Page80
Income Tax Note 2076/77

separate income years. Further, as per section 96(1) of the act, tax return should be filed
within three months from the expiry of the income year as the company has not sought for
extension of time. Similarly, section 101 of the act fixes the time limit for amended
assessment as four years from the due dates of filing of return.
In the given case, date of transfer of ownership is 2068.11.10. Hence, periods before and
after this date is segregated into two income years. From 2068.04.01 to 2068.11.10 is one
period whose due date will be 2069.02.10. From 2068.11.11 to 2069 Ashadh end is another
income year whose due date will be 2069 Ashoj end. For income tax department, time limit
for amended assessment in the first case will be 2073.02.10 and for the latter 2073 Ashoj
end.
Note for evaluator:
As per sec. 2(jhha), an income year starts from 1st of Shrawan and ends at Ashad end of next
year. Thus, an income year could not be defined otherwise. The return filing date in both the
conditions shall be Ashoj 31, 2069.

Example (13): Mr. Shyam, a sole shareholder of M/s Ganpati Industries Pvt. Ltd., was
worried about the performance of the Company as it incurred losses to the tune of Rs. 5
crores during last 4 financial years ending the F/Y 2068/69. Mr. Mohan, an expert acquired
60% stake in the Company on Ashadh 31, 2069. Miracally, the company has managed to
earn Rs. 1.5 crore as profit in F/Y 2069/70. The Company has submitted the Income Tax
Return by assessing a taxable loss of Rs. 3.5 crore for F/Y 2069/70 under self-assessment by
adjusting the carry forward losses of Rs. 5 crores up to F/Y 2068/69 u/s 20 of Income Tax
Act, 2058. The Chief Tax Officer issued an order to pay income tax on Rs. 1.5 crore along
with interest thereon. The management of the Company seeks your advice on the said order
of Inland Revenue Office. CAP II, June 2014 (4 Marks)
Answer:
As per section 57 (1) & (2) of Income Tax Act, 2058, if the ownership of any entity changes
by 50% or more during the last three financial years, the Company is not allowed to carry
forward its accumulated losses of the period prior to such transfer of ownership. In this case,
the ownership of M/s Ganpati Industries Pvt. Ltd. was change by 60% as the shares of the
Company was sold by old management to the new management, therefore, the Company
cannot adjust any accumulated losses for the period untillAshad 31, 2069. Thus, the
assessment order issued by the Inland Revenue Office is correct & the Company has to pay
tax on the profit of the Company earned during the F/Y 2069/70.

8) Dividend Stripping

As per section 58, provisions related to dividend stripping are as follows:


 If the dividend is distributed by an entity in the following manner, it shall be treated as
distributed in the course of dividend stripping arrangement.
 If the entity has accumulated current or expected profits.
 a person, the acquirer acquires an interest in the entity and the acquirer or an associate
of the acquirer makes a payment, whether or not in respect of the acquisition and
Naresh Singh Bhandari
bnsingh5102@gmail.com Page81
Income Tax Note 2076/77

whether or not the payment is at the time of acquisition, to another person who is or
was a beneficiary in the entity or an associate of such another person;
 the payment is reflected, in whole or in part, in the profits of the entity; and
 After the acquirer acquires the interest in the entity, the entity distributes a dividend
to the acquirer that represents, in whole or in part, the profits.
 In the case where the dividend is distributed under the dividend stripping arrangement by
an entity, the arrangement shall be treated as the following: -
a) The payment is not made by the acquirer or an associate of the acquirer but is a
dividend distributed as referred to in paragraph (b) by the entity to the original or
current beneficiary; and
b) The dividend distributed by the entity to the acquirer is in an amount equal to the
dividend referred to in paragraph (a) less the amount of the payment.
Example: See page no. 212 of Directives.

Naresh Singh Bhandari


bnsingh5102@gmail.com Page82
Income Tax Note 2076/77

Retirement Fund

1) Definition
2(Gha): Retirement Fund: means an entity established solely for the purpose of accepting
and investing retirement contribution in order to provide retirement payment from fund to a
beneficiary natural person or his/her dependent.

2(Nga): Retirement Payment: means a payment to the following person:


 Payment to a natural person in case of his/her retirement
 Payment to a dependent in case of the natural person’s death.

2(cha): Retirement Contribution: means a payment made to a retirement fund for the
provision or future provision of retirement payment.

2) Approval for Retirement Fund


As per section 63, provisions for approval of retirement fund are as follows:

 Application shall be made to the department for getting the approval.


 No approval shall be required to operate Citizen Investment Trust established under
Citizen Investment Trust Act 2074, Employee Provident Fund established under
Employee Provident Fund Act 2019, Retirement Fund operated by Social Security Fund
established under Contribution Based Social Security Fund Act 2074 and Pension Fund
established under Pension Fund Act 2075.
 A natural person making contribution to ARF may claim to have their taxable income
reduced by amount lower of the following (Rule 21):
 Actual Contribution
 Maximum Rs 300,000 or
 1/3rd of Assessable Income, w/h is lower.
On receipt of application for the ARF, the department may give approval on the basis of
the Rule 20(2). This rule prescribes the condition to be satisfied for getting approval of
ARF which are as follows:
 Amount deposited to fund shall only be invested in approved investment.
 Paid up Capital should be at least Rs 1 crore.
 Number of beneficiary, worker or employees should be at least 1,000.
 If the fund accepts retirement contribution from an employer on behalf of the employees
or workers, the fund is managed independently of the employer. However, this does not
apply in case of employees or workers of the fund itself.

Naresh Singh Bhandari


bnsingh5102@gmail.com Page83
Income Tax Note 2076/77

 Retirement contribution shall be deposited within 1 month of the contribution made if the
contribution is made during Ashad and in any other case, within 15 days of the
contribution made.
 Retirement Payment shall be made to the beneficiary only:
 Upon the retirement from service.
 Upon the beneficiary reaching the age of 58 years.
 Upon the death or permanent disability of the beneficiary.
 The fund shall be audited annually by an auditor registered with the auditor general.

If the fund no longer complies with the above conditions, the department may withdraw an
approval granted to the retirement fund.

3) Retirement Payment
Retirement payment in the form of gratuity, accumulated leave, provident fund or similar
retirement scheme accumulated till 2058.12.18 is tax exempted. In case of medical cost not
utilized on that date; payment of retirement medical cost up to Rs. 180,000 is tax exempted.

As per section 65, the provisions related to retirement payment are as follows:

 Taxable income of a natural person in respect of retirement payment as a result of


contributory in nature in any ARF or made by GON shall be as follows:
 Retirement payment made by the fund shall be included in income of the person.
 If Retirement payment are made in lump sum it shall be treated as follows:
Retirement Payment accumulated ****
Less: Retirement Payment accumulated till 2058.12.18 ****
Retirement Payment accumulated after 2058.12.18 ****
Less: Higher 50% of Actual or NPR 500,000 ****
Remaining Amount ****
Tax @ 5% ****
Such tax amount is final withholding tax.

Example (1): Suppose Mr. A is working in the Nepal Bank Limited. He is retired from the
service on 2072.03.31. During his service, the amount deducted as retirement contribution is
deposited to the employee provident fund. From the record maintained it is seen that total
contribution made by him is Rs. 2,000,000 out of which Rs 900,000 is made till 2058.12.18.
Calculated tax to be deducted at the time of payment.

Solution:
Retirement Payment accumulated 2,000,000
Less: Retirement Payment accumulated till 2058.12.18 900,000
Naresh Singh Bhandari
bnsingh5102@gmail.com Page84
Income Tax Note 2076/77

Retirement Payment accumulated after 2058.12.18 1,100,000


Less: Higher of the following:
 50% of Actual i.e Rs 550,000
 Rs 500,000 550,000
Remaining Amount 550,000
Tax @ 5% 27,500
Such tax amount is final withholding tax.

 In case of calculating gain of a natural person from an interest in an unapproved


retirement fund:
 If the proceeds are paid by the resident person than gain shall be taxed in the hands of
the natural person beneficiary in the form of the final withholding tax; and
 If the proceeds are paid by the non-resident person than gain shall be included in
calculating income of the natural person beneficiary.
Note:
a) Gain means the difference between the contribution made by the employee to URF
and amount paid by the URF to the beneficiary as a retirement payment.
b) Gain accrued till 2058.12.18 is also tax exempted income.
c) Payment for gratuity, leave encashment and voluntary retirement compensation shall
be taxed @ 15 % which is final withholding.

Example (2): Suppose Ms. A is working in Bhandari Pvt. Ltd. She is retired from the service
on 2072.03.31. During her service, the amount deducted as retirement contribution is
deposited to the unapproved retirement fund. From the record maintained it is seen that
retirement contribution made by her is Rs. 500,000 till 2058.12.18 and after that Rs.
1,100,000. The interest accrued till 2058.12.18 is Rs 100,000. At the time of retirement, she
gets Rs. 2,000,000 as retirement payment. Calculated gain for the tax purpose and tax
liability.

Solution:
Retirement Payment accumulated 2,000,000
Less: Retirement Contribution accumulated 1,600,000
Less: Gain accrued till 2058.12.18 100,000
Gain accrued after 2058.12.18 300,000
Tax @ 5% 15,000

Such tax is final withholding tax if paid by resident person. If paid by non-resident person,
gain Rs. 300,000 shall be included in the income of the beneficiary while calculating her
taxable income.
Example (3): Mr. Ramesh has been retired from Government of Nepal on 15th Jestha, 2071.
He has received the following retirement payments in Ashadh, 2071. He has not any other

Naresh Singh Bhandari


bnsingh5102@gmail.com Page85
Income Tax Note 2076/77

sources of income except salary. Remuneration tax already has been deducted and deposited.
Assume, no retirement payment was accrued at the commencement of this act in connection
with this employment.
Particulars Amount (Rs.)
Payment from GON against accumulated leave and medical allowances 600,000.00
Payment from Employee Provident Fund against contribution 700,000.00
Payment from Citizen Investment Trust against contribution 500,000.00
Calculate the withholding tax to be deducted at the time of retirement payment.

Answer:
Fifty percent of the paid amount or Rs. 5 Lakhs, whichever is higher, shall be deducted on
the lump sum retirement payment from any Approved Retirement Fund or Nepal government
in computing tax liability of any natural person as per section 65(1).
Further, section 88 (1) (1) states that in the case of the retirement payment made by the
Government of Nepal or by the approved retirement fund, at the Rate of Five percent shall be
deducted from the benefits calculated pursuant to section 65(1). Such retirement payment is
final withholding as per section 92 (1) (Chha).
Retirement payment to Mr. Ramesh from retirement fund and Nepal government and 50
percent thereof are as follows:
Particulars Amount (Rs)
Payment from GON against accumulated leave and medical allowances 600,000
Payment from Employee Provident Fund against contribution 700,000
Payment from Citizen Investment Trust against contribution 500,000
Total amount (WN 1) 1,800,000
50% of Total Payment (a) 900,000
Fixed amount u/s 65 (b) 500,000
Deduction allowed for the higher amount (a) or (b) above 900,000
Taxable Amount 900,000
The EPF will withhold Rs. 10,000 @ 5% on Rs. 200,000 (Rs. 700,000- Rs. 500,000) and
similarly, the CIT will not withhold any amount being only Rs.5,00,000.00. But, the GON
will withhold Rs. 35,000 (i.e. Rs. 45,000, @ 5% on 900,000, minus the amount Rs. 10,000
already withheld by the EPF). It will be a final withholding u/s 92.

4) Taxation of Retirement Fund


As per section 64, provision related to taxation of the retirement fund is as follows:

 While calculating the income of the retirement fund, all the inclusions and deductions
will be the mount required to be included and deducted under the other provisions of this
act but;

Naresh Singh Bhandari


bnsingh5102@gmail.com Page86
Income Tax Note 2076/77

 Retirement contribution received shall not be included in income.


 Retirement payment shall not deduct6ed while calculating income.
 Interest of the beneficiary in the retirement fund shall not be a liability of the fund.
 Income of the approved fund shall be exempted from the tax.
 In case, if the approved retirement fund ceases to be an approved fund, then it has to pay
tax @ 25% on the amount calculated as follows:
Inclusions:
 Sum of contributions received from beneficiaries and
 Sum of other income i.e. interest, commission etc from the date of approval to date of
cease.
Deductions:
 Retirement payment made by the fund from the date of approval to the date of ceases.

Example (4): Suppose Bhandari Pvt. Ltd. is operating retirement fund for its employees after
obtaining approval from Department on 2070 Ashad end. The account of retirement fund
shows following:
Retirement contribution for the F/Y 2070/71 3,000,000
Interest received in investment during F/Y 2070/71 250,000
Retirement contribution for the F/Y 2071/72 4,000,000
Interest received in investment during F/Y 2071/72 600,000
Retirement payment during F/Y 2070/71 1,400,000
Retirement payment during F/Y 2071/72 1,600,000
Operating Expenses for F/Y 2070/71 150,000
Operating Expenses for F/Y 2071/72 200,000
The department has withdrawn approval on 2072 Jestha end due to its non-compliance with
the prescribed condition.
Calculate the tax liability of the fund at the time of cease from approved retirement fund.

Solution:
Retirement contribution for the F/Y 2070/71 3,000,000
Interest received in investment during F/Y 2070/71 250,000
Retirement contribution for the F/Y 2071/72 4,000,000
Interest received in investment during F/Y 2071/72 600,000
Total inclusion (A) 7,850,000
Retirement payment during F/Y 2070/71 1,400,000
Retirement payment during F/Y 2071/72 1,600,000
Total Deductions (B) 3,000,000
Taxable Income (A-B) 4,850,000
Tax @ 25% 1,212,500

Naresh Singh Bhandari


bnsingh5102@gmail.com Page87
Income Tax Note 2076/77

International Taxation

1) Definition
2(Sha) Natural Resource Payment: means amount of any of the following payment;
 Payment for getting the right to take water, minerals, or other living or non-living
resource from land; or
 Amount calculated on the basis of the quantity or value of a living or non-living natural
resource and minerals taken in whole or in part from the land.

2(Ha) Payment: means;


 The transfer by one person of money or an asset to another person or the transfer by
another person of a liability to the one person;
 The creation by one person of an asset that on creation is owned by another person or the
decrease by one person of a liability owned by another person.
 The provision by one person of services to another person; and
 The use or availability for use, of an asset owned by one person to another person.

2(KaDda) Repatriated Income: means an amount of income to be repatriated to a foreign


country through a bank or paid through a bank or paid through anyway by a foreign
permanent establishment of a non-resident person referred to in section 68.

2(KaDhha) Foreign Income tax: means income tax levied by a foreign country as
mentioned in section 69(8) and includes a final withholding tax levied by a foreign country.

2(KaNna) Foreign Permanent Establishment: means permanent establishment of a non-


resident person.

2(KaDa) Permanent Establishment: means a place where a person carries on business


whether fully or partially, and includes;
 A place where a person is carrying on business partially or fully through an agent, other
than a general agent of independent status acting in the ordinary course of business as
such;
 A place where a person has, is using, or is installing substantial equipment or substantial
machinery;
 One or more places of a country where a person provides technical, management or
consultancy services through employee or otherwise for 90 days or more in any 12
months’ period;

Naresh Singh Bhandari


bnsingh5102@gmail.com Page88
Income Tax Note 2076/77

 A place where a person is engaged in a construction, assembly, or installation project for


90 days or more, including a place where a person is conducting supervisory activities in
relation to such a project.

2) Source of Income, Loss, Gain and Payment


As per section 67, provisions related to source of Income, Loss, Gain and Payment are as
follows;

 A person’s following income from any employment, business or investment shall be


treated as having a source in Nepal;
Amount included in calculating income having source in Nepal ×××
Less: Amount deducted in calculating income having source in Nepal ×××
Person’s income having source in Nepal ×××
 A person’s following loss from any business or investment shall be treated as having
source in Nepal;
Amounts deducted in calculating income having source in Nepal ×××
Less: Amount included in calculating that income having source in Nepal ×××
Loss from business or investment having source in Nepal ×××
 Following amount included in calculating income shall be treated as having source in
Nepal;
 Net gain from disposal of business asset or liabilities of business and non-business
chargeable assets of investment to the extent that net gain consists of gains from
disposal of assets or liabilities with a source in Nepal as reduced by losses from
disposal of assets or liabilities with a source in Nepal.
 Net gain from disposal of depreciable assets of the business or investment of the
person.
 Except above, payment that have a source in Nepal.
 A gain or loss from the disposal of an asset or liability shall be treated as having source in
where a domestic asset or domestic liability in Nepal.
 Following amount deducted in calculating income shall be treated as having source in
Nepal;
 Cost of trading stock relating to assets situated in Nepal,
 Repair and Improvement expenditure and depreciation allowance incurred with
respect to or relate to assets situated in Nepal,
 Except above, payment that have source in Nepal.
 The following payment shall be treated as having source in Nepal;
 Dividends paid by resident entity,
 Interest paid by a resident person,

Naresh Singh Bhandari


bnsingh5102@gmail.com Page89
Income Tax Note 2076/77

 Natural resource payments made in respect of or calculated by reference to natural


resources taken from land situated in Nepal,
 Rent paid for the use of the asset situated in Nepal,
 Royalties arising from the use of, right to use, or forbearance from using an asset
situated in Nepal,
 Premium for general insurance paid to and proceeds from general insurance paid by a
person in respect of insurance of any risk in Nepal,
 Payment received in Nepal, other than as a result of transshipment, by a person who
conducts a business of land, sea or air transport operator or charterer from;
- The carriage of passenger who embark; or
- Mail, livestock, or other moveable tangible assets that are embarked
 Payment received by a person who conducts a business of transmitting message by
cable, radio, optical fiber or satellite communication in respect of transmission of
messages by apparatus established in Nepal, whether or not such message originate in
Nepal,
 Payments including service fee for or attributable to employment exercised, service
rendered, or forbearance from exercising employment or service -
- In Nepal, regardless of the place of payment; or
- Where the payer is GON, irrespective of place of employment.
 Amount of annuities, proceeds of investment insurance and retirement payment paid
by a resident person and any premium or other payment to a resident person to secure
such amount,
 Gift received in respect of business or investment conducted with domestic assets,
and
 The following payment other than above;
- In respect of the disposal of a domestic asset or the incurring of a domestic
liability, or
- Made in respect of activity conducted in Nepal
 Payment except above shall be treated as having a foreign source and for the purpose of
determining in which country the income, loss, amount or payment is sourced, all the
above provision shall apply as though references in this act to Nepal were a reference to a
particular foreign country.
 Domestic asset means land or buildings situated in Nepal as well as an asset of a resident
person other than land or a building situated in foreign country or an interest in an entity
that is a controlled foreign entity within the meaning of section 69 where the person is an
associate of the entity. Domestic liability means a liability of a resident person.

Example (1): State whether the source of following payment/ income is considered to be in
Nepal or not. If yes, calculate the amount of Tax.

Naresh Singh Bhandari


bnsingh5102@gmail.com Page90
Income Tax Note 2076/77

i) Mr. Walls of Canada invested Rs. 500,000 from his income from Canada in a company
incorporated in Nepal and got dividend of Rs. 200,000.
ii) XYZ cable TV pays Royalty of Rs. 10 lakhs to Crick Africa Channel (a channel
incorporated in South Africa) for buying the copy rights to telecast, in Nepal, the match
between India and South Africa, which is going to be held in South Africa.
CAP III, Dec 2014 (5 Marks)
Answer:
i) As per section 67(6) (ka) ; dividend paid by a resident entity is treated to be having a
source in Nepal & thus Rs. 2 lacs paid as dividend is taxable in Nepal.
Tax on dividend Rs. 200,000 @ 5%
= Rs. 10,000
ii) As per sec 67(6) (ja) payments made for transmitting messages by apparatus established
in Nepal, whether or not such messages are originated in Nepal are treated having source
in Nepal. Hence, royalty paid by XYZ cable TV is considered to be having source in
Nepal.
Calculation of Tax
Royalty paid = Rs. 10 lacs
TDS on royalty as per Sec 88 (1) @ 15% = Rs. 150,000

3) Foreign Permanent establishment


As per section 68, provisions related to Foreign Permanent establishment are as follows:

 A foreign permanent establishment of a non-resident person situated in Nepal shall be


liable to tax with respect to its income.
 The income of foreign permanent establishment shall be allocated to its owner in
accordance with section 69.
 The repatriated income of a foreign permanent establishment of a non-resident person
situated in Nepal shall be taxed in the hands of permanent establishment in accordance
with section 3 (Kha).
 The amount repatriated by a Foreign Permanent Establishment of a non-resident person
shall be equal to the amount of dividend distributed by FPE.
Note:
i) The tax rate applicable for FPE is normal as other entity.
ii) As per section 2(6) of schedule 1, tax will be charged at 5 % of the income remitted
during the year to a foreign country by a foreign permanent establishment.

Example (2): AMEXO Bank is registered in the USA and operating its liaison office in
Kathmandu. During the financial year 2067/68, it has following summarized transactions.
(Rs. In ‘000)
Income recognized 5,000
Expenses 4,000 except income tax
Naresh Singh Bhandari
bnsingh5102@gmail.com Page91
Income Tax Note 2076/77

The liaison office has policy to repatriate all the remaining profits to its corporate office.
Compute maximum amount that can be repatriated from Kathmandu liaison office.
Answer:
As per the provision of Income Tax Act, AMEXO is taxed at two stages i.e.
 Resident bank having taxable income @ 30% of taxable amount.
 5% on repatriated amount.
Now,
Taxable amount = Rs. 5000 thousand – Rs. 4000 thousand
= Rs. 1000 thousand
Tax @ 30% = Rs. 1000 thousand * 30%
= Rs. 300 thousand
Remaining profit = Rs. 1000 thousand - Rs. 300 thousand
= Rs. 700 thousand

Tax of repatriated amount = Rs. 700 thousand * 5/105


= Rs. 33.33 thousand

Therefore, amount that can be repatriated = Rs. 700 thousand – Rs. 33.33 thousand
= Rs. 666.66 thousand

4) Controlled Foreign Entities


As per section 68, provisions related to controlled foreign entities are as follows;

 Where at the end of an income year an entity distributes dividend being a controlled
foreign entity out of the attributable income derived during the year, the entity shall be
treated as proportionately distributing a dividend as follows to its beneficiaries;
 In accordance with the beneficiaries’ right to that income upon distribution, or
 Where those rights are not reasonably certain, in such manner as Department thinks
appropriate in the circumstances.
 Dividend distributed during an income year, other than by reason of above, by an entity
that is a controlled foreign entity at the end of the year are exempt from tax.
 All the distribution by a controlled foreign entity to beneficiaries who are associated with
the entity at the time of distribution shall be treated as;
 Having the same character as to type and source as the entity’s attributable income;
and
 Made proportionately out of each type and source of entity’s attributable income.
 The corporate tax paid by a Controlled foreign entity in relation to the income treated as
having distributed to resident controlling persons in foreign country shall be allocated for
the beneficiaries of Controlled Foreign Entities.

Naresh Singh Bhandari


bnsingh5102@gmail.com Page92
Income Tax Note 2076/77

 Such allocated tax is deemed to be paid by the resident controlling persons and the
resident controlling person may be allowed to claim foreign tax credit on such foreign tax
deemed to be paid.
 The amount deemed to be received by resident person for each income year shall be
treated as outgoings in relation to the asset or liability in connection to interest held by
resident person in such CFE.
 The dividend received by the resident person shall be treated as incomings in relation to
asset or liability in connection to interest held by resident person is such CFE.
 Any tax deemed to be paid as described as above, or any foreign tax paid in relation to
receipt of dividend by controlling person is deemed to be foreign tax paid by the
controlling person as per this act.
 Attributable income of a controlled foreign entity for an income year is its taxable income
for the year calculated as if the entity were a resident entity.
 Controlled foreign entity for an income year means a non-resident entity
 In which a resident person holds an interest, directly or indirectly through one or
more interposed non-resident entities; and
 The person is associated with the entity or would be if the person and not more than
four other resident persons were associated.

5) Taxation of Non-residents Providing Shipping, Air Transport or


Telecommunications Services in Nepal
As per section 70, provisions related to taxation of non-residents providing shipping, air
transport or telecommunication services in Nepal are as follows;

 The taxable income for an income year of a nonresident person who carries on a business
of ship operator, charterer, or air transport operator shall include the following amount
derived during other than as a result of transshipment during the year;-
 The carriage of passenger who embark from territory of Nepal and
 Transport of mail, livestock or other goods that embark from territory of Nepal.
 The taxable income for an income year of a nonresident person who carries on a business
of transmitting messages by cable, radio, optical fiber, or satellite communication shall
include amount derived during the year from the transmission of messages by apparatus
established in Nepal, whether or not such message originate in Nepal.
 The amount included in a nonresident person’s income as above shall be taxed at the rate
of 5 % (as per clause 2(7) of schedule 1).
Provided that,

Naresh Singh Bhandari


bnsingh5102@gmail.com Page93
Income Tax Note 2076/77

 The amount as specified above shall not be included while calculating tax liability of
the person in relation to other taxable income of a person generated within Nepal
besides providing service as air or water transport operator.
 The expenses in relation to the generation of income as specified above shall not be
included while calculating tax liability in relation to any other taxable income of the
person.
 The tax payable under this section cannot be used for the purpose of tax credit.
 Non- resident (for the purpose of this section) person includes a resident entity that is part
of a group of associated entities the main headquarter of which is situated outside Nepal.

Example (3): Rolls Airways is conducting the airplane services between Kathmandu and
Saudi Arabia. Rolls Airways has a branch office at Kathmandu for the purpose of operation
of its services. The income of Rolls Airways for the month of Shrawan, 2067 while
conducting the airplane services and other operations are as below:
SN Particulars Amount (Rs.)
1 Amount received from the passengers departed from Kathmandu 10,000,000
Amount received from the transportation of the goods having first
2 point entry at Kathmandu 5,000,000
Income from the travels from Bhutan and Lhasa having transit point
3 at Kathmandu 5,000,000
Amount received from the rentals of the goods carried from
4 Kathmandu which were transported from Bhutan 5,000,000
Amount received from the services of restaurant operated for the
5 facility of passengers at Kathmandu Airport 2,500,000
Amount received from the services of Ground Handling for other
6 airlines‘ operation 2,500,000
Total Income 30,000,000

The followings are the details of expenditures of Rolls Airways:


SN Particulars Amount (Rs.)
7 Expenditures from Air tickets 5,000,000
8 Expenditures against the accommodation and food for crew members 5,000,000
9 Expenditures against ground handling services 3,000,000
10 Expenditure for the operation of restaurant 2,000,000
11 Expenditure for the transportation of passengers from Bhutan, Lhasa 5,000,000
Expenditure for the re-transportation of the goods transported from
12 Bhutan 5,000,000
Total Expenditure 25,000,000

Note: Rolls Airways has managed the ground handling against 100 flights of its own and 100
flights of other airlines during Fiscal Year 2067/68.

Naresh Singh Bhandari


bnsingh5102@gmail.com Page94
Income Tax Note 2076/77

Explain about the inclusion of above incomes and allowances of expenditures of a non-
resident person, for the purpose of section 70 of Income Tax Act, 2058 stating the relevant
provisions.
Answer:
Section 70 of Income Tax Act, 2058 prescribes the following provisions regarding taxation
on the income of a non-resident person in Nepal from transport or transmission business.
The gross receipts from the following activities of a non-resident person are treated as the
taxable income of the person from the activities.
A person engaged in Nepal in any road, air, water or a chartered transport services, other than
transmission in Nepal, for any of the following services;
i. The carriage of passengers who embark from within a territory of Nepal.
ii. The services of mail, livestock, or other movable tangible assets that are embarked from
the territory of Nepal.
Expenditure incurred in conducting such activities are neither allowed to be deducted from
the taxable income from the above sources nor allowed to be deducted from any other
sources of income of the person.
The tax rate applicable shall be as prescribed under section 2(7) of annexure 1 of Income Tax
Act (which is 5% of gross receipts during the year).
No tax credits shall be allowed to the person to reduce the tax payable by the person under
this section.
Accordingly, following treatment shall be done for the purpose of section 70 in case of Rolls
Airways.
1. For the purpose of section 70, the income of Rolls Airways for the month of Shrawan,
2067 as mentioned in point no. 1 and 2 of question shall be Rs. 15,000,000. Tax shall be 5%
of Rs. 15,000,000=Rs. 750,000 as per section 2(7) of annexure 1 of Income Tax Act.
2. The amount mentioned in point no. 3 and 4 in the question shall not be taxed as per
Income Tax Act, 2058 being sources of payment not in Nepal.
3. The amount mentioned in point no. 5 and 6 in the question are not the incomes to be
included for the purpose of section 70. These incomes shall be taxed like the entity having
general business income.
4. The expenditures incurred in connection with generation of income as mentioned in point
no 5 and 6 in the question are eligible for deduction.
5. The expenditures as mentioned in point no 7, 8 and 9 in the question are not eligible for
deduction for the purpose of computation of income as per section 70 even though they are
incurred in connection with the generation of income from ground handling services by its
own.
6. The expenditures as mentioned in point no 11 and 12 in the question are not eligible for
deduction for the purpose of computation of income under section 70 as these are incurred in
connection with generation of income, the taxation of which are not required to be paid under
Income Tax Act, 2058.
7. Half of the ground handling expenditures amounting to Rs. 1,500,000 and expenditure
against restaurant operation amounting to Rs. 2,000,000 (total Rs. 3,500,000) are allowable
expenditure for deduction as these are incurred in connection with generation of income.

Naresh Singh Bhandari


bnsingh5102@gmail.com Page95
Income Tax Note 2076/77

8. Corporate tax at the rate of 25 percent shall be paid on the amount as mentioned in point
no 5 and 6 in the question amounting to Rs 5,000,000 less expenditure of Rs. 3,500,000
(assuming no other eligible expenditure to be claimed).
Example (4): Explain on the taxability and the implication thereon, of the following
transactions as per the Income Tax Act, 2058.
i) A Den airline registered in Denmark, having contact office in Nepal and is operating its
airlines business. During Income Year 2068/69, it has sold the tickets in Nepal as
follows:
a. Sale of tickets from the passengers departing from Nepal - Rs. 50 crores.
b. Sale of tickets in Nepal, for the passengers departing from country other than Nepal
– Rs. 10 crores.
ii) Singtel Ltd. is a company registered in Singapore. The company, with its objective to
transmit information and storing data, has a communication hub in Nepal (without any
office in Nepal). Through such system, the companies in Europe and America are
storing data and transmitting information. Singtel has received USD 1 million for such
services.
Answer:
i) For the provision of section 70 of Income Tax Act, 2058, the 'non-resident' means a
resident entity that is a part of a group of associated entities the main office is situated
outside Nepal (explanation to section 70).
The gross receipts from the following activity of non-resident person are treated as the
taxable income of the person from the activities:
 A person engaged in Nepal in any, air, water or chartered transport services, other
than transmission in Nepal, for the carriage or passengers who embark from within
the territory of Nepal [section 70(1)].
Hence, the gross receipts of Den Airline from the sale of tickets in Nepal departing the
passengers from Nepal amounting to Rs. 50 crores and for the passengers departing from
country other than Nepal – Rs. 10 crores are taxable at the rates of 5% and 2% respectively
as per schedule 1 Section 2 subsection 7.
No tax credits shall be allowed to the person to reduce the tax payable by the person under
this section. Further, expenditures incurred in conducting such activities are neither allowed
to be deducted from the taxable income from the above sources nor allowed to be deducted
from any other source of income of the person.

ii) The gross receipts from the following activity of non-resident person are treated as the
taxable income of the person from the activity:
 A person engaged in the transmission of messages by cable, radio, optical fiber, or
satellite through an apparatus or equipment established in Nepal, irrespective of the
place of origin of messages [section 70(2)].
Hence, the gross receipts of Singtel from the communication hub (apparatus) established in
Nepal amounting to USD 1 million is taxable at the rates of 5 % as per schedule 1 Section 2
sub Section 7 irrespective of the place of origin of messages.

Naresh Singh Bhandari


bnsingh5102@gmail.com Page96
Income Tax Note 2076/77

No tax credits shall be allowed to the person to reduce the tax payable by the person under
this section. Further, expenditures incurred in conducting such activities are neither allowed
to be deducted from the taxable income from the above sources nor allowed to be deducted
from any other source of income of the person.
Example (5): Antarctica Airlines is an airline company registered in Netherland. The
company has operation in Nepal and for this it has opened a liaison office. In the financial
year 2067-68 the airlines sold the tickets as per following:
i) Tickets sold to the passengers flying from Nepal Rs. 50 crores,
ii) Tickets sold to the passengers flying from places other than Nepal Rs. 10 crores,
iii) Total expense for the operation of Nepal office 5 crores.
You are required to mention the provision of the Act for the Nepal operations of this
company and the taxable income, applicable tax rates and the tax liabilities.
CAP III, Dec 2011 (5 Marks)
Answer:
Antartika Airlines is a non-resident person as per Section 70 of the Income Tax Act. The Act
has prescribed the method of taxation for this type of entity operating in Nepal. Section 2 (7)
of the Annexure 1 of the Act prescribes the applicable tax rate, which are different for
different activities. As per above section read with the rates prescribed following will be the
taxable income, applicable tax rate and the tax liabilities.
Income by way of sale of ticket to the passengers flying from Nepal 50 crores
Income by way of sale of ticket to the passengers flying from elsewhere 10 crores
Taxable income 60 cores
Tax rate applicable
On income by the passenger flying from Nepal 5%
On income by the passenger flying from elsewhere 2%
Tax liabilities
On 50 crores 2.5 crores
On 10 crores 0.2 crores
Total tax liabilities 2.7 crores
As per the provisions of Section 70(3) (kha) no deduction shall be allowed while deriving the
taxable income in such case. Therefore, the operating expenses of Rs. 5 crores will not be
allowed as deduction for Atlantic Airlines.

6) Foreign Tax Credit


As per section 71, provisions related to foreign tax credit are as follows;

 Foreign income tax paid by the resident person with respect to the person’s assessable
foreign income for the year may claim as a foreign tax credit.
 The limit of foreign tax credit shall be lower of following;

Naresh Singh Bhandari


bnsingh5102@gmail.com Page97
Income Tax Note 2076/77

 Tax paid in foreign country, or


 Amount calculated by multiplying the foreign assessable income of each country by
average rate of taxation in Nepal.
 Average rate of tax = (total tax calculated before claim of foreign tax credit/taxable
income of the person including assessable foreign income) *100
 The remaining amount of tax paid in foreign country which cannot be claimed as tax
credit due to the ceiling of average tax paid in Nepal;
 May be carried forward, and
 Shall be treated as paid with respect to assessable foreign income of the person for the
future income year that is sourced in same country.
 Average tax paid in Nepal = Average rate of tax * Foreign assessable income of each
country.
 In case the person rescinds the claim of foreign tax credit, the person is eligible to claim
the tax paid during any income year in relation to the income from foreign source in ny
foreign country as deduction.

Example (6): Mr. A, a resident natural person, is a musician deriving income from concerts
performed from various countries outside Nepal. During Financial Year 2066/67, he
performed concerts in India. Details of his assessable income and tax paid in the country as
aforesaid where the concerts were given are:
Income Tax
India Rs. 1,000,000 Rs. 300,000
Also, he earned Rs. 500,000 in Nepal during the financial year 2066/67.
Assuming that he has chosen to be couple, find his tax liability on his total income and
amount for foreign tax credit available to him under section 71(1) for the income year
2066/67. CAP II, Dec 2010 (10 Marks)
Solution:
Income earned
Nepal Rs. 500,000
India Rs. 1,000,000
Total assessable income Rs. 1,500,000
Total taxable income Rs. 1,500,000
Total tax liability (working note) Rs. 265,000
Less Allowable foreign tax credit:
On Indian Income (working note) Rs. (176,667)
Net tax liability Rs. 88,333

Working note:
Computation of average tax rate for foreign tax credit
Assessable income earned in Nepal Rs. 500,000
Assessable income earned in India Rs. 1,000,000
Total assessable income Rs. 1,500,000
Less: deductions ______-_____

Naresh Singh Bhandari


bnsingh5102@gmail.com Page98
Income Tax Note 2076/77

Total taxable income Rs. 1,500,000

Tax liability computation:


Up to Rs. 400,000 Rs. Nil
Next Rs. 100,000 @ 15% Rs. 15,000
Balance Rs. 1,000,000 @ 25% Rs. 250,000
Total Rs. 265,000
Therefore, average tax rate= 265,000/1,500,000 * 100 = 17.67%
a) Assessable income earned in India = Rs. 1,000,000
b) Tax paid in India = Rs. 300,000
c) Tax to be paid as per average tax rate = Rs. 1,000,000 * 17.67% = Rs. 176,667
d) Excess tax paid to be carried forward for next year (b-c) = Rs. 123,333

Note: Due to different interpretations, in case any student charges 1% or Rs. 400,000 that should also
be treated as right answer. Further the answer has been calculated on the basis of finance bill 2073/74.

Example (7): Mr. Z has a source of income in Nepal and also in more than one foreign
country. During the Fiscal Year 2067/68, income and tax paid in each foreign country is
given below;
Name of the Country Income (Rs.) Tax Paid (Rs.)
USA 200,000 60,000
Australia 150,000 30,000
UAE 100,000 5,000
Nepal 250,000 -
He is a resident natural person and selected for the couple as tax payers during the year.
Calculate his tax liability during the Fiscal Year 2067/68.
Answer:
Calculation of the tax liability of Mr. Z for the Fiscal Year 2067/68
Total assessable income from all the sources
Particulars Amount (Rs)
Net Income from Nepal 250,000
Net Income from USA 200,000
Net Income from Australia 150,000
Net Income from UAE 100,000
Total Income 700,000

Tax Calculation
First Rs. 400,000 @ 0% 0
Next Rs. 100,000 @ 15% 15,000
Remaining Rs. 200,000 @ 25% 50,000
Total Tax 65,000
Average Tax Rate (65,000/700,000*100) = 9.29%

Tax credit for the year shall be available for;


Tax Calculated Tax credit Unabsorbed tax
Country Income Tax Paid at average rate available for credit to be

Naresh Singh Bhandari


bnsingh5102@gmail.com Page99
Income Tax Note 2076/77

the year carried forward


USA 200,000 60,000 18,580 18,580 41,420
Australia 150,000 30,000 13,935 13,935 16,065
UAE 100,000 5,000 9,290 5,000 -
Total 37,515 57,485

Tax payable during the year comes to Rs. 65,000-37,515 = Rs. 27,485

Note: It is supposed that all the incomes are from business. If income is supposed from
employment, then tax should be calculated on Rs. 400,000 @ 1 %.
Example (8): Mr. Gopal is a professor in TU. He earned Rs. 600,000 employment income in
Nepal in Fiscal Year 2067/68. Out of which Rs. 150,000 was deposited by employer in
Approved fund. In that year he went to India for 2 months and earned Rs. 330,000 from
consultancy and paid tax Rs. 49,500 as per Indian Income Tax Act. Again he went to USA
for 2 months, worked in a research centre and earned Rs. 300,000 out of which tax was
deducted Rs. 60,000 and deposited in USA as per USA tax law. Nepal has no double taxation
Treaty with USA but it has double taxation Treaty with India.
TDS on employment income of Nepal is not deducted. He opted for couple.
Calculate Income tax to be paid in Nepal. CAP III, Dec 2011 (5 Marks)
Answer:
Calculation of taxable Income:
Employment Income in Nepal 600,000
Employment Income in USA 300,000
Employment Income in India 330,000
Assessable Income 1,230,000
Less: Contribution to ARF
Lower of
- Actual = 150,000
- 1/3*1,230,000 = 410,000
- Max. Rs. 300,000 150,000
Taxable Income 1,080,000

Tax Calculation
First 400,000 @ 1% 4,000
Next 100,000 @ 15% 15,000
Next 580,000 @ 25% 145,000
Total Tax 164,000
Average tax rate = (164,000/1,080,000) *100 = 15.19%

Tax credit for the year shall be available for;


Tax Tax Calculated Tax credit available Carried forward
Country Income Paid at average rate for the year tax credit
USA 300,000 60,000 45,570 45,570 14,430

Naresh Singh Bhandari


bnsingh5102@gmail.com Page100
Income Tax Note 2076/77

India 330,000 49,500 50,127 49,500 -


Total 95,070 14,430
Net tax payable during the year = Rs. 164,000 – 95,070 = Rs. 68,930.

Naresh Singh Bhandari


bnsingh5102@gmail.com Page101
Income Tax Note 2076/77

Tax Administration and Official Documentation

1) International Agreement
International agreement means any treaty or agreement which is enforceable in relation to
Nepal concluded with a foreign country to make provisions in connection to following;
 Avoidance of double taxation and the Prevention of Fiscal Evasion, or
 Reciprocal Administrative Assistance in relation to implementation of Tax Liability.
As per section 73, provisions related to international agreement are as follows;

 In case any income of a person is subject to income tax under Income Tax Act or any
other prevailing law of Nepal during any income year and the same income is subject to
tax under laws of any other country, GON may conclude a double tax avoidance
agreement to avoid such double taxation on same income.
 Where Inland Revenue Department receives a request pursuant to an international
agreement from the competent authority of another country for the collection in Nepal of
an amount payable by a person under the tax laws of another country, IRD may, by
service of notice in writing, require a tax debtor to pay the amount to IRD by the date
specified in the notice and for transmission to the competent authority.
 In case where an international agreement provides Nepal to exempt income or a payment
or subject income or payment to reduced tax, the exemption or reduction is not available
to any entity;
 Who, for the purpose of the agreement, is resident of the other contracting state
 50 % or more of whose underlying ownership is owned by individuals or entities in
which no individual has an interest who, for the purposes of the agreement, are not
residents of the other contracting state or Nepal.

2) Taxpayer’s Right
As per section 74, provisions related to tax payer’s right are as follows;
 Tax payer shall abide by the duties in accordance with this act.
 A tax payer with respect to paying tax under this act shall have the following rights;
 Right to get respectful behavior,
 Right to receive tax related information as per the prevailing laws,
 Right to get opportunity of submitting proof in own favor in respect of tax matter,
 Right to appoint lawyers or auditors for defense, and
 Right to secrecy in respect of tax matters and keeps inviolable.

Naresh Singh Bhandari


bnsingh5102@gmail.com Page102
Income Tax Note 2076/77

3) Public Circular
As per section 75, provisions related to public circular are as follows;
 The department may issue in writing public circular setting out the department’s
interpretation of the act for the following purposes;
 To achieve consistency in the implementation of this act,
 To make tax administration simple and
 To provide guidance to the person affected by this act.
 The department shall make public circulars issued as above available to the public at
offices of the Department and at such other locations or by such other medium as
prescribed by the department.
 A public circular issued as above shall be binding on the Department.

4) Personal Ruling
As per section 76, provisions related to advance rulings are as follows;

 In case a person makes an application in writing to the IRD seeking IRD’s position
regarding the application of this act with respect to an arrangement proposed or entered
into by the person, IRD under signature of DG may issue, in writing, an advance ruling in
this regard.
 IRD shall not issue such an advance ruling on the matters under consideration of any
court or decided by court.
 Under the following circumstances the ruling shall be binding on every officer of IRD,
IRO’s and a civil servant who is authorized to work as tax officer;
 The application has made a full and true disclosure of all aspects of arrangement
relevant to the ruling, and
 The arrangement proceeds in all material respects as described in the application for
the ruling.
 Where the advance ruling issued by IRD contradicts with a public circular, for the person
who applied for the advance ruling, the advance ruling shall be applicable.
 The department may allow the applicant or its representative to submit inadequate
document prior to issuing personal rulings.

As per Rule 22, procedures for Personal Ruling are as follows;


 Any person seeking a personal ruling shall apply in the format prescribed by the
Department.
 The department shall issue a personal ruling within 45 days after the application is
received from the person.

Naresh Singh Bhandari


bnsingh5102@gmail.com Page103
Income Tax Note 2076/77

 If the Department has not issued a personal ruling to a person within 45 days of
application received, the person may apply for administrative review to the Department
or may file an appeal against the ruling with the Revenue Tribunal.

5) Form of Documentation
As per section 77, provisions related to form of Documentation are as follows;

 The department may from time to time specify the form of documents required under this
act, which shall contain such information as is required by this act, by the rules, and for
the efficient administration of this act.
 The department shall make the forms available to the public at offices of the department
and at such other locations or by such other medium as the department may determine.
 The department may prescribe a person to furnish any notice or details or documents to
be submitted to the department through electronic medium.

6) Permanent Account Number


As per section 78, provisions related to Permanent Account Number are as follows;

 The department shall be issue a person with a number to be known as PAN for the
purposes of identifying a person.
Provided that an entity authorized from the Department may provide such PAN with due
procedures of this Act. The taxpayer with such PAN cannot carry import export
transaction till the period as prescribed by Department.
 The Department may require the person to show their PAN in any return, statement, or
other document used for the purposes of this Act.
 Department may prescribe situations in which a person is required to show or quote their
PAN.
 A person designated as above (Subsection 3) should have PAN before carrying out
transaction.
 Person operating transaction after obtaining PAN shall update registration information in
biometric system within prescribed time as prescribed by IRD.
 No immunity will be availed in lack of PAN.
As per Rule 23, provisions related to PAN are as follows;
 Any person who is not the current holder of PAN at the commencement of this Rules and
intends to derive assessable income or make payment from which the person will be

Naresh Singh Bhandari


bnsingh5102@gmail.com Page104
Income Tax Note 2076/77

required to withhold tax shall apply to the Department for a PAN before deriving the
income or making payment, as the case requires.
 Any person who is not required to apply for PAN as above and is not the current holder
of a PAN, may also apply to the Department for a PAN.
 If application is received, the Department shall provide the person with a PAN certificate.
As per rule 24, provisions related to modification of details in PAN certificate are as
follows;
 A person issued with a PAN certificate shall notify the Department of any changes in the
details recorded in the Certificate within 15 days of the change.
 Upon receipt of a notification as above, the Department shall make any necessary
amendment in the PAN.

7) Service of Documents
As per section 79, provisions related to service of documents are as follows;

 If A document to be served on a person under this act shall be considered as sufficiently


served if:
 Served via telefax, telex, email or similar other electronic medium,
 Handed to the person or, in the case of an entity, a manager of the entity, or
 Left at or sent by post to the usual or last known place of abode, business, office, or
other address of the person.
 A document issued, served, or given by the Department under this act shall be treated as
sufficiently authenticated if the name or title of an authorized officer of the Department is
signed, encrypted or encoded by means of computer technology, stamped, or written on
the document.
 If the document cannot be served by above ways, then such information may be served to
the concerned person by broadcasting or publishing the order in radio or television or
newspaper. The order broadcasted and published deemed to be served to the concerned
person.

8) Defective Documents
As per section 80, provisions related to defective documents are as follows;

 A documents issued as follows under this act shall not be treated as defective;-
 If it is in substance and effect in conformity with the act, and
 If the person to whom the document is addressed is designated in it according to the
common understanding.
Naresh Singh Bhandari
bnsingh5102@gmail.com Page105
Income Tax Note 2076/77

 Where a document issued by the department under this act contains a defect that does not
involve a dispute as to the interpretation of this Act or facts involving a particular person,
the Department may, for the purposes of rectifying the defect, amend the document.

Naresh Singh Bhandari


bnsingh5102@gmail.com Page106
Income Tax Note 2076/77

Withholding

1) Withholding by Employer
As per section 87, provisions related to Withholding by employer are as follows;

 Every resident employer shall be required to withhold tax at the rate mentioned below
from a payment with a source in Nepal that is to be included in calculating income of an
employee from the employment.
In case of individual In case of couple Rate
First Rs. 400,000 First Rs. 450,000 1%*
Next Rs. 100,000 Next Rs. 100,000 10%
Next Rs. 200,000 Next Rs. 200,000 20%
Next Rs. 1,300,000 Next Rs. 1,250,000 30%
Balance Balance 36%
* 1% rate shall not be applied to the proprietorship firm and natural person having
pension income and contributing to contribution based Social Security Fund.
 The obligation of an employer to withhold tax as above shall not be reduced or
extinguished because of the following;
 Right or obligation of the employer to deduct and withhold any other amount from
the payment; or
 Any other law that provides that an employee’s income from employment shall not be
reduced.

2) Withholding on Investment Return and Service Fee


As per section 88, provisions related to withholding on Investment Return and service fee are
as follows;

 A resident person shall withhold tax @ 15 % while making payment of following that
have a source in Nepal;
 Interest,
 Natural Resource Payment,
 Rent,
 Royalty,
 Service Fee,
 Commission,
 Sales Bonus,
 Retirement Payment and
 Any other benefit/return
Naresh Singh Bhandari
bnsingh5102@gmail.com Page107
Income Tax Note 2076/77

 Tax withholding will be as follows in the following payment;

Particulars Rate
Retirement payment from GON and Approved Retirement Fund 5%
Commission paid to non-resident by a resident employment company 5%
Aircraft lease 10%
Payment of service fee to VAT registered/VAT exempted service provider 1.5%
Rent Paid by resident person to other than natural person 10%
Rent paid to the vehicle rental service provider registered on VAT 1.5%
Rent paid to Vehicle service provider registered under VAT 1.5%
Return payment by Mutual Fund to natural person 5%
Payment against use of satellite, bandwidth, optical fiber, telecommunication
equipment or electricity transmission line by resident person 10%
Payment of Dividend 5%
Gain from Investment Insurance 5%
Gain from unapproved retirement fund 5%
Rent paid in relation to the transport service 2.5%
*withholding tax shall not be deducted on the VAT refund made to the person while
making payment of through electronic medium (Section 25(1kha) of VAT Act 2052).
 If all of the following condition satisfied, the withholding tax on payment of interest shall
be 5 %;
 Interest is paid by the resident bank or financial institutions, Cooperative or any
entities issuing debentures or companies listed under prevailing law,
 Interest is paid on deposit, debentures, loan papers or government bonds,
 Interest is paid to natural person,
 Interest should have a source in Nepal,
 Interest shall not be paid/ received in connection to operation of business.
 Tax shall not be with hold on the following payment;
 Payment made by natural person except the payment related to his/her business,
 Payment for articles published in newspaper,
 Interest paid to the resident bank or other resident financial institution,
 Payment that are exempt from tax or subject to withholding by employer (section 87),
 Interregional interchange fee paid to a credit card issuing bank,
 Dividend and interest paid to a mutual fund.

3) Tax Withholding on Windfall Gain


As per section 88(ka) provisions related to tax withholding on windfall gain are as follows;

Naresh Singh Bhandari


bnsingh5102@gmail.com Page108
Income Tax Note 2076/77

 Tax withholding rate on windfall gain = 25 %


 GON can exempt tax on windfall gain by publishing notice in Nepal gazette on national
or international level prize received by contributing in literature, arts, culture, sports,
journalism, science, technology and public administration.
 National or international prize up to Rs. 5 lakhs received by contributing in literature,
arts, culture, sports, journalism, science, technology and public administration shall not
be taxed.

4) Withholding from Contract Payments


As per section 89, provisions related to withholding from contracting payment are as follows;

 A resident person making a payment under a contract exceeding Rs. 50,000 shall
withhold tax on the gross amount @ 1.5%.
 The amount Rs. 50,000 as mentioned above shall be calculated by aggregating a payment
made by a person under a contract with any other payment made by the person or an
associate of the person during the previous 10 days under the same contract to the same
payee or an associate of the payee.
 A resident person shall be required to withhold tax from a payment to a non-resident
person under contract as follows;
 5 % of aircraft repair and other contract,
 1.5% on premium paid to non-resident insurance company,
 In case department has served the resident person with a written notice - at the rate
specified in the notice, in other than the above cases.
 There shall be withholding of tax @ 5% in case where payment of more than NPR 5
million is made for the work done through Consumer Committee.
 Under the following situation the withholding agent is not required to withhold tax even
if the following payment are covered by definition of contract;
 Payment by a natural person except in the following condition;
 If the payment in the course of operation of his/her business, or
 If the payment is for rent against the use of house, land or apparatus or equipment
attached to such land or house.
 In case the payments are exempt from tax.
 In case the payments attract WHT under section 87 and 88.
 For the purpose of this section, “contract” means contract related to followings;
 The agreement for supply of goods or labor or
 Construction, installation or establishment of a tangible assets or structures,
 The jobs prescribed by the Department as a contract and

Naresh Singh Bhandari


bnsingh5102@gmail.com Page109
Income Tax Note 2076/77

 If the contract includes the payment for the service in regard to the construction,
installation, or establishment, the payment under the contract for the services will also
be taken as the payment under contract.

5) Withholding Return and WTH Payment


As per section 90, provisions related to filing withholding return and payment of withholding
tax are as follows;

 Withholding agent shall file withholding return to respective office within 25 days after
the end of each month in a manner and form prescribed.
 Withholding agent shall be required to deposit withholding tax collected/withhold or
deemed to be withheld by it during any month within 25 days of end of such month.
 A withholding agent who fails to withheld tax shall be treated as though the tax had been
withheld at the time required.
 A person paying tax on turnover shall deposit tax withhold on various payment by it at
the time when it pays installment tax.
 Responsibility to deposit deemed withholding tax;
 Both the parties are jointly or separately responsible to deposit tax in respective tax
office.
 In case withholding agent deposit the amount of withholding tax that is deemed to be
withheld to the tax office, he gets right to recover the amount from the withholdee.
 There is no charging of interest in case the withholdee deposit the deemed
withholding tax within 25 days after lapse of 25 days for a withholding agent.
 In case the withholding agent has withhold tax and deposited to the tax office, then the
withholding amount is deemed to be paid to the withholdee.

6) Withholding Certificate
As per section 91, provisions related to withholding certificate are as follows;

 A withholding agent shall be required to prepare and serve on a withholdee a withholding


certificate in the following form within the 25 days of end of month;
 Certified in a manner, if any prescribed by the Department; and
 Set out the amount of payment made and tax withheld under section 87, 88 or 89.
 In case of the tax withhold by employer, withholding certificate shall be provided as
follows;
 The certificate shall be for the part of the income year during which the employer is
employed; and

Naresh Singh Bhandari


bnsingh5102@gmail.com Page110
Income Tax Note 2076/77

 The certificate shall be required to serve within 30 days after the end of the year or,
when the employee ceased employment with the withholding agent during the year,
no more than 30 days from the date on which the employment ceased.

7) Final Withholding Payment


As per section 92, provisions related to Final Withholding Payments are as follows;

 The following payment shall be treated as final withholding payments;


 Dividend paid by resident company and proprietorship firm,
 Rent for lease of land or a building and associated fittings and fixtures, having source
in Nepal, and that is received by a natural person other than in conducting business,
 Payment of gain on investment insurance by resident person,
 Payment of gain on unapproved retirement fund by resident person,
 Payment of interest by the bank, financial institution, or any other institution issuing
debentures, or company listed under prevailing laws and having following conditions;
 Payment having source in Nepal and received by natural person other than
conducting business,
 Payment made to exempt organization.
 Payment made to the non-resident person after withholding tax,
 All types of retirement payments including the retirement payments made by GON,
an approved retirement fund or an unapproved retirement fund (except regular
pension payment),
 Meeting fee up to NPR 20,000 per meeting, occasional teaching fee, payment of
setting question paper and checking answer sheets,
 Payment for windfall gain and
 Return amount distributed to a natural person by a mutual fund,
 Payment made to the natural person for leasing vehicle and transport which ate not
operated thorough private firm.
 Tax withhold or treated as withhold from a final withholding payment that is paid to
Department by the withholding agent or the withholdee shall be treated as satisfied the
withholdee’s tax liability.

8) Inclusions and Credit for Non-Final Withholding Tax


As per section 93, provisions related to inclusions and credit for non-final withholding tax
are as follows;

 The tax withhold from any payment are treated as part of such payments.

Naresh Singh Bhandari


bnsingh5102@gmail.com Page111
Income Tax Note 2076/77

 The withholdee is deemed to deposit the amount as follows as tax in case of withholding
of taxes on payments other than on final withholding payments;
 Tax amount withheld on payment attracting WHT under section 87, 88 & 89.
 In case the withholding agent or withholdee has deposited withheld amounts or
amounts deemed as withheld (section 90(3)) in tax office
 The amount deemed to be deposited as tax as above can be claimed as advance tax in the
year when the related payment is included in income by the withholdee.

Naresh Singh Bhandari


bnsingh5102@gmail.com Page112
Income Tax Note 2076/77

Installment and Advance Tax

1) Payment of Income Tax by Installment


As per section 94, provisions related to payment of income tax by installment are as follows;

 A person who derives or expects to derive any assessable income during an income year
from business or investment shall require paying tax for the year by three installments as
follows;
Date Payable Amount Payable
By the end of Poush Due tax amount out of 40% of estimated tax
By the end of Chaitra Due tax amount out of 70% of estimated tax
By the end of Ashad Due tax amount out of 100% of estimated tax
 A person who pays tax on turnover shall require paying tax for the year by two
installments as follows;
Date Payable Amount Payable
By the end of Poush Tax calculated at the prescribed rate on the basis of
actual turnover till Poush 20.
By the end of Ashad Remaining amount of tax payable based on the
estimated turnover calculated by applying
prescribed tax rate. Estimated turnover for the
income year shall be calculated on the basis of
actual turnover till Ashad 20.
 If the total amount of estimated tax of all three installments during the income year is less
than Rs. 7,500, then the person is not required to make payment of installment tax.
 The person paying tax under installment can claim such tax as advance tax while
calculating tax payable by him/her for the year.

Example: Details of annual estimated tax and withholding tax of A & Co. are as follows:
Estimated Tax
Poush end Rs. 1,000,000
Chaitra end Rs. 1,200,000 (Re-estimated
Ashad end Rs. 1,200,000 (Re-estimated)

Withholding Tax details:


Upto Poush end Rs. 20,000
Magh to Chaitra end Rs. 10,000
Chaitra to Ashad end Rs. 15,000
Compute the total Advance Tax to be paid for each installment.
CAP II, June 2011 (5 Marks)

Naresh Singh Bhandari


bnsingh5102@gmail.com Page113
Income Tax Note 2076/77

Solution:

1st Installment (Poush end)


= 40% of estimated tax – withholding tax – Advance tax
= 40% of Rs. 1,000,000 – Rs. 20,000 – 0
= Rs. 380,000
2nd Installment (Chaitra end)
= 70% of Rs. 1,200,000 – Rs. 30,000 – Rs. 380,000
= Rs. 430,000
3rd Installment (Ashad end)
= 100% of Rs. 1,200,000 – Rs. 45,000 – Rs. 810,000
= Rs. 345,000

Total Advance Tax paid:


Withholding Tax Rs. 45,000
1st Installment Rs. 380,000
2nd Installment Rs. 430,000
3rd Installment Rs. 345,000
Total Rs. 1,200,000

2) Return of Estimated Tax Payable


As per section 95, provisions related to return of estimated tax payable are as follows;

 Every person who is an installment tax payer for an income year shall file an estimated
tax return within the due date of making payment of first installment estimating the
amount as follows in the form and manner prescribed by IRD;
 Assessable income from each of business, employment or investment and source of
such income,
 Taxable income of the person for the income year,
 Tax amount payable by person with respect to that income without deducting medical
tax credit,
 In case of foreign permanent establishment, the amount to be repatriated by such FPE
and the tax payable on such repatriation.
 Other matters prescribed by IRD.
 The estimated tax amount shall be sum of tax calculated by the person as follows;
 Tax amount payable by person as per section 3(ka) without deducting medical tax
credit.
 In case of foreign permanent establishment, the amount to be repatriated by such FPE
and tax payable on such repatriation.

Naresh Singh Bhandari


bnsingh5102@gmail.com Page114
Income Tax Note 2076/77

 The estimated amount of tax paid for income having source outside Nepal shall include
the sum of following;
 Any tax paid in foreign country by such person, and
 The amount of estimated by the person as payable as tax in foreign source income.
 An installment tax payer’s estimate shall be valid for the whole income year unless the
person files a revised estimate, in the form and specifying the information as above, to
the Department together with a statement of reasons for revisions.
 A revised estimate filed by a person as above shall be used only in calculating
installments payable for the income year after the date the revised estimate is filed with
the Department.
 The Department may specify that an installment tax payer or class of installment payers
are not required to submit an estimate.
 In case the tax payer has not submitted the estimated tax return or IRD is not satisfied
with the estimation filed by the tax payer, IRD may issue a notice to the taxpayer stating
reason of dissatisfaction over the figures of estimation by the taxpayer, bases of the
revised estimation by IRD, and the figures of revised estimation and make a revised
estimation based on previous year’s figures of the same tax payer.
In such circumstances the tax payer has to deposit the amount to IRD according to the
revised estimation made by the latter.

3) Advance tax collection


As per section 95(ka), provisions related to advance tax collection are as follows;

 Advance tax will be recovered by the entity conducting commodity future market
business @ 10% on the profit and gains earned by a person from trading under
commodity future market.
 Following advance tax will be recovered on the gain calculated (as per section 37) in case
of gain from disposal of interest in resident entity by a person except by a resident entity
registered under prevailing law for carrying out transaction of buying and selling of
securities;
 In case of gain from disposal of interest of an entity listen in SEBON, by the entity
transacting in security exchange market – 5 % on the gain for resident natural person
and 10% on the gain for resident entity and 25% for others.
 In case of gain from disposal of interest of an entity not listed in SEBON, by the
entity of which the interest is disposed off – 10% on the gain for a resident natural
person and 15% on the gain for resident entity and 25% for others.
 The outgoing for the securities shall be calculated on by using weighted average
method.

Naresh Singh Bhandari


bnsingh5102@gmail.com Page115
Income Tax Note 2076/77

 Company Registrar Office shall record the change in shareholding status of the unlisted
entity only after obtaining proof of deposit of advance tax collected as above.
 Advance tax shall not be collected on disposal of interest by the mutual fund.
 The Land Revenue Office will collect advance tax as follows at the time of registration
on the capital gain from disposal of land or private building of a natural person;
 @ 2.5% in case the ownership of the NBCA (land and private building) has been 5
years or more.
 @ 5% in case of ownership of the NBCA (land and private building) has been less
than 5 years.
 In case of the person other than natural person as above, Land Revenue Office shall
collect advance tax @ 1.5% of gain on disposal of land and building.
 Custom office shall collect advance tax 5% on custom value at the time of releasing of
he-buffalo, buffalo, he-goat, boka, sheep, chyangra falling under part 1 of Harmonized
Cos System, Live, fresh and frozen fish falling under Part 3 of Harmonized System, fresh
flowers falling under part 6 of Harmonized System, fresh vegetables, potato, onion, dry
vegetables, garlic and baby corn are imported for business propose.
 Custom office shall collect advance tax 2.5% on custom value at the time of releasing of
meat (part 2 of HS System), milk products eggs, honey falling under part 4 of HS system,
millet, fapar, junelo, rice, kanika falling under part 10 of HS system, maida, aanta, pitho
falling under part 11 of HS system, herbs, sugarcane falling under part 12 of HS System
and plants products falling under part 14 of HS system imported for business purpose.
 A person requiring collecting advance tax is deemed to have collect advance tax at the
time of collecting advance tax as above even if it has not collected the advance tax.
 The advance tax return and advance tax amount should be deposited in the Department
by the person collecting advance tax in the method and format as prescribed by the
Department within 25 days of elapsing of every month.
 In the following conditions, both the person collecting advance tax and the person from
whom it is to be recovered shall jointly and separately be liable, for the payment of the
tax, to the department;
 If the advance tax was not collected by a person required to collect; and
 If the tax deemed to have collected was not deposited in the department.
The person requiring depositing tax in this case should deposit tax within 25 days from
the elapse of 25 days of the end of month and advance tax amount can be recovered from
the person from whom it was required to be collected in case the amount is deposited into
the Department by the person required to recover advance tax.
 The advance tax deposited can be reduced from the annual tax liability by the person
require to deposit advance tax.

Naresh Singh Bhandari


bnsingh5102@gmail.com Page116
Income Tax Note 2076/77

Return of Income and Assessments

1) Return of Income
As per section 96, provisions related to filing return of Income are as follows;

 Every person shall file return of Income for the year within 3 months after the end of
each income year.
 The return of income shall include;
 Assessable income from each of business, employment or investment and source of
such income,
 Taxable income of the person for the income year,
 Tax amount payable by person with respect to that income without deducting medical
tax credit,
 In case of foreign permanent establishment, the amount to be repatriated by such FPE
and the tax payable on such repatriation.
 Other matters prescribed by IRD.
 Return of income shall be signed by the person or the manager and include a declaration
that the return is complete, true and accurate.
 In the following case, the department may, prior to date for filing a return of income for
an income year, by notice in writing served on the person, require the person to file a
return of income for the year or part of the year within the date specified in the notice;
 A person becomes bankrupt, is wound up or goes into liquidation.
 The person is about to leave Nepal definitely.
 The person is otherwise about to cease activity in Nepal and
 The Department otherwise considered it appropriate.
 If a person submitted income return within due date and requires amendment in such
return, then such return may be amended within 30 days of filing return as per the
procedures prescribed by IRD.

2) Return Filing Not Required


As per section 97, no return of income for an income year shall be required by the following
person;
 A person who has no taxable income in an income year,
 A person receiving a final withholding payment in an income year,
 A resident natural person to whom section 4(3) applies for the year or

Naresh Singh Bhandari


bnsingh5102@gmail.com Page117
Income Tax Note 2076/77

 A natural person who is owner of vehicle other than owing vehicle through private firm
used as public transportation and is liable to pay tax under section 1(13) of schedule.
 A natural person deriving income only from disposal of NBCA and opts for non-filing oif
return.

3) Extension of Time to File Return


As per section 98, provision related to extension of time to return file are as follows;

 If the person makes an application to the department for the extension of time to file
return, the Department may, on such terms and condition as prescribed by Department
and where reasonable cause is shown, extend the date by which the return is to be filed
and serve the person with written notice of the Department’s decision on the application.
 The Department may grant multiple extensions with respect to filing of return of income
but the extension shall not in total exceed three months.

4) Tax Assessment
As per section 99, provisions related to tax assessment are as follows;

 If the person files a return of income for an income year, an assessment is treated as made
on the due date for filling the return of;
 The tax payable by the person for the year in the amount shown in the return and
 The amount of tax that is still to be paid for the year being the amount show in the
return.
 If the person fails to file a return for an income year then, until such time as the return is
filed, as assessment is treated as made on the due date for filing the return that;
 The amount of tax payable by the person = tax withheld from payments derived by
person + tax paid by person in installment for the year
 There is no tax payable on the assessment.

5) Jeopardy Tax Assessment


As per section 100, provisions related to jeopardy tax assessment are as follows;

 If the person is required to file a return of income year under following circumstances for
part of an income year, jeopardy tax assessment shall apply;
 A person becomes bankrupt, is wound up or goes into liquidation.
 The person is about to leave Nepal definitely.

Naresh Singh Bhandari


bnsingh5102@gmail.com Page118
Income Tax Note 2076/77

 The person is otherwise about to cease activity in Nepal and


 The Department otherwise considered it appropriate.
 If the jeopardy tax assessment is made than;
 The assessed person shall not file a return of income for the full year or
 The assessed person is still required to file a return of income for part of income year.
 Any tax paid on an assessment of the part of an income year shall be credited against tax
payable on an assessment made for full income year.
 The Department shall provide 7 days to produce proof, if any, in own favor while making
jeopardy tax assessment.

6) Amended Tax Assessment


As per section 101, provisions related to Amended tax assessment are as follows;

 The Department may amend a self-assessment or jeopardy tax assessment to adjust the
tax liability of the assessed person according to the Department best judgment in
consistent with the intention of this act.
 The Department many amend the assessment as many time as it thinks appropriate.
 The Department shall be required to make an amended assessment within a period of four
years after the following date;
 In case of self-assessment: the due date for filing a return.
 In case of jeopardy tax assessment: the date on which notice of assessment is served
on person.
 In case of amended assessment: the due date for filing a return if self-assessment is
amended and the date on which notice of assessment is served if jeopardy assessment
is amended.
 The Department may amend assessment at any time if the assessment is inaccurate by
reason of fraud. Such assessment shall be made within 1 year after receiving a
information about filing return and making an assessment by fraud.
 The Department shall not amend an assessment if the assessment has been amended or
reduced pursuant to the order of Revenue Tribunal or a court of competent jurisdiction
except where the order is reopened.
 While making an amended assessment, the Department shall be required to grant an
opportunity in writing to produce proof, if any, in own favor with respect to assessment,
specifying the basis that led to amended assessment and giving a time limit of 15 days.

Naresh Singh Bhandari


bnsingh5102@gmail.com Page119
Income Tax Note 2076/77

7) Notice of Tax Assessment


As per section 102, if the Department makes jeopardy tax assessment or amended
assessment, the Department shall provide a written notice of the assessment on the person
stating the following;
 The tax payable by the person and the tax still to be paid on the assessment for the
income year or a period to which the assessment is made.
 The manner in which tax payable by the person on assessment is calculated.
 The reason why the Department has made the assessment.
 The date on which the tax still to be paid on the assessment is payable and
 The time, place and manner of objecting to the assessment.

Naresh Singh Bhandari


bnsingh5102@gmail.com Page120
Income Tax Note 2076/77

Review and Appeal

1) Reviewable Decisions and Procedures:


As per section 114, provisions related to reviewable decisions and procedures are as follows;

 The administrative review may be conducted on following decisions;


a) Personal ruling issued by the Department (Section 76).
b) The order issued by the Department for depositing tax not deposited or less deposited
in case the Department is less convinced on not deposited the statement or tax by the
person (Section 90(8)).
c) An estimate made by the Department or the decision to make an estimate, of the
person’s estimated tax payable (Section 95(7)).
d) A decision by the Department requiring a person to file a return of income (Section
95(5) or 97).
e) A decision by the Department on an application by a person to extend the due date by
which the person must file a return (Section 98).
f) An assessment of tax payable by the person on jeopardy tax assessment or amended
tax assessment, assessment of cost of charge and auction of asset sold (section
105(5)), and assessment of interest and charges (section 122).
g) Notification by the Department of an amount to be set aside by a person as a receiver
(Section 108(2)).
h) A decision by the Department requiring a person to pay monies owing to tax debtor to
the Department (section 109(1)).
i) A decision by the Department to require a person to pay tax on behalf of non-resident
person (section 110(1)).
j) A decision by the Department on an application by a person for refund of tax (section
113(5), and
k) A decision made by the Department on an application by a person for an extension of
time within which to file an objection (section 115(3)).
 If the Department makes decision with respect to matters referred in clause e, j and k but
fails to serve a person making an application with the required notice of the decision
within 30 days of the application being made, the Department shall be treated as having
decided to refuse the application and the administrative review on such decision may be
conducted.

2) Apply for Administrative Review


As per section 115, provisions related to apply for administrative review are as follows;
Naresh Singh Bhandari
bnsingh5102@gmail.com Page121
Income Tax Note 2076/77

 A person who is aggrieved by a reviewable decision may file an objection to the decision
with the Department within 30 days after the decision is made.
 An objection as mentioned above must be in writing and specify in detail the grounds
upon which it is made.
 If a person made an application for extension of time limit of 30 days as a result of its
expiry within a period of 7 days from the expiry, the Department may extend for a period
not exceeding 30 days if the reasonable cause is shown and shall serve the person with
the written notice of the Department decision on the application.
 The enforcement of decisions for which administrative review is conducted shall not be
treated as stayed or otherwise affected by filing of an objection as above.
 The person applying for the administrative review shall deposit the following amount to
the Department;
 100 % of undisputed tax and
 1/4th of disputed tax.
 After consideration of a person’s objection filed, the Department may allow or disallow
the objection in whole or part and shall serve the person with written notice of the
decision on an objection.
 If the Department fails to serve a person with the notice of the decision on an objection
within 60 days of an objection being files with the Department, the person may make an
appeal to the Revenue Tribunal.
 If a person made an appeal to the Revenue Tribunal as above, then such information
should be given to the Department in writing along with the copy of the appeal within 15
days of making appeal.

3) Appeal to Revenue Tribunal


As per section 116, provisions related to appeal to revenue tribunal are as follows;
 A person who is aggrieved by a decision on an objection may appeal to the Revenue
Tribunal in accordance with Revenue Tribunal Act, 2031.
 A person who appeals to revenue Tribunal shall, within 15 days of doing so, file a copy
of the notice of appeal with the Department.
 The enforcement of decision made under administrative review shall not be treated as
stayed or otherwise affected by an appeal made to revenue tribunal.
 If reviewable decision under administrative review is made by Director General, an
appeal may be filed with the Revenue Tribunal against the decision.

Naresh Singh Bhandari


bnsingh5102@gmail.com Page122
Income Tax Note 2076/77

Charges and Interest

1) Charges for Failure to Maintain Documents or File Returns


As per section 117, provisions related to charges for failure to maintain documentation or file
returns are as follows;
 If the person does not file the return, following charges shall be imposed on such persons;
Particulars Section Charges
Not Filing of Return of estimated 117(1ka) Rs. 5,000 or 0.01% of Assessable Income
tax payable as per section 95(1) as specified in Income Return whichever is
higher.
Return not filed by the person 117(1kha) 1.5 % p.a. calculated for a month and part
responsible for collecting advance of month of the amount to be recovered as
tax as per section 95 ka(5) advance tax from due date of return filing
to the date of actual return filed.
Income tax return not filed as per 117 (1Ga) Rs. 100 per month.
section 96 (1) by small tax payer
mentioned in section 4(4).
Return not filed by person as per 117 (1Ga) Higher of following;
section 96 (1) by large tax payer  0.1% p.a. of the person’s assessable
other than mentioned in section income without any gross deductions or
4(4).  Rs. 100 per month.
Financial statements not submitted 117 (1Gha) 0.1% p.a. of amount shown in the income.
by the tax exempted entity within
prescribed time period.

 A person who fails to maintain the documents as mentioned in section 81 shall be liable
to pay a charge for the year during which the documentation is not maintained calculated
as higher amount of 0.1% of the person’s assessable income without any deductions or
Rs. 1,000.
 A withholding agent who fails to file as withholding return shall be liable to pay a charge
for each month and part of month during which the failure continues calculated as 2.5 %
p.a. on tax amount required to be withheld from payment made by agent during the
month to which the failure relates.

2) Interest for understating Estimated Tax Payable by Installment


As per section 118, provisions for interest for understating estimated tax payable by
installment are as follows:
 The person shall be liable to pay the interest @ 15 % on the excess amount if the amount
of each installment paid by the person for an income year is lower than 90 % of amount
to be paid on installment calculated on the actual tax payable by the person.
Naresh Singh Bhandari
bnsingh5102@gmail.com Page123
Income Tax Note 2076/77

 A person shall be liable to pay the interest as above for each month and part of a month
for a period from due date of an installment to the following date;
 In case of person making self-assessment – up to the date of return filing.
 In case of person tax assessed for the first time as amended tax assessment by reason
of tax not assessed under self-assessment - up to the date of notice served for revised
tax assessment by Department under section 102.

Example: A Ltd. has a taxable income of Rs. 600,000 during the year 2068/69. It had paid
advance tax of Rs. 100,000 on 25th Ashadh 2069. The return was filed on 25th Ashwin 2069
after paying the balance tax and interest. How much amount the A Ltd. has to pay as interest
under the Income Tax Act? CAP III, Dec 2013 (2 Marks)
Solution:
A Ltd. has not paid the advance tax as per Sec. 118(2):

Tax Paid Balance Interest@15%


Poush end (40%) 60,000 0.00 60,000 2,250.00
Chaitra end (70%) 105,000 0.00 105,000 3,937.50
Ashadh end (100%) 150,000 100,000 50,000 1,875.00
Total Interest u/s 118 8,062.50

3) Interest for Failure to Pay Tax


As per section 119, provisions related to interest for failure to pay tax are as follows;
 A person who fails to pay tax on or before the date on which the tax is payable is liable to
pay interest for each month and part of month for which any tax is outstanding calculated
@ 15% applied to the amount outstanding.
 The extension granted for payment of tax under section 98 is ignored for the purpose of
calculating interest as above.
 A person who fails to collect advance tax under 95(ka) and deduct withholding tax under
90(4) may not recover interest payable from the person required to deposit advance tax
and from the with-holdee.
 If the agent of the non-resident fails to pay the tax on or before the date is payable, the tax
arrears will be recovered along with additional interest @ of 5%.

Example: My Life Pvt. Ltd. has submitted Income Tax Return for Fiscal Year 2067/68 on
2068 Magh 10 in related IRO. Total taxable income for Fiscal Year 2067/68 was Rs. 20 lacs
and the tax liability as per self-tax assessment is Rs. 5 lacs. My Life Pvt. Ltd. has deposited
advance tax of Rs. 100,000, Rs. 100,000 and Rs. 180,000 on Poush 2067, Chaitra 2067 and
Ashadh 2068 respectively. Total amount deposited was Rs. 380,000. Balance of Rs. 120,000
was deposited along with Income Tax Return in the month of Magh 2068.
Naresh Singh Bhandari
bnsingh5102@gmail.com Page124
Income Tax Note 2076/77

Calculate the interest to be paid as per section 118 (under estimating advance tax payments)
and 119 (delay in payment of tax) of Income Tax Act, 2058. CAP III, June 2012 (8 Marks)
Solutions:
Interest as per section 118

Time period for Installment to 90% of Amount Amount less


deposit be deposited Installment Deposited deposited
2067 Poush end (40%) 200,000 180,000 100,000 100,000
2067 Chaitra end (70%) 350,000 315,000 200,000 150,000
2068 Ashad end (100%) 500,000 450,000 380,000 120,000

Since, less than 90% of the installment is deposited interest @ 15% (Standard rate of interest)
shall be levied on difference amount up to the date when tax return is to be submitted i.e.
Ashwin end 2068. Calculation is as follows:

Time period for Amount less Period for which Interest (Rs.)
deposit deposited interest to be paid
2067 Poush end (40%) 100,000 3 month 3,750
2067 Chaitra end (70%) 150,000 3 month 5,625
2068 Ashad end (100%) 120,000 3 month 4.500
Total Interest 13,875

Interest as per section 119

Particulars
Time limit for submission of income tax return Ashwin End 2068
Amount to be deposited = 500,000 - 380,000
= 120,000
Income tax return submitted and tax deposited Magh 10, 2068
Time period for Interest 4 Month
Rate of Interest 15%
Interest = 120,000 *15*4/12
= Rs. 6,000

Thus, My Life Pvt. Ltd. needs to pay Rs. 9937.85 interest under section 118 & Rs. 6,000 under
Section 119.

Note: The question is solved as per the procedure given by Income Tax Manual 2066 (Updated 2068)
which is different from the answer published by ICAN.

Naresh Singh Bhandari


bnsingh5102@gmail.com Page125
Income Tax Note 2076/77

4) Charge to be levied
As per section 19(ka), a person not following the provisions of this act and the rules there
under except for those provisions specifically mentioned in the act shall be liable to pay fine
of Rs. 5,000 to Rs. 25,000.

5) Penalty for Making False or Misleading Statements


As per section 120, a person who makes a statements to the department that is false or
misleading in a material particular; or omits from a statements made to the department any
matter or thing without which the statement is misleading in a material particular, shall be
liable for penalty as follows:
 Where the statement is happened to be false or misleading without knowingly or
recklessly, 50% of the underpayment of tax; or
 Where the statement is made false or misleading knowingly or recklessly, 100% of the
underpayment of tax.

6) Charges for Aiding and Abetting


As per section 121, a person who knowingly or recklessly aids or abets another person to
commit an offence of a type referred to in this Act, or counsels or induces another person to
commit such an offence shall be liable for a penalty equal to 100 % of the underpayment of
that tax.

Naresh Singh Bhandari


bnsingh5102@gmail.com Page126
Income Tax Note 2076/77

Offences and Penalties

1) Offence of Failure to Pay Tax


As per section 123, if any person who without reasonable excuse fails to pay any tax on or
before the date on which the tax is payable shall be liable on conviction to a fine of not less
than Rs. 5,000 and not more than Rs. 300,000 or an imprisonment for a term not less than
one month and not more than three months, or both.

2) Offence of Making False or Misleading Statements


As per section 124, a person who makes a statement to the department that is false or
misleading in a material particular, or omits from a statement made to the department any
matter or thing without which the statement is misleading in a material particular, shall be
liable to a fine of not less than Rs. 40,000 and not more than Rs. 160,000 or an imprisonment
for a term of not less than 6 months and not more than 2 years or both.

3) Offence of Impeding and Coercing Tax Administration


As per section 125, provisions related to offence of impeding and coercing tax administration
are as follows;
 A person committing the following offences shall be liable to a fine of not less than Rs.
5,000 and not more than Rs. 20,000 or an imprisonment for a term of not less than one
month and not more than 3 months or both;
 Obstructs an officer of the Department acting in the performance of duties under this
act,
 Fails to comply with a notice to obtain information under section 83 or
 Otherwise impedes the enforcement off the act.
 A person who attempts to commit the offences as above shall be liable to a half of the
penalty mentioned in that subsection.

4) Offences by Authorized and Unauthorized Persons


As per section 126, provisions related to offences by authorized and unauthorized persons are
as follows;
 Any authorized person violating official confidentiality shall be liable to a fine of not
more than Rs. 80,000 or an imprisonment for a term off not more than one year, or both.

Naresh Singh Bhandari


bnsingh5102@gmail.com Page127
Income Tax Note 2076/77

 Any person who is not being an authorized under this act, collects or attempts to collect an
amount of tax payable under this act shall be liable to fine or not less than Rs. 80,000 and not
more than Rs. 240,000 or an imprisonment for a term of not less than 1 year and not more
than 3 years or both.

5) Offences of Aiding or Abetting


As per section 127, any person, who knowingly aids or abets another person to commit an
offence under this act or counsels or induces another person to commit such an offence, shall
be liable to a half of the penalty that is imposed on the main officer.
But, government officials to commit such an offence shall be liable to the full penalty that is
imposed on the main offender.

6) Offence of Failure to Comply with Act


As per section 128, except as otherwise provided in this act, any person who fails to comply
with any provisions of this act and the rules under this act, shall be liable to a fine of not less
than Rs. 5,000 and not more than Rs. 30,000.

Naresh Singh Bhandari


bnsingh5102@gmail.com Page128

You might also like